Sunteți pe pagina 1din 134

Universidad La Salle.

Facultad Mexicana de Medicina.


Curso de Extensión Universitaria para la Preparación del Examen Nacional para Aspirantes a
Residencias Médicas.
Examen de Diagnóstico.
3 de febrero 2010.
Modalidad a Distancia.

1.- Se trata de paciente masculino de 66 años con antecedentes de tabaquismo positivo consumiendo mas de
30 cigarrillos al día desde hace mas de 40 años, hipertensión arterial descontrolada, acude a consulta por
presentar hematuria franca y alteraciones en la función renal, a la exploración física se aprecia masa
abdominal en flanco derecho se realiza una TAC demostrando masa sólida de aproximados 9 cm. de
diámetro en riñón derecho. ¿Cuál es el diagnóstico presuncional más probable?

a) Nefroblastoma.
b) Adenocarcinoma.
c) Carcinoma epidermoide.
d) Liposarcoma.

La incidencia de cáncer de riñón ha aumentado sustancialmente en el mundo. El carcinoma de células renales


(CCR) representa 80 a 85% de todos los tumores malignos de riñón. Varios estudios epidemiológicos indican
que el tabaquismo constituye un importante factor de riesgo. Un estudio reciente mostró que dejar de
fumar se asocia con una reducción lineal del riesgo de CCR. El tabaquismo pasivo ha sido relacionado con
varias consecuencias sobre la salud incluyendo cáncer (de pulmón, estómago y mama). Además,
recientemente se ha establecido la asociación causal entre tabaquismo activo y cáncer de riñón. El cigarrillo
parece ser el factor de riesgo más importante para cáncer de riñón y explica la mayoría de los casos en casi
todo el mundo.

Presentación clínica

La mayoría de los casos de carcinoma renal presenta una tríada clásica de hematuria, dolor abdominal en el
flanco y una masa palpable. La hematuria gruesa se encuentra en un 45% de los pacientes. Un 40% de ellos
presentan un cuadro clínico llamado síndrome paraneoplásico que comprende anemia, dolor óseo,
hipercalcemia, fiebre, pérdida de peso.

La evaluación inicial de un paciente con cáncer renal debe incluir un buen historial y examen físico,
laboratorio de rutina y una tomografía computarizada (CT scan) del abdomen y la pelvis, así como una
radiografía de pecho. Si hay evidencia o sospecha de trombosis de la vena cava o trombosis tumoral venosa
(que ocurre en un 5% a 10% de los casos), se recomienda hacer una resonancia magnética (MRI).

En general, no se recomienda realizar una biopsia percutánea de la masa renal, a menos que se sospeche que
se trate de una metástasis de otra neoplasia, como cáncer de colon o linfoma, entre otros.

La resección quirúrgica sigue siendo el tratamiento de elección para los tumores localizados, ya que se sabe
que dicha neoplasia es resistente a la quimioterapia y a la radioterapia.

1. Klein EA, RM Bukowski, JH Finke: Renal Cell Carcinoma. Immunotherapy and cellular Biology. MARCEL
DEKKER, Inc. 1993.

2. RE Lenhard, Jr, RT Osteen, T Gansler: The American Cancer Society’s Clinical Oncology. American Cancer
Society, 2001.

3. 2008 Genitourinary Cancers Symposium: A Multidisciplinary Approach. February 14-18 2008.

4. Managing the Complex Journey of Renal Cell Carcinoma. Institute for medical Education & Research.
March 2008.

5. Highlights of the NCCN 13th Annual Conference. Clinical Practice Guidelines & Quality Cancer Care.
March 5-9, 2008.

6. JS Lam, J Bergman, A Breda, P Schulam: Importance of Surgical Margins in the Management of Renal
Cell Carcinoma. Nature Clinical Practice Urology, Medscape.com. May 13, 2008.

7. R. Nelson: Renal cell carcinoma is being increasingly diagnosed at early stages. Medscape Medical News.
Medscape.com. May 22, 2008.

2.- Paciente masculino de 40 años de edad, empleado en una fábrica, homosexual, con infección por VIH
diagnosticada hace 2 años, acude al servicio de urgencias por presentar fiebre de 15 días de evolución,
ataque al estado general, pérdida de peso, disnea progresiva y tos productiva. Se realizó tinción de la
expectoración, que reportó bacilos ácido-alcohol resistente. Su cuenta de CD4 realizada hace 1 mes fue de
100 cél/mL y una carga viral reciente fue de 250,000 copias. ¿En qué estadio CDC se encuentra el paciente?

a) B3
b) C3
c) B2
d) A3
El estadio C3 está definido por una cuenta de CD4 menor de 200 cél/mL y la presencia de una condición
clínica indicativa de Síndrome de Inmunodeficiencia Adquirida, entre las que se encuentra la infección por
Mycobacterium tuberculosis en cualquier variedad.

CLASIFICACION DE INFECCION VIH


VIGILANCIA, ADULTOS Y ADOLESCENTES
CDC 1993
A B C
Categoría CD4 Asintomático Sintomático* Indicador de
Infxn aguda (no A o C) SIDA

> 500/mm3 A1 B1 C1
200-499/mm3 A2 B2 C2
< 200/mm3 A3 B3 C3

* muguet oral, candidiasis vaginal persistente, displasia cervical, fiebre-diarrea


de >1mes, leucoplasia vellosa, zoster (2 episodios o >1 dermatoma), PTI, PID

NO TOMA EN CUENTA LA CARGA VIRAL


5

1. Kasper DL, Braunwald E, Fauci AS, Hauser SL, Longo DL, Jameson JL. Harrison´s Principles of Internal
Medicine. McGraw Hill. 16 Ed. 1076 p.

3.- Ante una paciente de 35 años que acude a consulta con antecedentes de G3 C2 A1, refiere que ha
presentado durante el primero y segundo trimestres de su embarazo manchado con frecuencia
intermitente, a las 34. SDG inicia con hemorragia abundante, repentina e indolora, su principal sospecha es:
a) Coriocarcinoma.
b) Ruptura uterina.
c) Placenta previa.
d) Desprendimiento grave de placenta normoinserta.

PLACENTA PREVIA
DEFINICIÓN:
Es cuando la placenta se implanta sobre o muy cerca del orificio cervical interno y una parte de la placenta
precede a la parte fetal que se presenta.
INCIDENCIA:
Esta es difícil determinar ya que muchos casos pasan desapercibidos, sobre todo cuando ocurren los abortos
en embarazos tempranos.
La prevalencia varía de 1 en 100 a 1 en 850 nacidos vivos, pero solo el 20% total.
ETIOLOGÍA:
Edad avanzada, multíparas, paciente con cesáreas previas, paciente con aborto de repetición, esto debido a
las gestaciones previas.
TIPOS:
Inserción baja.- Es cuando el borde placentario se encuentra en el segmento inferior a menos de 6 cm. del
orifico cervical interno.
Marginal.- Es cuando el borde placentario alcanza los márgenes del orificio cervical interno.
Parcial.- Es esta la placenta cubre parcialmente el orificio cervical interno.
Total.- La placenta cubre la totalidad del orificio cervical interno aún con dilatación cervical avanzada.

DIAGNOSTICO:
La característica es el STV de aparición brusca en forma indolora en el segundo o tercer trimestre.
Frecuentemente hay ausencia de dolor a actividad uterina que son parámetros para hacer el diagnóstico.
La mayor incidencia de sangrado aparece a las 33-34 SDG.

ESTUDIOS DE GABINETE:
El estudio mas utilizado es la ultrasonografía obstétrica.

TRATAMIENTO:

El manejo va a depender de factores como son:


Edad gestacional, magnitud del sangrado, si hay trabajo de parto, variedad de placenta previa y
complicaciones materna.
BIBLIOGRAFÍA:
1.- Waxler P, Gottesfeld KR. Early diagnosis of placenta previa. Obstet Gynecol 1979;54:231-32.
2.- Cabrero-Roura L. Riesgo elevado obstétrico. Ed. Masson 1996; pp; 109-118.
3.- Patrick J, Placenta Previa, Clinical Obst and Gynecology 1990;33(3): 414-421.
4.- Chapman M, Furtenes ET, Significance of ultrasound in location of placenta in early pregnancy Br J Obst
Gynecol 197;86: 846. 57

4.- Se trata de paciente femenino de 22 años, que presenta lesiones eritematoescamosas, edema y alguna
vesicula en la cara, escote, dorso de las manos y antebrazos. Las lesiones tienen 12 horas de evolución y han
aparecido tras una escursión al campo. Entre los antecedentes personales destaca acné vulgar en
tratamiento con retinoides tópicos y doxiciclicina oral. El diagnóstico más probable es:
a) Erupción lumínica poliforma.
b) Eritrodermia por fármacos.
c) Urticaria solar.
d) Reacción fototóxica.
DEFINICIÓN
Enfermedades cutáneas que se producen por el aumento de capacidad de reacción de la piel a las radiaciones
lumínicas tras la administración de una sustancia fotosensibilizante. Se conocen como reacciones de
fotosensibilidad y pueden desencadenarse tanto por contacto como por la administración sistémica del
agente fotosensibilizante. Si existe implicación inmunológica se denomina dermatitis fotoalérgica y si no
dermatitis fototóxica.

Dermatitis fototóxica
No existe un mecanismo inmunológico, puede afectar a muchas personas siempre que exista dosis elevada de
irradiación y cantidad suficiente de sustancia química. Las lesiones aparecen tras la primera exposición, son
monomorfas, y se caracterizan por eritema intenso, edema y vesiculación en áreas de piel fotoexpuestas,
marcando claramente los bordes de las zonas descubiertas, y onicolisis ungueal . Formas particulares de
fototoxia: fitofotodermatitis (dermatitis de los prados, apio) (Fig. 3), dermatitis de Berloque,
fotosensibilidad en tatuajes (sulfuro de cadmio), fármacos (tetraciclinas, AINEs, amiodarona (color
azulado), clorpromacina (color gris…).

Figura 3. Fitofotodermatitis.

1. Litt Jz. Drug eruption reference manual 2001. New York: Parthenon, 2001.
2. Sullivan JR, Shear NH. Drug eruptions and other adverse drug effects in aged skin. Clinics in geriatric
medicine 2002;18(1).
3. Lim HW, Gigli. Complement-derived peptides in phototoxic reaction. En: Daynes RA, Spikes JD, editors.
Experimental and clinical photoimmunology. Boca Raton: CRC Press, 1983:81-93.
4. Torinuki W, Tagami H. Role of complement in chlorpromazine-induced phototoxicity. J Invest Dermatol
1986;86:142-4.
5. Hearst JE, Issacs ST, Kanne D, Rapoport H, Straub K. The reaction of the psoralens with
deoxyribonucleic acid. Q Rev Biophys 1984;45:891-5.
6. Athar M, Elmets CA, Bickers DR, Mukhtar H,. A novel mechanism for the generation of superoxide anions
in hematoporphyrin derivative-mediated cutaneous photosensitization. Activation of the xantine oxidase
pathway. J Clin Invest 1989;83:1137-43.
7. Matsuo I, Inukai N, Fujita H, Ohkido M. Possible involvement of oxidationmof lipids in inducing
griseofluvin photosensitivity. Photodermatol Photoimmunol Photomed 1990;7:213-7.
8. Harber LC, Bickers DR. Photosensitivity diseadses. Principles of diagnosis and treatment. Ontario: BC
Decker Inc, 1989:160-202.
9. Kockevar IE. Phototoxicity of nonsteroidal inflammatory drugs. Coincidence or specific mechanism?. Arch
Dermatol 1989;125:824-6.

5.- Femenino de 39 años de edad la cual inicia con un cuadro de exoftalmos axial de varias semanas de
evolución, con predominio en su ojo derecho. En la exploración se aprecia una conjuntiva con síntomas
discretos de hiperemia y edema, una queratitis de carácter punteado en tercio inferior corneal y se
sospecha una retracciónpalpebral al observar cómo el borde del párpado superior se encuentra por encima
del limbo, permitiéndonos visualizar la esclerótica. La paciente no refiere disminución de visión ni
alteraciones tipo visión doble y toma presión intraocular con parámetros dentro de los normales. ¿Cuál de los
siguientes diagnósticos le parece más compatible con el cuadro?:

a) Tumor intraorbitario.
b) Enfermedad de Graves-Basedow.
c) Queratoconjuntivitis epidémica.
d) Tumor intraocular.

OFTALMOPATÍA TIROIDEA.
Es la causa más frecuente de exoftalmos tanto bilateral como unilateral en adultos. La forma típica aparece
en pacientes con enfermedad de Graves-Basedow, en los que podemos encontrar exoftalmos y síndrome
palpebro-retráctil. Este síndrome ocular puede aparecer en pacientes eutiroideos o hipotiroideos, pudiendo
constituir el signo más precoz de una tirotoxicosis incipiente.

CLÍNICA.
Se distinguen dos formas clínicas:
a) Tirotóxica (hipersensibilidad a las catecolaminas y habitualmente hay hipertiroidismo): exoftalmos
moderado depresible.
Hay edema del contenido orbitario, pero no fibrosis ni oftalmoparesia.
b) Maligna (inflamación orbitaria autoinmune y puede haber normo o hipotirodismo): exoftalmos irreductible
severo con oftalmopejia progresiva y queratitis por exposición. Puede afectar al nervio óptico por
compresión y producir pérdida visual. Hay fibrosis de la grasa y vientres musculares.
Manifestaciones oculares asociadas: retracción palpebral bilateral que permite ver la esclera por encima
del limbo, disminución de la motilidad palpebral, alteración de la pigmentación palpebral, hiperemia
conjuntival.
DIAGNÓSTICO.
Por los signos clínicos descritos y exploraciones complementarias, como la exoftalmometría (medida de la
protrusión ocular), la radiología (aumento de densidad de los tejidos blandos), el engrosamiento del vientre
de algunos músculos extraoculares (apreciados en la TC, la RM y la ecografía orbitaria) y la analítica
sistémica

1.-AACE Thyroid Task Force. American Association of Clinical Endocrinologists medical guidelines for
clinical practice for the evaluation and treatment of hyperthyroidism and hypothyroidism. Endocr Pract.
2002;8(6).

2.-Davies TF, Larsen PR. Thyrotoxicosis. In: Kronenberg HM, Melmed S, Polonsky KS, Larsen PR, eds.
Williams Textbook of Endocrinology. 11th ed. Philadelphia, Pa: Saunders Elsevier; 2008:chap 11.

3.-Ladenson P, Kim M. Thyroid. In: Goldman L, Ausiello D, eds. Cecil Medicine. 23rd ed. Philadelphia, Pa:
Saunders Elsevier; 2007:chap 244.
6.- ¿Para el diagnóstico radiológico de sinusitis aguda, la proyección que mejor valora los senos maxilares y
las estructuras intranasales es?

a) Lateral.
b) Submentoniana.
c) Waters.
d) Anteroposterior.

La proyección Waters es la proyección que mejor permite valorar la neumatización, opacificación o


engrosamiento de mucosa de los senos maxilares así como las estructuras intranasales.

Proyección de Waters u occipito-mentoniana para senos maxilares (Radiografía normal).

González-Saldaña N, Infectología Clínica Pediátrica, 7ª edición, páginas 63-98.

7.- Se trata de paciente femenino de 39 años la cual presenta amenorrea secundaria de 2 años y medio de
evolución. Los niveles reportados de prolactina son de 150ng/ml (normal hasta 20 ng/ml). La resonacia
magnética detecta macrotumor de 2,8 cm. de diámetro con expansión lateral izquierda. Sin alteraciones
visuales. ¿Cuál sería el tratamiento de elección?:

a) Cirugía por tratarse de un macrotumor.


b) Tratamiento médico con agonistas dopaminérgicos.
c) Somatostatina previa a cirugía.
d) Radioterapia hipofisaria previa a cirugía.
TRATAMIENTO:
Los dopaminérgicos han revolucionado el tratamiento del prolactinoma y virtualmente han dejado fuera a la
cirugía; así independientemente del tamaño del adenoma la primera opción terapéutica es la
farmacológica.1,2 Con los dopaminérgicos se consigue en poco tiempo restaurar el funcionamiento ovárico y
corregir la esterilidad, incluso antes de que se normalice la concentración de prolactina; asimismo se
consigue reducir el tamaño del adenoma.
Los dopaminérgicos actúan sobre los receptores localizados en las células mamotrópicas de la hipófisis
anterior y suprimen la síntesis y secreción de prolactina con la consecuente normalización del eje
hipotálamogonadotropico hipotálamogonadotropico- ovárico. La acción dopaminérgica puede ocasionar
efectos colaterales indeseables como náusea, hipotensión arterial, constipación nasal, mareo y
estreñimiento, los cuales no necesariamente corresponden con la dosis utilizada, pero sí se relacionan con el
tipo de dopaminérgico. Está ampliamente documentada la superioridad de la farmacoterapia para el
tratamiento de los prolactinomas; además la cirugía es raramente curativa, incluso en el caso de
microadenoma.

Cuadro I. Agentes dopaminérgicos que se usan como tratamiento de la hiperprolactinemia y el


prolactinoma.
Genérico. Comercial Dosis (mg)
Bromocriptina. Parlodel 2.5-5 diaria.
Lisurida . Dopergin 0.2 diaria.
Quinagolida . Norprolac 25-50 diaria.
Cabergolina . Dostinex 0.5 c/4 días.

Los adenomas hipofisarios representan el 10% de todos los tumores intracraneales diagnosticados y 25% de
los tumores cerebrales que son intervenidos quirúrgicamente. Los objetivos del tratamiento de un paciente
con un adenoma de la pituitaria son: eliminar el efecto de la masa tumoral (compresión sobre estructuras
vecinas) disminuir la producción excesiva de hormonas, restaurar la función normal de la pituitaria y evitar
la recurrencia.
El tratamiento de elección para todos los prolactinomas es con un agonista de la dopamina. La bromocriptina
y la cabergolina son efectivas para reducir el tamaño del tumor y para restaurar la función gonadal. El
tratamiento quirúrgico debe recomendarse sólo cuando falla el tratamiento médico.
Los tumores de la pituitaria productores de hormona de crecimiento son tratados preferentemente
mediante adenomectomía transesfenoidal, pero la normalización de los niveles de HC y de IGF-1 ocurre en
menos de la mitad de los pacientes con macroadenomas; por lo tanto, un importante número de pacientes
acromegálicos requiere un tratamiento adicional. Los análogos de la somatostatina son en la actualidad los
medicamentos que más usados para el control de la acromegalia. En grupos especiales de pacientes, el
tratamiento con agonistas de la dopamina y somatostatina parece que suprimen mejor los niveles de HC que
cuando se administran esos fármacos en forma separada
Figura 1. Paciente de 20 años quien consultó por amenorrea primaria. El estudio de RMN muestra un
macroprolactinoma que invade el seno cavernoso izquierdo y envuelve la carótida del mismo lado.

RMN de control 10 meses después de tratamiento con un agonista dopaminérgico. Corte coronal en
T1. No se observa tumor. Tallo hipofisario central y quiasma óptico libre.

REFERENCIAS BIBLIOGRÁFICAS:

1. Gac Méd Méx Vol. 140 No. 5, 2004.


2. Schlechte JA. Prolactinoma. N Engl J Med 2003;349:2035-2041.
3. Zárate A, Canales ES, Jacobs LS, Soria J, Daughaday WH. Restoration of ovarian function in patients
with the amenorrhea-galactorrhea syndrome after long-term therapy with L-Dopa. Fertil Steril
1973;24:340.
4. Tyson JE, Carter JN, Andreassen B, Huth J, Smith B. Nursing mediated.
5. Ezzat,Shereen, Asa,Silvia : Current views on pathogenesis of pituitary tumors. Curr Opin Endocrinol Diab
2004;11:281-286.
6. Vance Mary Lee. Medical treatment of funtional pituitary tumors: Neurosurg Clin NA 2003;14:81-87.
7. Missale C, Losa M, Boroni F, Giovanelli M, Balsari A, Spano PF : Nerve growth factor and bromocriptine: a
sequential therapy for human bromocriptine – resistant prolactinomas . Brit J Cancer 1995; 72: 1397-9.
8.- Paciente de 25 días de nacido al cual se desprendió el cordón umbilical a los 9 días y cuyo ombligo no ha
cicatrizado. Presenta en este nivel una tumoración roja, prominente, circular y que segrega un contenido
alcalino. Refiere la madre que a veces nota en esta tumoración ruido de gases. El diagnóstico más probable
será:

a) Fístula vésico-umbilical.
b) Granuloma umbilical.
c) Persistencia del conducto onfalomesentérico.
d) Onfalocele.

Durante la vida fetal el conducto onfalomesentérico une al saco vitelino con el intestino medio y se cierra
normalmente para desaparecer por completo. Se encuentra conectado con el intestino primitivo en el saco
amniótico. En el desarrollo embriológico normal, el conducto onfalomesentérico involuciona entre las 5a y 7a
semanas de vida intrauterina. Un fracaso en la regresión produce varias anormalidades, en dependencia del
lugar donde se localice este fallo: en el lado umbilical o en el intestinal.

Los vestigios del conducto onfalomesentérico (vitelino) pueden presentarse como anomalías relacionadas con
la pared abdominal. Sin embargo, puede ocurrir que todo o parte del conducto fetal se mantenga y entonces
se produzca sintomatología clínica. También puede persistir como una estructura permeable en toda su
longitud o mantenerse como un divertículo o quiste cuando persiste en su periferia, parte central o media; o
quizás quede representado simplemente por un resto de epitelio intestinal ectópico a nivel umbilical o como
cordón fibroso.

La explicación embriológica de la persistencia del conducto onfalomesentérico puede explicarse diciendo que
el conducto onfaloentérico o vitelino que representa una comunicación entre el vértice del asa umbilical
fetal y el saco vitelino se oblitera y desaparece junto con sus vasos acompañantes arteria y vena
onfalomesentéricas hacíanla séptima semana-fetal. Sin embargo, cuando este proceso no ocurre, da lugar a
la patología conocida de las estructuras remanentes del conducto onfalomesentérico.

Después de los cuidados iniciales en la edad del recién nacido que se han dado al cordón umbilical y una vez
que éste se ha desprendido, esta zona recibe poca atención en los días subsiguientes; sin embargo, puede
ser asiento de muchas lesiones, tanto congénitas como adquiridas, con las cuales el médico debe estar
familiarizado para poder diagnosticarlas.
El diagnóstico se hace mediante la observación, cuando el recién nacido realiza esfuerzos como el provocado
por el llanto, pues se produce la salida de material fecal o gases por el ombligo.
REFERENCIAS
1. Pomeranz A. Anomalies, abnormalities, and care of the umbilicus. Pediatr Clin N Am 51 (2004)819.
2. Rowe M, Chapter 48: Disorders of the umbilicus. Essentials of Pediatric Surgery, 1995.
3. Care of the umbilical cord, a review of the evidence. OMS. 1998.
4. Cushing A. Omphalitis: a review. Pediatric Infect Dis J 1985, 4(3); 282-285.
5. Janssen P. To dye or not to Dye: A randomized, Clinical Trial of a Triple Dye/alcohol Regime Versus Dry
cord care. Pediatrics 2003; 111, 15-20
6. Zupan J. Topical umbilical cord care at birth The Cochrane Database of Systematic
Reviews,2004.Issue3.
7. Sawardekar K. Changing spectrum of neonatal omphalitis. Pediatric Infect Dis J, 2004;23:22-6.
8. Soto G. Anomalías uracales: un diagnóstico rara vez planteado. Pediatría al día 2001;17(5) 331)
9. Cilento BG Jr. Urachal anomalies: defining the best diagnostic modality. Urology 1998, 52 (1)120-2.
10. Ossandón F. Malformaciones congénitas: patología umbilical. Pediatría al día 1995;11(2)
11. Lotan G. Double ligature: A treatment for pedunculated umbilical granulomas in children.
Am.Fam.Physician2002;65:2067-8.
12. Novoa A. El pediatra ante un lactante con caída tardía del cordón umbilical. Arch arg
Pediatr2004;102(3).

9.- Las siguientes manifestaciones clínicas son las más frecuentes en el lupus:

a) Las cutáneas y las articulares.


b) Las cardíacas.
c) Las pulmonares.
d) Las musculares.
Después de la fatiga, la fiebre, la astenia, las alteraciones de la piel, el pelo y las mucosas ocupan el
segundo lugar entre las manifestaciones clínicas (85% de los casos).

El clásico eritema facial en alas de mariposa ocurre en 52% y es consecuencia frecuente de


fotosensibilidad; también se localiza en el tórax, espalda y brazos como lesiones eritematosas simétricas
superficiales con zonas centrales atróficas anulares.

El lupus eritematoso discoide se localiza en la piel cabelluda, pabellones auriculares, cara y cuello y se asocia
con frecuencia a fotosensibilidad y fenómeno de Raynaud.

REFERENCIA BIBLIOGRÁFICA:

1.- GUTIÉRREZ C, MOZO L. Diagnóstico inmunológico: enfermedades auto inmunes.


En: Inmunología
Clínica. Bases moleculares y celulares. J. Peña, ed. Madrid. Arán. 107-118, 1996.

10. – Femenino de 26 años G2 P1. C1, acude al servicio médico por referir, una secreción transvaginal
bastante líquida, de baja viscosidad, maloliente de color amarillo y gris espumosa. Esta entidad es propia de
infección por:

a) Cándida albicans
b) Gardnerella
c) Tricomonas
d) Gonococos

Tricomoniasis.

Es causada por el Trichomonas vaginalis, parásito protozoario unicelular de cola tipo látigo que utiliza
para impulsarse a través del moco de la vagina y la uretra. ETS que se presenta a nivel mundial.

Síntomas de la tricomoniasis

Los síntomas de la enfermedad son bastante diferentes en hombres y mujeres:


En los hombres, la infección es asintomática(no tiene síntomas) y desaparece espontáneamente en algunas
semanas. Pero en caso de presentar síntomas, son los siguientes:

• Picazón uretral leve o secreción.


• Ardor ligero después de orinar o eyacular.
• Pueden presentar prostatitis o epidimitis por la infección.

En las mujeres:

• Secreción vaginal espumosa blanca-verde o amarillenta, de olor fétido.


• Cantidad considerable de secreción que ocasiona picazón en los labios y parte interna de los muslos y
labios, parte que también pueden inflamarse.
• Incomodidad durante el coito.

Es una infección frecuente, más de la mitad de mujeres con gonorrea también presentan tricomonas, ya que
con frecuencia las personas con una enfermedad de transmisión sexual diagnosticada, tienen una o más
infecciones transmitidas sexualmente.

REFERENCIA BIBLIOGRAFICA:

1.- Krieger JN and Alderete JF. Trichomonas vaginalis and trichomoniasis. In: K. Holmes, P. Markh, P.
Sparling et al (eds). Sexually Transmitted Diseases, 3rd Edition. New York: McGraw-Hill, 1999, 587-604.

11.- Un varón de 47 años de edad con cirrosis hepática tuvo dolor abdominal generalizado durante 24 h sin
náuseas ni vómitos. Su temperatura es de 38.3°C y ha tenido distensión abdominal con onda de líquido claro.
Hay hipersensibilidad difusa en la palpación abdominal. En la paracentesis se obtuvo líquido transparente con
816 leucocitos/mm3 (85% polimorfonucleares, 15% linfocitos). La tinción de Gram no muestra bacterias.
¿Cuál de los siguientes diagnósticos es más probable?

a) Enfermedad ulcerosa péptica.


b) Pancreatitis.
c) Colecistitis.
d) Peritonitis primaria.

El diagnóstico más probable es peritonitis primaria. Aunque es difícil diferenciar la peritonitis primaria
(espontánea) por rotura de víscera hueca y contaminación peritoneal, la presencia de fiebre y la leucocitosis
la ascitis sugiere alguna clase de infección peritoneal. La pancreatitis se caracteriza por dolor localizado
intenso (mesoepigástrico), que se irradia a la espalda. En general, las náuseas y vómitos no se relacionan con
pancreatitis aguda. El dolor abdominal en caso de colecistitis se sitúa en el cuadrante superior derecho y
suele haber náuseas y vómitos. El absceso hepático tiende a ser un trastorno subagudo sin datos
peritoneales prominentes. En caso de ascitis crónica infectada, las enfermedades ulcerosas pépticas son
causa poco probable.
REFERENCIA BIBLIOGRAFICA:

1.- Allen R. M. MMS Medicina Interna. 5ª. Edición. National Medical Series. Mc. Graw Hill. 2006.
(capítulo 8 V E 1 a).

12.- El diagnóstico más probable en un paciente masculino de 17 años de edad que desconoce sus
antecedentes familiares. Desarrolla dolor abdominal progresivo, posterior a un traumatismo directo en la
región lumbar; se documenta un hematoma en el músculo posas izquierdo La Bh con 11 gr. de HB,
reticulocitos 35, LEUCOCITOS DE 13500, 350 mil plaquetas, TTP: 60”/35”, TP: 12”/12”, TT: 26”/26”
fibrinógeno 350 MG/dl. ¿Cuál es su sospecha diagnóstica?

a) Enfermedad de Von Willebrand


b) Hemofilia
c) Trombocitopatía
d) Deficiencia de factor XII

¿Cuáles son los síntomas de la hemofilia?

El síntoma más común de la hemofilia es la hemorragia incontrolable y excesiva por causa del factor de
coagulación que falta o está en bajos niveles en la sangre. Puede producirse una hemorragia incluso cuando
no haya ninguna lesión. La mayoría de las veces se produce en las articulaciones y en la cabeza.

A continuación, se enumeran los síntomas más frecuentes de la hemofilia. Sin embargo, cada individuo puede
experimentarlos de una forma diferente. Los síntomas pueden incluir:

• Equimosis (Moretones).
Pueden producirse moretones por pequeños accidentes, que pueden producir un hematoma grande
(una acumulación de sangre debajo de la piel que causa inflamación).
• Sangra con facilidad.
La tendencia a sangrar por la nariz, la boca y las encías por un traumatismo sin importancia,
cepillarse los dientes y\ o trabajo dental es a menudo una indicación de hemofilia.
• Hemorragia en una articulación.
La hemartrosis (hemorragia en una articulación) puede producir dolor, inmovilidad y, con el tiempo,
deformación si no se trata médicamente de la forma adecuada. ésta es la zona más común de
complicaciones debido a la hemorragia por hemofilia. Estas hemorragias de las articulaciones pueden
producir la artritis crónica y dolorosa, deformaciones y parálisis si se repiten.
• Hemorragia en los músculos.
La hemorragia en los músculos puede causar hinchazón, dolor y enrojecimiento. La hinchazón por el
exceso de sangre en estas zonas puede producir un aumento de la presión en los tejidos y nervios de
la zona, provocando daño y\o deformación permanente.
• Hemorragia por lesiones o hemorragia cerebral.
La hemorragia por lesión o espontánea en el cerebro es la causa más común de muerte en los niños
que tienen hemofilia y la complicación hemorrágica más grave.
• Otras fuentes de hemorragia:
La sangre en la orina o en las heces puede ser también un síntoma de hemofilia.

Los síntomas de la hemofilia pueden parecerse a los de otros trastornos de la sangre o problemas médicos.
Siempre consulte a su médico para el diagnóstico.

1.- Roberts HR. Hemophilia A and Hemophilia B. In: Lichtman, MA, et al., eds. Williams Hematology. 7th ed.
The McGraw-Hill Companies, Inc.; 2006:chap 115.

13.-Femenino de 50 años que manifiesta en 2 años cinco crisis de vértigos rotatorios de entre una y tres
horas de duración, con náuseas y vómitos, diaforesis, plenitud de oído. Nistagmo. Acúfenos en el oído
izquierdo que preceden a las crisis vertiginosas. Hipoacusia del oído izquierdo que se confirma con la
audiometría de tipo neurosensorial con mayor pérdida en las frecuencias graves. A la exploración por micro-
otoscopia, se aprecian conductos auditivos externos y tímpanos normales. La Resonancia Magnética cerebral
con contraste es normal. ¿Qué diagnóstico considera más acertado?:

a) Neurima Vestibular izquierdo.


b) Enfermedad de Menière.
c) Neuronitis vestibular izquierda.
d) Vértigo Postural Paroxístico Benigno.

La enfermedad de Meniere (EM) se define como un cuadro clínico que se caracteriza por la presencia de
episodios espontáneos de vértigo (sensación de giro de objetos) recurrente, hipoacusia fluctuante, acúfeno
y plenitud ótica de origen idiopático. Cuando se relaciona con una etiología concreta se llama Síndrome de
Meniere (SM) o hidrops endolinfático secundario. El marcador histopatológico de la EM es la presencia de un
hidrops endolinfático.

La fisiopatología está en relación con una homeostasis anormal de los fluidos del oído interno, ya sea un
aumento en la producción de endolinfa o un problema para su reabsorción (Kimura, 1967). El exceso de
endolinfa en los espacios del oído interno (laberinto membranoso), provoca un aumento de presión o hidrops
endolinfático que sería el responsable de las microroturas de las membranas del oído interno y de la
posterior mezcla de endolinfa y perilinfa, responsables de las manifestaciones clínicas en la EM (Zenner,
1994).

La causa inicial que desencadena el hidrops endolinfático sigue siendo desconocida. La presencia de hidrops
no es causa suficiente para explicar las manifestaciones de la EM (Honrubia V, 1999; Rauch SD, 2001).
Hasta la fecha se han encontrado asociaciones con: enfermedad autoinmune del tiroides (Brenner M, 2004),
predisposición genética con patrón autonómico dominante (Klockars T, 2007), HLA-DRB1*1101 (López-
Escamez JA, 2007), virus (Vrabec JT, 2003) y migraña (Boyev KP, 2005).

Las causas de hidrops endolinfático secundario o SM son:

Enfermedad autoinmune del oído interno (primaria o secundaria).


Traumatismo acústico.
Otitis media crónica.
Síndrome de Cogan.
Hipoacusia congénita.
Tumores del saco endolinfático.
Fenestración de la cápsula ótica.
Concusión laberíntica.
Enfermedad de Letterer-Siwe.
Leucemia.
Displasia tipo Mondini.
Otosclerosis .
Enfermedad de Paget .
Laberintitis serosa.
Trauma quirúrgico sobre el oído interno.
Sífilis.
Traumatismo cefálico sobre el hueso temporal.
Laberintitis viral.

Diagnóstico

No existe ninguna prueba diagnóstica, ni marcador analítico ni radiológico que identifique la EM. El
diagnóstico se basa en una historia clínica detallada, en la exploración neurotológica, así como en la evolución
clínica y la respuesta al tratamiento.

Los síntomas de la EM inicialmente suelen ser unilaterales y consisten en:

Vértigo rotatorio de al menos 20 minutos de duración asociado con náuseas y vómitos (Saeed, 1998).
Hipoacusia neurosensorial inicialmente fluctuante que afecta a las frecuencias graves del
audiograma, con el tiempo suele ser más severa, permanente y afectar a todas las frecuencias.
Sensación de plenitud ótica o de presión en el oído que puede durar desde minutos hasta horas y con
frecuencia suele preceder al vértigo (Saeed, 1998).
Acúfeno habitualmente de tonalidad grave y que puede acompañarse de distorsión auditiva.

La exploración debe ser completa y excluir otras causas de vértigo, hipoacusia y acúfenos, prestando una
atención especial a enfermedades del sistema cardiovascular, del sistema nervioso central (SNC) y del área
ORL (Warrell, 2003).

Se aconseja estudiar sistemáticamente (CKS, 2007):


• Tensión arterial acostado y de pie.
• La presencia de arritmias y de soplos carotídeos.
• Pares craneales (buscando la presencia de un nistagmo).
• Equilibrio postural y coordinación (Romberg y dedo-nariz).
• Otoscopia en busca de inflamaciones, así como realizar las pruebas de acumetría (Rinne y
Weber).
• Columna cervical para valorar la presencia de dolor o cervicoartrosis que justifiquen un vértigo
cervical.

Es aconsejable realizar un hemograma, glucemia, colesterol, hormonas tiroideas, ANA/ENA (DynaMed,


2007) y FTA-Abs (Pulec JL, 1997) en busca de anemia (CKS, 2007), diabetes mellitus, hipercolesterolemia,
hipotiroidismo, enfermedades autoinmunes y neurosífilis. En un 30% de los pacientes con EM se detectan
anticuerpos frente a antígenos del oído interno sin un claro significado.

La Academia Americana de Otorrinolaringología (AAO-HNS) ha establecido, por consenso, los criterios


clínicos que permiten diagnosticar la EM en:

1. EM cierta o segura: Cumple todos los requisitos clínicos y se ha podido confirmar histológicamente.
2. EM definitiva: Dos o más episodios de vértigo rotatorio de al menos 20 minutos de duración que
asocian hipoacusia fluctuante constatada en la audiometría tonal liminar en alguna ocasión,
acompañándose de acúfeno y sensación de plenitud ótica en el mismo oído. Se han excluido otras
etiologías.
3. EM probable: Una crisis única de vértigo de al menos 20 minutos de duración que se acompaña de
hipoacusia constatada audiométricamente en al menos una ocasión. Puede o no acompañarse de
acúfeno y sensación de plenitud ótica.
4. EM posible: Presenta episodios de vértigo recurrente de al menos 20 minutos de duración pero la
hipoacusia no está documentada o, la hipoacusia está documentada pero no tiene los episodios de
vértigo rotatorio de al menos 20 minutos de duración.
Para considerar una EM bilateral, cada uno de los oídos por separado debe cumplir los criterios
diagnósticos de la AAO-HNS. Habitualmente suele ser un oído el afectado, pero en la evolución
pueden verse involucrados ambos en el 30-50% de los pacientes en un período de 15 años desde el
diagnóstico.

Las pruebas que han demostrado utilidad en el diagnóstico de la EM son:

Audiometría tonal liminar. Es necesaria para confirmar la presencia de hipoacusia. No es


aconsejable realizarla en las crisis agudas y en las fases iniciales de la enfermedad puede ser
normal.
Potenciales Evocados Auditivos del Tronco del Encéfalo (PEATE). Para valorar si la lesión se
localiza a nivel coclear o retrococlear, cuando la forma inicial de presentación es una hipoacusia
progresiva unilateral sin vértigo. Si el resultado corresponde a una hipoacusia retrococlear deberá
ser evaluada mediante RM (Turski, 2006).

Resonancia Magnética (RM). No es diagnóstica pero resulta imprescindible para descartar la


presencia de lesiones del SNC que pueden presentarse como una EM (Lorenzi, 2000):
Tumores (neurinoma del acústico).
Aneurismas o estenosis de la circulación posterior del SNC.
Malformaciones tipo Arnold-Chiari.
Enfermedades degenerativas, esclerosis múltiple.

Pruebas vestibulares. En los períodos iniciales de la enfermedad no aportan información sobre la


función vestibular, pero pueden aportar datos de lesión central (esclerosis múltiple, neurinoma de
acústico). En la evolución de la EM muestran una pérdida de la función vestibular en el oído enfermo.
Son de utilidad para seleccionar candidatos a tratamientos más agresivos o identificar una
enfermedad bilateral.
Bibliografía

• British National Formulary. 53rd ed. London: British Medical Association and Royal Pharmaceutical
Society of Great Britain; 2007.
• Boyev, KP. Meniere's disease or migraine? The clinical significance of fluctuating hearing loss with
vertigo. Arch Otolaryngol Head Neck Surg 2005; 131:457-9 [PubMed]
• Brenner M, Hoistad D, Hain TC. Prevalence of thyroid dysfunction in Meniere's Disease. Arch
Otolaryngol Head Neck Surg 2004; 130: 226-228. [PubMed] [Texto completo]
• Celestino D, Ralli G. Incidence of Meniere’s disease in Italy. Am J Otology 1991; 12; 135-8.
[PubMed]
• Chia SH, Gamst AC, Anderson JP, Harris JP. Intratympanic gentamicin therapy for Meniere's
disease: a meta-analysis. Otol Neurotol 2004; 25:544. [PubMed]
• CKS. Meniere disease whole view. [internet]. NHS; 2008 [acceso 28/2/2008]. Disponible en:
http://www.cks.library.nhs.uk/menieres_disease/
• Cohen-Kerem R, Kisilevsky V, Einarson TR, Kozer E, Koren G, Rutka JA . Intratympanic gentamicin
for Meniere's disease: a meta-analysis. Laryngoscope 2004; 114:2085. [PubMed]
• Committee on Hearing and Equilibrium. Committee on Hearing and Equilibrium guidelines for the
diagnosis and evaluation of therapy in Meniere’s disease. Otolaryngol Head Neck Surg 1995; 3: 181-
185. [PubMed]
• Conlon BJ, Gibson WP. Electrocochleography in the diagnosis of Meniere's disease. Acta Otolaryngol
2000; 120:480. [PubMed]
• DynaMed. Meniere's disease [Internet]. EBSCO Publishing; 2007 [acceso 21/02/2008].
• Filipo R, Barbara M. Natural history of Menière's disease:staging the patients or their
symptoms?. Acta OtolaryngolSuppl.1997;526:10-3.

14.- En un paciente del sexo masculino de 35 años de edad con una dermatosis diseminada a codos y rodillas
con placas eritemato-escamosas ¿Cuál de los siguientes diagnósticos es el de mayor probabilidad?

a) Dermatitis atópica.
b) Dermatitis seborreica.
c) Psoriasis.
d) Dermatitis de contacto.

La psoriasis es una enfermedad crónica que evoluciona en brotes de causa desconocida, que se caracteriza
por placas eritemato escamosas en diferentes partes de la piel. Se presenta por igual en hombres y en
mujeres, en todas las edades, predominando en jóvenes y más frecuentemente en personas de piel blanca
(parece que la presencia de melanina protege contra la enfermedad.

También es ampliamente reconocido que la psoriasis es una enfermedad familiar y hereditaria (diátesis
psoriásica).
Los sitios de predilección para que aparezcan las lesiones, son los salientes óseos como codos, rodillas, y la
piel cabelluda así como la región sacrocoxígea.

1.- Christophers E, Krueger G G. Psoriasis. En: Fitzpatrick TB, EISEN AZ, Wolff K. Dermatología en
Medicina General Buenos Aires: Editorial Panamericana 1988. P. 585-591

2.- Christophers E, Schubert C, Schröder J M. Psoriasis. Dermatología. 1992; 45.

15.- En un paciente con diagnóstico de Enfermedad de Hirschsprung, ¿Cuál de las siguientes


complicaciones es la más grave?

a) Vólvulus.
b) Invaginación Intestinal.
c) Sangrado de Tubo Digestivo.
d) Enterocolitis.

Los problemas con un niño que padece la enfermedad de Hirschsprung dependen de la porción de intestino
que tiene células nerviosas normales. El setenta por ciento de los bebés con la enfermedad de Hirschsprung
carecen de células nerviosas solamente en una porción de los últimos treinta o sesenta centímetros (uno o
dos pies) del intestino grueso. Siendo una de las complicaciones mas severas la enterocolitis.

REFERENCIAS BIBLIOGRAFICAS:

1. Operative Pediatric Surgery. Moritz M. Ziegler. International Edition, pág 445– 463.
2. Bibliografía: Urgencias en Pediatría, Interamericana.McGraw – Hill. Capítulo: Urgencias Médico
Quirúrgicas, Sección XXIII, Pág. 182-194.
3. Enciclopedia Médico Quirúrgica-Pediatría 4-014-L-10-2005.

16.-Ingresa al servicio de Traumatología masculino de 35 años postraumatizado por colisión automovilística


con cuadro de pérdida progresiva de fuerza en miembros inferiores, los reflejos osteotendinosos están
abolidos. Al realizarse radiografías simples muestran una fractura por compresión de L1 con
desplazamiento del muro posterior y acuñamiento anterior de un 50%. ¿Para valorar la ocupación del canal
raquídeo, cual de las siguientes pruebas indicaría?

a) Una gammagrafía ósea.


b) Una tomografía cervical.
c) Rx ap y lateral con foco en L5.
d) Una TAC vertebral centrada en región dorso lumbar.

Tomografía computarizada (TC)

En líneas generales, podemos decir que sus indicaciones (con carácter urgente) en los TRM, son todas
aquellas lesiones detectadas o sospechadas en las radiografías simples, que puedan suponer riesgo de lesión
medular por desplazamientos ulteriores (lesiones inestables), o que ya estén produciendo daño neurológico
susceptible de mejorar o estabilizarse tras descompresión quirúrgica. Por lo tanto debe realizarse en todos
los pacientes con fracturas, luxaciones y fracturas-luxaciones inestables, y en aquellos con déficit
neurológico, preferentemente incompleto. En pacientes con lesiones estables en las radiografías simples y
sin déficit neurológico, puede diferirse en función de la presión asistencial y de la disponibilidad del servicio
de Radiología. En la práctica, suele indicarse también cuando no se visualizan determinadas zonas de la
columna, generalmente C1-C2 y C6-Dl. En pacientes con TCE grave deben realizarse cortes de estas zonas,
si no se ven claramente en las radiografías o en el "scout" cervical, e incluso de forma rutinaria, dada la
frecuente asociación de estas lesiones. La TC proporciona una excelente visualización de las estructuras
raquídeas principalmente de los elementos posteriores y del canal medular, por lo que se pueden ver con
nitidez los desplazamientos y fragmentos óseos que puedan estrecharlo o invadirlo. Asimismo, pueden verse
fracturas o desplazamientos inadvertidos en las radiografías y nos permite valorar mejor estas lesiones,
definiendo claramente las líneas de fractura, cuantificando exactamente los desplazamientos, y en
definitiva, evaluando la estabilidad vertebral.

Habitualmente se realizan cortes de 5 mm de espesor, aunque la exploración detallada de la columna cervical


pueda requerir cortes más delgados, lo que alargará el tiempo de estudio, factor importante en el manejo de
los traumatismos. Los equipos de últimas generaciones han conseguido acortar este tiempo y permiten
además, la reconstrucción sagital o coronal a partir de los cortes axiales, mejorando la definición y la
evaluación del canal medular.
La TC es útil también para valorar las articulaciones interapofisarias y los agujeros de conjunción, así como
los hematomas paravertebrales y retroperitoneales. Una ventaja adicional es la de ofrecernos información
suplementaria sobre ciertas partes blandas del cuello y de las cavidades torácica y abdominal.

Las fracturas horizontales que no coincidan con el plano de la TC pueden no visualizarse, como la de
odontoides o algunas por compresión. Los hematomas epidurales, hematomielia y hernias discales pueden
verse también con TC, aunque la RM define mejor estas lesiones. La contusión y el edema medular, las
lesiones y avulsiones radiculares y los desgarros durales requieren estudio mielográfico adicional y/o RM. La
mielo-TC se realiza con inyección de contraste intratecal por punción lumbar o cervical, que obliga a la
movilización del paciente o retirada del collarín cervical, además de otros inconvenientes como tiempo de
estudio y reacciones adversas, por lo que generalmente no son útiles en el manejo urgente del paciente con
TRM.

REFERENCIAS BIBLIOGRAFICAS:

1. Ballinger, Phillip W. Cerril. Atlas de posiciones radiográficas y procedimientos radiológicos. 7. ª ed.;


España: Masson.
2. Goaz P. W. Radiología oral (principios e interpretación). 3.ª Ed. España; ed.; Mosby.1995.

17.-Masculino de 40 años que de manera inesperada y espontánea inicia cuadro constituido por disnea,
sensación de ahogo o de paro respiratorio, sensación de inestabilidad, “siente que se desmaya”, palpitaciones
o taquicardia, mareo, nauseas, dolor abdominal. ¿Cuál de los siguientes el diagnóstico mas probable?

a) Trastornos de ansiedad generalizada.


b) Trastorno fóbico.
c) Trastorno de pánico.
d) Trastorno mixto ansioso-deperesivo.
La característica fundamental del Trastorno de Pánico es la presencia de Crisis de Pánico recurrentes,
inesperadas y que no se encuentran relacionadas con ninguna circunstancia en particular (es decir son
espontáneas), sin un factor externo que las desencadene, es decir que no son desencadenadas por una
exposición a una situación social (como es el caso de la Fobia Social) o frente a un objeto temido (en cuyo
caso se trataría de una Fobia Específica ). Otra característica del Trastorno de Pánico es el miedo
persistente a padecer una nueva Crisis de Pánico, esto es miedo al miedo y se lo denomina Ansiedad
Anticipatoria. Esta ansiedad puede a llegar a ser tan importante que puede llevar a confundir el diagnóstico
de Trastorno de Pánico con el de Trastorno de Ansiedad Generalizada (TAG).

Estas Crisis (o Ataques) de Pánico, inesperadas y recidivantes, suelen presentar cuatro o más de los
siguientes síntomas:

• Miedo intenso a morir o a estar sufriendo un ataque cardíaco o alguna enfermedad física grave que
ponga en riesgo la vida.
• Miedo intenso a volverse loco o a perder el control de si mismo.
• Palpitaciones (percepción del latido cardíaco) o pulsaciones aceleradas (taquicardia).
• Sudoración.
• Palidez.
• Temblores o sacudidas musculares.
• Sensación de ahogo o falta de aire.
• Opresión en la garganta (sensación de no poder respirar) o en el pecho.
• Náuseas, vómitos o molestias y dolores abdominales.
• Inestabilidad, mareos o desmayos.
• Sensación de irrealidad (sentir al mundo externo como algo extraño).
• Sensación de no ser uno mismo (despersonalización).
• Hormigueos (parestesias).
• Escalofríos o sensación de sufrir frío intenso.

Las crisis se inician bruscamente, alcanzan su máxima intensidad en los primeros diez a 15 minutos y suelen
durar menos de una hora. Dejan a quien las sufre en un estado de total agotamiento psicofísico y con un gran
temor (ansiedad anticipatoria) a volver a padecer una nueva crisis (miedo al miedo). También pueden
aparecer síntomas de tipo depresivo, miedo a salir o alejarse del hogar o necesidad de hacerlo acompañado
por una ser muy cercano (familiar). La persona suele sentirse muy hipersensible y vulnerable. Este tipo de
trastorno es tan traumático de experimentar que quien lo padece suele cambiar en forma brusca y
desfavorable sus hábitos de vida: no querer salir solo de la casa o viajar, retraimiento social, abandono de
sus actividades laborales o académicas.

Muchas veces la persona que se encuentra padeciendo una Crisis de Pánico tiene la necesidad de "salir
corriendo" del lugar donde se encuentra o de consultar urgentemente a un médico en el caso que crea que se
esta muriendo de verdad. Si esta crisis se experimenta por primera vez en un lugar determinado, un tren o
autobús por ejemplo, suele quedar temor de volver a ese mismo sitio o medio de locomoción, desarrollándose
de este modo una fobia al mismo.

Un paciente que sufría de Crisis de Pánico describió su primera crisis de la siguiente forma:
"Súbitamente sentí una oleada de miedo sin que hubiera razón alguna. El corazón me latía apresuradamente,
me faltaba el aire, sentía que no podía respirar, el corazón latía tan fuerte que parecía que iba a salirse por
mi boca. Me dolía el pecho, tenía mareos, ganas de vomitar, no podía parar de temblar. Sentía que me moría.
Era como una agonía mortal que nunca terminaba"

Este trastorno se lo clasifica dentro de las neurosis, no tiene nada que ver con la locura (psicosis), ni
desencadena con el tiempo en un cuadro de locura. Es importante recalcar esto ya que muchos pacientes que
sufren este trastorno, o los familiares del mismo, piensan que se están volviendo locos. Lo que sí es
frecuente es que las personas que padecen de Trastorno de Pánico desarrollen, si no son tratados a tiempo
y adecuadamente, cuadros de tipo depresivos, fobias múltiples (especialmente agorafobia) o abuso de
sustancias (alcohol o drogas).

Referencias Bibliográficas:

- Diagnostic and Statistical Manual of Mental Disorders, 4° edition (DSM-IV TR). American Psychiatric
Association. American Psychiatric Press, 2000.
- Kaplan and Sadock's Synopsis of Psychiatry, 9° edition. Lippincott Williams & Wilkins Press, 2003.

18.- En la inserción anormal placentaria, cuando se infiltra hasta la serosa uterina se le denomina:

a) Increta
b) Acreta
c) Percreta dextruens
d) Percreta

En un embarazo normal, la placenta se adhiere a la pared uterina lejos del cuello del útero. Placenta accreta
se refiere a una placenta que se fija a demasiada profundidad y con demasiada firmeza en la pared del
útero. Placenta increta es una placenta que se fija con una profundidad aún mayor en la pared uterina.
Placenta percreta es una placenta que se fija a través del útero y a veces se extiende hasta los órganos
circundantes, como la vejiga.

Estos trastornos se producen en aproximadamente 1 de cada 2.500 embarazos. Suelen causar hemorragia
vaginal en el tercer trimestre y con frecuencia resultan en un parto prematuro. Dado que la placenta no
puede separarse fácilmente de la pared del útero después del parto, por lo general se extirpa
quirúrgicamente. Suele ser necesario realizar una histerectomía (la extirpación del útero), aunque pueden
utilizarse otros procedimientos quirúrgicos para salvar el útero.

Hay poco que una mujer con una de estas condiciones pueda hacer para tratarla o prevenirla. Si se le
diagnostica placenta accreta antes del parto (mediante un ultrasonido), puede planificarse el parto y
considerarse un procedimiento quirúrgico para salvar el útero. Esto es algo que debe discutir con su médico,
especialmente si desea tener otro hijo en el futuro. Pero si la condición es grave, puede que no sea posible.
REFERENCIA BIBLIOGRAFICA:

1. Perucca E, Domínguez C, Yahng Ch, García R. Placenta previa percreta con invasión vesical. Rev Chil
Obstet Ginecol 1997; 62(3): 206-10.

2. Abbas F, Talati J, Wasti S et al. Placenta percreta with bladder invasion as a cause of life
threatening hemorrhage. J Urol 2000; 164: 1270-4.

19.- La maniobra que nos indica en un recién nacido, que la articulación de la cadera esta luxada, se conoce
como:

a) Signo de Galeazzi.
b) Signo de Barlow.
c) Signo de Ortolani.
d) Signo de Pistón.

Prueba de Ortolani : Con esta prueba se detecta una cadera ya luxada, se coge con la mano el miembro
flexionado, la cadera se coloca en abducción mientra se levanta el fémur con cuidado y se sitúa los dedos a
nivel del trocánter mayor. Si la prueba es positiva se siente la reducción de la cadera dentro del acetábulo.

Prueba de Barlow: Es una prueba inductora para identificar una cadera inestable pero aún localizada en su
sitio; no es una prueba adecuada para diagnosticar luxación de cadera. La cadera en aducción ligera y con la
palma de la mano se empuja suave y cuidadosamente hacia atrás, la presencia de un movimiento de “pistón”
o al percepción de una cabeza femoral subluxada sobre el borde posterior del acetábulo.
Prueba de Galeazzi: con el niño acostado se le flexionan las caderas y rodillas de modo que los talones se
apoyen sobre la mesa y reconocer el acortamiento relativo del muslo.
1. Skinner, H. Diagnóstico y tratamiento en Ortopedia. Ed. Manual Moderno. México, 2004. pp. 625

20.-.- El signo radiográfico de Thurston Holland es característico de las lesiones fisiarias tipo (Salter y
Harris):

a) I
b) II
c) III
d) IV

Siguiendo la clasificación de Salter y Harris, podemos distinguir cinco tipos de epifisiólisis.

1) Tipo I. El trazo lesional cursa en su totalidad por la fisis (zona hipertrófica profunda).
2) Tipo II. El trazo cursa por la fisis pero asciende hacia la metáfisis desprendiendo un fragmento
metafisario triangular.
3) Tipo III. El trazo provoca la discontinuidad de la epífisis y prosigue a través de la fisis.
4) Tipo IV. El trazo provoca discontinuidad de la epífisis y asciende hacia la metáfisis desprendiendo un
fragmento metafisario triangular como en las tipo II. Es típica del cóndilo humeral lateral infantil.
5) Tipo V. Compresión axial con destrucción de las zonas de reserva y proliferativa.

Manual CTO 5ª edición, Traumatología y Ortopedia, Pág. 38.


21.-En un paciente diagnosticado con esclerosis múltiple (considerada como un proceso inflamatorio y
desmielinizante del SNC). En el diagnóstico de esta enfermedad es muy útil la presencia de:

a) Más de 100 liníocitos por microlitro en el líquido cefalorraquídeo.


b) Más de 50 polimorfonucleares por microlitro en el líquido cefalorraquídeo.
c) Elevaciones del ácido úrico en plasma.
d) Bandas oligoclonales en el líquido cefalorraquídeo.

La esclerosis múltiple (EM) es una enfermedad desmielinizante del sistema tema nervioso central, que
habitualmente se caracteriza por ataques recurrentes de disfunción neurológica focal y multifocal.

Los síntomas y signos clásicos de EM dependen de la localización del foco de desmielinización y pueden
incluir una variedad de disfunciones que no son específicas para la EM, tales como alteración de la visión,
ataxia y temblor intencional, debilidad o parálisis de una o más extremidades, espasticidad y problemas
vesicales. Hay sin embargo tres anormalidades, altamente sugestivas de EM:

• Neuritis óptica, la cual es el síntoma inicial en cerca del 25% de los pacientes.
• Oftalmoplejia internuclear que se asocia con nistagmus monoocular.
• Signo de Lhermitte, que es una sensación "eléctrica" por detrás del cuello y de la espalda que llega
hasta las piernas (al flexionar el cuello).

En fibras nerviosas normales mielinizadas, la conducción ocurre de manera saltatoria; las corrientes de
acción se confinan a las secciones no mielinizadas del axón saltando de un nódulo de Ranvier al siguiente.
Este tipo de conducción es mucho más eficiente desde el punto de vista energético que la transmisión a
través de la superficie del axón entero, aumentando así la velocidad de conducción con una pérdida mínima
de energía.

La desmielinización de una porción de la fibra nerviosa normalmente mielinizada, puede conducir a: bloqueo
de la conducción en el sitio de la pérdida de mielina, disminución de la velocidad de conducción nerviosa a
través de la fibra alterada y fatiga subjetiva aumentada o un mayor consumo de energía.

La disfunción neurológica observada en la EM es una reflexión de la alteración de la conducción a través de


segmentos parcial o completamente desmielinizados o de una fibra nerviosa mielínica. Además, el hecho de
que el tiempo de conducción a través de los segmentos desmielinizados disminuye con aumento de la
temperatura puede explicar por qué los síntomas clínicos de EM se empeoran al aumentar la temperatura
corporal.

Diagnóstico

El diagnóstico de EM se basa fundamentalmente en dos parámetros. Por un lado los hallazgos en los estudios
de imagen por Resonancia Magnética en los que se presentan (en las imágenes dependientes de T2) áreas
hiperintensas de localización fundamentalmente periventricular en la substancia blanca subcortical en
múltiples localizaciones así como también en la médula espinal; este estudio permite visualizar no solamente
la localización de las áreas desmielinizantes sino también en forma seriada pueden realizarse evaluaciones
para valorar la evolución clínica y subclínica de la enfermedad. Los otros estudios básicos para el diagnóstico
son la determinación de bandas oligoclonales, la determinación de la proteína básica de la mielina y la
medición de las inmunoglobulinas en el líquido cefalorraquídeo. Estos dos factores, en conjunto con la
información clínica, permiten establecer con ciertas bases de seguridad el diagnóstico de esta enfermedad.

Referencias bibliográficas:

1. Noseworthy JH, Lucchinetti C, Rodríguez M, Weinshenker BJ. Multiple sclerosis. N. England J Med.
2000;343:938-52.
2. Omari KM, John GR, Sealfon SC, Raine CS. CXC chemokine receptors on human oligodendrocytes:
implications for multiple sclerosis. Brain 2005;128:1003-15.
3. Mi S, Miller RH, Lee X, et al. LINGO-1 negatively regulates myelination by oligodendrocytes. Nat
Neurosci. 2005;8:745-51.
4. John GR, Shankar SL, Shafit-Zagardo B, et al . Multiple sclerosis: re-expression of a developmental
pathway that restricts remyelination. Nat Med. 2002;8:1115-21.
5. Lucchinetti C. The spectrum of idiopathic inflammatory demyelinanting disease. In: American
Academy of Neurology. Syllabi on CD ROM. 2000.
6. Hartung HP, Grossman RI. ADEM: distinct disease or part of the MS spectrum? Neurology 2001;
56:1257-60.
7. Capello E, Voskuhl RR, and McFarland HF, Raine CS. Multiple sclerosis: re-expression of a
developmental gene in chronic lesions correlates with remyelination. Ann Neurol. 1997; 41:797-805.
8. Atlanta. Georgia. AAN. 115 Anual Meeting. Natural Course of Multiple Sclerosis Redefined: National
Inst of Neurological Disorder and Stroke, 16, 1990.
9. Hemmer B, Archelos JJ, Hartung HP. New concepts in the inmunopathogenesis of MS. Nat Rev
Neurosci. 2002;3:291-301.
10. Kurtzke JF. Geography in multiple sclerosis. J Neurol. 1977;215:1-26.

22.- Masculino recién diagnosticado por infección de VIH ¿Cual de las siguientes opciones es la correcta
para el tratamiento inicial?

a) Inmunoestimulantes mas un antiviral.


b) Dos inhibidores de proteasas, más un inhibidor de transcriptasa reversa análogo de nucleósidos.
c) Dos inhibidores de transcriptasa reversa no análogos de nucléosidos, y un inhibidor de proteasa.
d) Dos inhibidores de transcriptasa reversa análogos de nucleósidos y un inhibidor de proteasa.

El tratamiento inicial para el paciente con VIH debe seguir dos caminos: o utilizar inhibidores de proteasa o
utilizar inhibidores de transcriptasa reverse no análogos de nucleósidos. De estos grupos se elige un
compuesto, el cual se debe acompañar con una base de dos inhibidores de transcriptasa reversa análogos de
nucléosidos.
REFERENCIA BIBLIOGRAFICA:
1.-Kasper DL, Braunwald E, Fauci AS, Hauser SL, Longo DL, Jameson JL. Harrison´s Principles of Internal
Medicine. McGraw Hill. 16 Ed. 1076.

23.- Se trata de masculino de 24 años que acude por esterilidad; es azoospérmico, tiene fenotipo
eunucoide, ginecomastia, distribución feminoide del vello púbico, testículos pequeños y cromatina sexual
positiva en 8%. ¿El diagnóstico a considerar es?

a) Síndrome de Noonan.
b) Síndrome de Turner.
c) Síndrome de Klinefelter.
d) Varón XX.

SINDROME DE KLINEFELTER (SK)


Es la cromosomopatía más frecuente y la causa más habitual de hipogonadismo hipergonadotrópico en el
varón. Descrito en 1942 como un síndrome caracterizado por hipogonadismo, testes pequeños y duros,
azoospermia y ginecomastia. Se comprobó posteriormente que el cuadro corresponde a una patología
genética, cuya alteración cromosómica más habitual es la presencia de un cromosoma X adicional, reflejando
un cariotipo 47 XXY, que representa el 80% de los casos de SK, pero se han descrito otras variantes como
mosaicismos: 47XXY/46XY, 47XXY/46XX, 47XXY/46XY/45X, etc. y formas con más de un cromosoma X ó
Y (48XXYY ó 47XXY/46XX/poliX). Esto hace que estos sujetos presenten una cromatina de Barr con masa
presente, siendo esto propio de las mujeres por la presencia de 2 cromosomas X. La aparición de más de 2
cromosomas X, ocasiona una patología que se diferencia del cuadro clásico de SK y se denomiona polisomía X
del varón: 48XXY, 49XXXXY.
La fórmula XXY se debe a una no disyunción del cromosoma X en la primera ó segunda división meiótica,
siendo más frecuente la aparición de SK en relación a la edad materna más avanzada.
Su frecuencia se estima en 1/1000 recién nacidos varones, pero posiblemente sea más elevada: en torno a
1/500 concepciones masculinas, y en varones con retraso mental moderado la frecuencia aumenta a 8/1000
aproximadamente.
CLINICA: en general el diagnóstico es tardío, debido a la pobre expresividad clínica de este síndrome en la
infancia.

1) Talla alta: Es propia del síndrome de Klinefelter en la edad de la adolescencia, pero no lo es en la época
prepuberal de forma habitual. Las proporciones corporales son eunucoides, con aumento
desproporcionado de las extremidades inferiores y con inversión del cociente: segmento
superior/inferior. Cuando este cociente es inferior a 1 en la edad puberal, sugiere fuertemente la
existencia de un SK si se asocia a un cuadro de hipogonadismo aunque sea leve.

La velocidad de crecimiento suele aumentar a partir de los 4 – 5 años, y se adopta ya en la infancia un


cierto hábito eunucoide. La edad ósea es normal ó poco retrasada, coincidiendo con el marcado retraso de
otras variantes benignas de retraso puberal.
2) Alteraciones genitales: Los testes pequeños y duros en la edad prepuberal, pueden presentarse en
ocasiones, siendo por el contrario un hallazgo típico pasada la adolescencia.

La presencia de criptorquídea, micropene, hipospadias, etc y a veces un fenotipo peculiar, puede orientar al
diagnóstico.
En la edad puberal, la atrofia testicular es un signo constante, los testes son pequeños y su tamaño va
involucionando con el tiempo, contrastando con un desarrollo normal del pene y del relativamente normal
desarrollo puberal. La histología del testículo demuestra atrofia testicular con hialinización de los túbulos
seminíferos.

3) Ginecomastia: Está presente en un elevado nº de casos (más del 60%), siendo el riesgo de neoplasia
mamaria 20 veces superior al de la población normal.

4) Desarrollo intelectual: La inteligencia en general suele ser normal, ó “bordeline”, especialmente es el área
verbal la más precoz y afectada, lo que condiciona dificultades de aprendizaje y en general peor
rendimiento escolar.

5) Alteraciones hormonales: En la edad prepuberal, tanto el nivel de gonadotrofinas como de testosterona


se encuentran en rango normal, tanto basal como tras estímulo. En la pubertad se va estableciendo ya
respuesta propia del hipogonadismo hipergonadotrópico, especialmente el nivel de FSH que se eleva de
forma llamativa.

6) Otras alteraciones asociadas: Anomalías del tracto urinario, cúbito valgo etc. (se recogen en la tabla
adjunta). Especial relevancia por la ayuda que puede suponer para el pediatra, es la relativamente
aumentada incidencia de incontinentia pigmenti.

Son frecuentes las asociaciones del SK con neoplasias especialmente las de células germinales mediastínicas,
y con otros tumores en la edad adulta.

REFERENCIA BIBLIOGRAFICA:

– Rossodivita A, Colabucci F. Short stature in a patient with Klinefelter syndrome and growth
hormone deficiency.Am J of Med Genet 1994; 49:244-6.

24.-Un paciente con insuficiencia renal crónica debida a hipotensión prolongada grave es atendido porque
presenta dolor retroesternal. Se le indicó hemodiálisis dos veces por semana en los últimos dos años y en
fechas recientes ha experimentado episodios de hipotensión al inicio del tratamiento. El dolor se localiza
sobre el músculo trapecio. Se reduce un poco al adoptar la posición de pie y se exacerba con la respiración
profunda. ¿Cuál de los siguientes trastornos es la causa más probable del dolor retroesternal que sufre este
paciente?

a) Arteriopatía coronaria.
b) Espasmo esofágico difuso.
c) Pericarditis.
d) Embolias pulmonares.
El dolor torácico que sufrió este paciente es característico de pericarditis e inflamación del pericardio,
complicaciones comunes en personas con insuficiencia renal crónica en hemodiálisis. Estos enfermos también
pueden tener inflamación de varios recubrimientos serosos, como peritoneo y pleura; no se conoce el
mecanismo de esta complicación. Aunque es común la arteriopatía coronaria en pacientes en diálisis, las
características de dolor en este individuo sugieren que no es el diagnóstico. La enfermedad esofágica
también es común en sujetos en diálisis y debe descartarse específicamente como posible causa. La relación
aparente con la diálisis, así como la frecuencia de los síntomas, van contra el diagnóstico de embolia
pulmonar como causa del dolor torácico. Además, el dolor musculoesquelético debido a diversos trastornos
se observa en pacientes en diálisis y puede deberse a anomalías en el metabolismo del calcio y el fósforo,
que produce depósitos de calcio en diversos componentes del sistema musculoesquelético.

REFERENCIA BIBLIOGRAFICA:

– Allen R. M. MMS Medicina Interna. 5ª. Edición. National Medical Series. Mc. Graw Hill. 2006.
(capítulo 6, parte I: III E 2).
– Alexander JS. A pericardial effusion of gold paint appearance due to the presence of cholesterin.
Br Med J 1919; 2: 463
– Brawley RK, Vasko JS, Morrrow AG. Cholesterol pericarditis: consideration of its pathogenesis and
treatment. Am J Med 1966; 41: 235-248.

25.- Femenino de 30 años con sospecha diagnóstica de endometriosis, el síntoma característico de esta
patología es:

a) Dispareunia.
b) Metrorragia.
c) Leucorrea.
d) Dismenorrea.

ENDOMETRIOSIS:

Es relativamente frecuente en personas jóvenes, 25-30 años. Se define como la localización de la mucosa
endometrial en un lugar ectópico.

La localización normal de endometrio es el útero. La mucosa endometrial sufre una serie de cambios durante
el ciclo. Cuando la mucosa endometrial penetra en el músculo uterino (miometrio) se denomina: Adenomiosis.

Patogenia:

No se conoce realmente por qué se produce y tampoco existe un tratamiento definitivo para ella.

Teorías: a) Teoría Metaplásica celómica (T. De Meyer): El celoma es un epitelio que recubre la cavidad
abdominal (metaplasia).
b) Teoría Transplantativa (Sampson): La mucosa endometrial a través de las trompas, cuando se produce una
menstruación si el orifico cervical está cerrado, la sangre refluye (por mecanismo retrógrado), y va a
cavidad abdominal donde se producen implantes de células endometriales (en fondos de saco de Douglas,
peritoneo...). También por intervenciones quirúrgicas. Esta teoría es la que parece más veraz.

c) Teoría embrionaria: Es poco importante. Dice que la endometriosis se produce por los restos embrionarios
de Wolf o Müller.

d) Teoría inmunológica: Suele influir sobretodo en las personas estériles, con disminución de la respuesta
inmunitaria por: - Disminución de la citotoxicidad de las células NK// - Aumento de resistencia de las células
endometriales a la destrucción// - Aumento de la actividad de los Macrófagos.

Clínica:

• Disfunción menstrual: Aparición de dismenorrea progresiva, que no aparece desde el principio (es
tardía), es intermenstrual y aumenta al final más intensa. Va aumentando con los años (al contrario
que la Dismenorrea normal).

• Exploración: Retracción de los ligamentos uterosacros, nodulaciones, rugosidades y dolorosos al


tacto. Dispareunia (dolor en las relaciones sexuales). Dolor en la defecación (porque la reacción
inflamatoria produce adherencias y dolor en la movilización del útero). Dismenorrea.

• Disfunción ovárica: Se hace quistectomía, son más conservadores en el tratamiento quirúrgico.


Insuficiencia lútea. Hiperprolactinemia. Amenorrea// No son muy frecuentes ni específicas puede
que no se den las tres.

• Disfunción reproductiva: Es causa de esterilidad (estadios III y IV de la clasificación americana:


Reacción inflamatoria que afecta a anejos, recto, sigma, apéndice, ureter...)// Se producen abortos
de repetición e insuficiencia lútea.

Bibliografía

1. Stenchever A. Comprehensive Gynecology. 4th ed. St. Louis, Mo: Mosby; 2001:1065-1070.
2. Noble J. Textbook of Primary Care Medicine. 3rd ed. St. Louis, Mo: Mosby; 2001:325.
3. Chen C, Cho S, Damoskosh AI. Prospective study of exposure to environmental tobacco smoke and
dysmenorrhea. Environ Health Perspect 2000; 108(11): 1019-1022.
4. Wilson ML, Farquhar CM, Sinclair OJ. Surgical interruption of pelvic nerve Pathways for primary
and secondary dysmenorrhea. Cochrane Database Syst Rev 2000; (2): CD001896.
5. Morrison B, Daniel S, Kotey P, et al. Rofecoxib, a specific cyclooxygenase-2 inhibitor, in primary
dysmenorrhea: A Randomized controlled trial. Obstet Gynecol 1999; 94(4): 504-508.
6. Schroeder B, Sanfilippo J. Dysmenorrhea and pelvic pain in adolescents. Pediatrics Clinics of North
America. 1999; 46 (3): 555-571.
7. Dawood MY. Dismenorrea. Clínicas Obstétricas y Ginecológicas. 1990; 1: 167-176.

26.- ¿Para realizar el diagnóstico preciso de endometriosis se requiere?

a) Visualización directa de las lesiones patognomónicas.


b) Corroboración histológica de glándulas y estroma en tejido obtenido por biopsia de las lesiones
sospechosas observándolas directamente por laparoscopia o laparotomía.
c) Detección ecográfica de un endometrioma.
d) Presencia de masa dolorosa en paciente fértil.

La endometriosis es una condición que se caracteriza por la presencia de síntomas variados. Lesiones
mínimas pueden estar asociadas con dolor severo, y por el contrario lesiones extensas pueden permanecer
asintomáticas. Las formas infiltrantes de la endometriosis han sido descritas desde 1913. En esta forma
particular de la enfermedad la endometriosis penetra por debajo de la superficie del peritoneo y puede no
ser visible. Las lesiones de endometriosis que invaden más allá de cinco mm se definen como profundamente
invasivas. 10 Debido a su localización subperitoneal, el diagnóstico es con frecuencia difícil, incluso por vía
laparoscópica. 11 Estas formas infiltrantes están estrechamente relacionadas con dolor pélvico y dispareunia
profunda. Asimismo, las lesiones están generalmente localizadas en el fondo del saco de Douglas, ligamentos
uterosacros y pliegue útero vesical. Junto con los endometriomas ováricos, estas lesiones son consideradas
la manifestación más severa de la enfermedad.

REFERENCIA BIBLIOGRAFICA:

1. Mangal R, Taskin O, Nezhat C, Franklin Robert. J Reprod Med 1995; 41: 64-6. Goh JT, Flynn V.
Inguinal endometriosis. Auzt NZ J Obstet Gynecol 1994; 34: 121.

27.-Femenino de 45 años de edad que se encuentra en terapia intensiva por manejo de sépsis abdominal.
Lleva dos días con tratamiento a base de ceftriaxona, metronidazol y amikacina, el cual provee de una
cobertura conveniente para los microorganismos probablemente involucrados. Después de revisar al paciente
que ha tenido una evolución favorable, usted sugiere que se aplique el aminoglucósido en monodosis ya que:

a) De esta forma aumenta su potencia.


b) Como tiene un efecto post antibiótico corto su efecto es mejor.
c) Disminuyen sus efectos adversos.
c) Hace sinergismo con el Metronidazol.

Una característica farmacológica de los aminoglucósidos es su actividad dependiente de concentración, por


lo que su aplicación en bolos, con lo que se logran niveles pico más altos, es de mayor conveniencia para
aprovechar este fenómeno. Además, el importante efecto post-antibiótico que tiene, permiten que la
actividad dure más allá del tiempo en que sus concentraciones se encuentran por arriba de la mínima
inhibitoria, favoreciendo una posología cómoda cada 24 horas. Por otra parte, se ha visto que los efectos
adversos (nefrotoxicidad y ototoxicidad), se encuentran más relacionados con los niveles valle. Estos se
mantienen a niveles bajos cuando se utiliza el esquema en monodosis, disminuyendo la probabilidad de la
presentación de estos efectos adversos.

REFERENCIA BIBLIOGRAFICA:

– Kasper DL, Braunwald E, Fauci AS, Hauser SL, Longo DL, Jameson JL. Harrison´s Principles of
Internal Medicine. McGraw Hill. 16 Ed. 789-806 pp.

28.- Al realizar en un paciente de 7 años de edad el diagnóstico de escarlatina, usted elige el siguiente
fármaco por ser el medicamento de elección en esta patología.

a) Penicilina benzatínica.
a) Cefuroxime.
b) Amoxicilina.
c) Eritromicina.

Streptococcus pyogenes (hemolítico del grupo A) es el agente etiológico de la escarlatina, no ofrece


resistencia a la penicilina benzatínica y con dosis única ofrece un buen índice de curación, erradicación y
reducción de complicaciones.

REFERENCIA BIBLIOGRAFICA:

– González-Saldaña N, Infectología Clínica Pediátrica, 7ª edición, páginas 403-407.

29.- Un sujeto de 30 años de edad adicto a las drogas intravenosas presenta debilidad del hemicuerpo
derecho y cefalea en un periodo de dos días. La exploración revela un individuo mal nutrido y afebril con
hemiparesia derecha leve. ¿Cuál de los siguientes es el diagnóstico más probable?

a) Endocarditis bacteriana.
b) Meningitis por virus de la inmunodeficiencia humana (VIH) .
c) Absceso cerebral.
d) Meningitis criptocócica.

Los adictos a drogas intravenosas están propensos a sufrir bacteriemia, que a su vez puede producir
absceso cerebral y disfunción neurológica progresiva. Los pacientes con aquél, por lo común se encuentran
afebriles a no ser que haya endocarditis acompañante u otro origen endovascular de infección. Los adictos a
drogas intravenosas tienden a presentar endocarditis bacteriana y pueden cursar con déficit neurológicos
en forma apoplética debido a embolia séptica del cerebro. Sin embargo, por lo regular tienen fiebre. La
meningitis por VIH produce cefalea y datos de irritación meníngea, pero no se presenta déficit neurológico
focal. La meningitis criptocócica se manifiesta con conducta alterada y cefalea y los pacientes están
afebriles. Sin embargo, es raro que haya datos de seudoapoplejía. Por último, el uso de drogas intravenosas
puede provocar embolia de cuerpo extraño pero con problemas neurológicos apopléticos. Un émbolo puede
llegar al cerebro por un cortocircuito cardíaco de derecha a izquierda o bien por una malformación
arteriovenosa pulmonar si la inyección es venosa. El émbolo puede entrar a la circulación cerebral de manera
directa en caso de inyección intracarotídea.

Referencias bibliográficas:

– Nath A. Brain abscess and parameningeal infections. In: Goldman L, Ausiello D, eds. Cecil Medicine.
23rd ed. Philadelphia, Pa: Saunders Elsevier; 2007: chap 438.
– Allen R. M. MMS Medicina Interna. 5ª. Edición. National Medical Series. Mc. Graw Hill. 2006.
(Capítulo 11 XVI B 1-2).

30.- Se solicita la realización de antígeno prostático específico a un paciente de 64 años como estudio
adicional por manejo de hipertensión arterial. La prueba revela un aumento de antígeno prostático específico
de 10.4 ng/ml. El tacto rectal revela una próstata de tamaño normal, pero en la ecografía se encuentra un
área hipoecoica pequeña que mide 5x7 mm en el lóbulo derecho. ¿Cuál de las siguientes medidas es el
siguiente paso apropiado?

a) Realizar gammagrama óseo.


b) Repetir el análisis de antígeno prostático específico en tres meses para verificar si hay aumento
ulterior.
c) Practicar biopsia transrectal del área anormal encontrada en la ecografía.
d) Comenzar el tratamiento con leuprolida de depósito.

El antígeno prostático específico es una prueba que puede usarse para detección de cáncer de próstata. Sin
embargo, los valores de éste también pueden estar un poco altos en la hipertrofia prostática benigna. Una
ecografía transrectal puede identificar lesiones pequeñas no palpables en la exploración rectal. Si los
pacientes tienen PSA alto y se confirmó en la ecografía un área anormal, se puede practicar biopsia por vía
transrectal bajo guía ecográfica. En general, a los varones con cáncer de próstata se les estudia en busca de
metástasis con gammagrama óseo; radiografía de tórax; tomografía computarizada de retroperitoneo y
pelvis; o resonancia magnética de retroperitoneo y pelvis con énfasis en la próstata, acompañados por
estudios de laboratorio. El carcinoma prostático metastásico puede tratarse con leuprolide, un agonista de
la hormona liberadora de hormona luteinizante que suprime la producción testicular de testosterona. El
tratamiento con leuprolide equivale a orquiectomía o a terapéutica con estrógenos en cáncer prostático o
metastásico.

Referencia Bibliográfica:

– Allen R. M. MMS Medicina Interna. 5ª. Edición. National Medical Series. Mc. Graw Hill. 2006.
(capítulo 4 X C 4 a (3), b (1)).

31.- Forma más frecuente de la infección por papiloma virus en varones:

a) Verrugas plantares.
b) Verrugas laríngeas.
c) Subclínica en genitales.
d) Condilomas prepuciales.

En diferentes estudios de varones compañeros de mujeres con lesiones cervicales por virus del papiloma
humano se demostró que el 88% tenían signos histológicos de condilomas, de los cuales el 72% eran en
forma subclínica es decir no había ningún tipo de lesión visible o síntoma. Otros estudios han
demostrado que hasta el 66% o mas de los compañeros de mujeres con neoplasias intraepiteliales
cervicales tienen infecciones subclínicas por virus del papiloma humano en el pene.

Referencia Bibliográfica:

– Obstet Gynecol 2001;185:308-17.


Kurtycz DFI, Hoerl HD. Thin-Layer Technology: Tempered Enthusiasm. Diag Cytopath 2000;23(1):1-
5.

32.-La dosis de prazicuantel como tratamiento en el diagnóstico de teniasis es el siguiente:

a) 20 mg/kg/día, administrado en 3 dosis.


b) 10 mg/kg/día, administrado en dosis única.
b) 15 mg/kg/día, administrado en dos dosis.
c) 30 mg/kg/día, administrado en 3 dosis.
Praziquantel

Grupo: agente antihelmíntico


Tabletas de 150 mg y 600 mg
Información general
El praziquantel, que no tiene ninguna relación estructural con los demás antihelmínticos, mata por igual
gusanos adultos y larvas. Los cestodos adultos (tenias) se contraen rápidamente y se desintegran en el
intestino. La mayor parte de las larvas mueren incluso cuando están enquistadas y se desintegran por
completo en el plazo de cinco meses.
El praziquantel se absorbe bien cuando se administra por vía oral; después de sufrir una primera etapa de
descomposición metabólica, el 80% de la dosis se elimina principalmente en forma de metabolitos por la
orina en un plazo de 24 horas.

Información clínica
Aplicaciones
Tratamiento de las infecciones por Taenia saginata, T. solium, Hymenolepis nana y Diphyllobothrium latum.

Dosificación y administración
Todas las dosis son aplicables por igual a adultos y a niños mayores de cuatro años.

Teniasis intestinal
Una dosis única de 5-10 mg/kg.

Himenolepiasis
Una dosis única de 15-25 mg/kg.

Difilobotriosis
Una dosis única de 10-25 mg/kg.

Ciclo Biológico y Características

La Taenia solium es un céstodo zoonótico cuyo hospedero definitivo es el hombre, albergando la forma
adulta en el intestino, y cuyo hospedero intermediario normal es el cerdo, albergando la forma larvaria o
cisticercosis (Figura 1). No hay otro hospedero definitivo natural; solamente se han podido cultivar gusanos
inmaduros en roedores como hamsters y chinchillas, previa inmunosupresión. Por el contrario, la forma
larvaria o cisticercosis puede afectar además del cerdo, al hombre y a otros mamíferos como perros y
osos(1). La cisticercosis humana es una causa frecuente de epilepsia y otros síntomas neurológicos en el Perú
y la mayoría de países en desarrollo, como se detalla más adelante
La teniasis intestinal se desarrolla cuando el FIGURA 1
humano ingiere carne de cerdo infectada con
cisticercos e insuficientemente cocida. Las larvas o
cisticercos, previa exposición a los jugos
intestinales y secreciones biliares, evaginan (sacan
la cabeza o escólex), y se fijan a la mucosa
intestinal por medio de cuatro ventosas y una doble
corona de ganchos presente en el rostelo o porción
distal del escólex. Los segmentos o proglótidos se
desarrollan a partir del cuello, la porción que sigue
al extremo cefálico, y a medida que van alejándose
del escólex maduran y se diferencian sexualmente.
Se consideran usualmente tres categorías:
Proglótido inmaduro, en el que el aparato genital
(masculino y femenino, con testículos, ovarios y
útero) empieza a formarse; Proglótido maduro, con
el aparato genital claramente diferenciado; y
Proglótido grávido, en el que la mayor parte del Ciclo biológico de Taenia solium (García HH,
proglótido se encuentra ocupada por el útero Martínez SM. Taenia solium
grávido, lleno de huevecillos. Los proglótidos de T. Teniasis/Cysticercosis, Segunda Edición. Lima:
solium son blanquecinos y pequeños, de Ed. Universo, 1999.
aproximadamente un centímetro por lado, y (Tornado con autorización).
usualmente son excretados con las deposiciones.
No presentan la motilidad de los proglótidos de T. saginata, que pueden salir por propio movimiento a través
del orificio anal y ser encontrados en la ropa interior del paciente (2).

Los huevecillos de T. solium son redondos, de aproximadamente 50 micras de diámetro, y consisten de un


embrión hexacanto (con seis ganchos) rodeado de una gruesa envoltura de colágeno que presenta apariencia
radiada al microscopio. Morfológicamente son indistinguibles de los huevos de Taenia saginata.
Recientemente se han publicado varias técnicas de biología molecular que permiten un diagnóstico de
especie utilizando partes de proglótidos, huevos, o material contaminado. La sensibilidad del examen
parasitológico de heces en teniasis por T. solium es baja, por lo que la introducción de técnicas inmunológicas
para la detección de antígeno parasitario en heces (detección de coproantígeno por ELISA) prácticamente
duplica la detección de casos (3).

El estadio larval o cisticerco es una vesícula de contenido líquido, en cuya pared interior se encuentra el
escólex invaginado. Mide aproximadamente medio centímetro (de 2 a 15 mm), y se distingue en la carne
infectada por su color blanquecino, lo que lleva a sus nombres populares en otros países, donde es llamada
"granizo", o "tomatillo". En el Perú y en Latinoamérica en general, el nombre más usual es "triquina". Esta
denominación es obviamente equívoca puesto que alude al nemátodo Trichinella spiralis, y la confusión debe
haberse originado en el hecho de que ambos tienen al cerdo como principal hospedero intermediario.

Patogenia

Se atribuyen a la teniasis síntomas gastrointestinales genéricos, como malestar abdominal, balonamiento,


flatulencia, o pérdida de peso. No hay ningún estudio controlado que confirme esto; la mayoría de pacientes
con teniasis en estudios de campo no refieren síntomas, y menos de la mitad de ellos han notado haber
pasado proglótidos con las deposiciones.

Por el contrario, la infección con la forma larvaria o cisticercosis es un problema de salud pública en la
mayor parte del mundo. La cisticercosis humana es producida por la ingesta de huevecillos de T. solium,
usualmente en el portador de una tenia adulta o en alguien de su ambiente cercano. Se tendía a implicar el
riego con aguas servidas y la ingesta de vegetales de tallo corto como fuentes de contaminación, pero la
evidencia epidemiológica no apoya esta hipótesis (4). Luego de ser ingeridos, la envoltura de los huevos es
disuelta y los embriones son liberados en el intestino delgado, atraviesan la mucosa intestinal activamente, y
llegan al torrente sanguíneo a través del cual son transportados a los diversos tejidos del organismo.

Al parecer la infección se establece en los diferentes tejidos pero sobrevive por mayor tiempo en lugares
inmunológicamente protegidos, como el sistema nervioso o el globo ocular. La infección en otros lugares del
organismo es raramente detectada. La cisticercosis subcutánea es asintomática pero de gran ayuda, puesto
que permite confirmar el diagnóstico mediante una biopsia en los pocos casos en los que se le encuentra. La
cisticercosis muscular es por lo general encontrada casualmente cuando se toma una radiografía por otras
razones, y la cisticercosis miocárdica es usualmente un hallazgo de necropsia, no relacionado a la causa de la
muerte.

Referencia Bibliográfica:

– Náquira, C. Taenia solium: biological cycle and characteristics. In: García HH, Martinez SM: Taenia
solium Taeniasis / Cysticercosis. 2da. Edición. Lima: Ed. Universo; 1999: 7-14

33.-Se trata de paciente femenino de 19 años que acude a consulta refiriendo presentar 6 meses sin regla.
Se manifiesta preocupada por su sobrepeso, ha estado a dieta y ha perdido 6 kg. en 8 meses. Actualmente
pesa 46 Kg. con talla de 1,65 mts. Signos vitales dentro de sus parámetros normales. Se realiza prueba de
embarazo con resultado negativa. ¿Cuál es la causa más probable de su amenorrea?:

a) Disgenesia gonadal.
b) Hipogonadismo hipogonadotropo.
c) Síndrome de ovario poliquístico.
d) Adenoma hipofisario.
El hipogonadismo femenino se caracteriza por un fallo gonadal debido a la alteración del propio ovario o
secundario a un fallo hipotálamo-hipofisario.
Este fallo de la gónada puede ocurrir en distintos momentos de la vida y por causas diversas, lo que va a
condicionar una presentación clínica diferente.
El ovario posee dos funciones relevantes: producir gametos femeninos (oogénesis) y secretar hormonas
esenciales en la regulación de la función reproductora y que influyen en la diferenciación y el desarrollo de
los órganos sexuales (hormonogénesis).
Los mecanismos celulares y moleculares responsables de la producción de oocitos y hormonas por la gónada
femenina son parcialmente independientes entre sí. Sin embargo, ambas funciones son llevadas a cabo de
modo concertado gracias a la acción de un complejo sistema de control que implica interacciones múltiples
entre el hipotálamo, la hipófisis y el ovario.
Para la adquisición de la capacidad reproductora a partir de la pubertad se requiere una adecuada
formación e integración funcional durante las etapas tempranas del desarrollo. Los elementos que componen
este eje reproductor son: 1) el sistema neuronal hipotalámico responsable de la producción del neuropéptido
GnRH; 2) las células gonadotropas de la hipófisis anterior que secretan LH (hormona luteinizante) y FSH
(hormona folículo estimulante), y 3) el ovario.
Hay que destacar que la formación de la gónada y de los elementos hipotálamo-
hipofisarios del eje gonadotropo se llevan a cabo de modo independiente en etapas tempranas del desarrollo
embrionario, produciéndose posteriormente su integración funcional en circuitos de retroalimentación
positivos y negativos, esenciales para una correcta función reproductora.

FALLO GONADAL PRIMARIO O HIPOGONADISMO


HIPERGONADOTRÓFICO
Incluye a aquellos pacientes en los que la producción y acción de los esteroides están reducidas. En los
primeros 4 años de vida y a partir de los 9-10 años, la hipófisis, al faltarle la retroalimentación negativa de
estos esteroides, aumenta la producción de gonadotropinas. Sin embargo, la elevación de las gonadotropinas
no puede normalizar la función gonadal.
Las causas pueden ser congénitas y adquiridas.

Etiología común a ambos sexos


Alteraciones congénitas del receptor de las gonadotropinas
Recientemente se han clonado y mapeado los genes de los receptores de
as gonadotropinas (LH-R y FSH-R) (20), estando ambos en 2p21, y se han identificado diversas mutaciones
y deleciones que conducen a una disminución de la función de estos receptores, independiente de las
gonadotropinas (hipogonadismo hipergonadotropo); en otros casos se produce un aumento de la función de
estos receptores.
Mutaciones que ocasionan pérdida de la función del receptor de la LH
Hasta la fecha se han encontrado seis sustituciones de una base del gen del LH-R que conducen a dos
mutaciones nonsense o sin sentido que ocasiona un codón de parada, y a cuatro mutaciones missense o
mutaciones con error de sentido en los que cambia un aminoácido en la proteína. La mayoría son homocigotos
y se considera que el trastorno es A-R. Clínicamente, existe una forma grave y una menos grave, lo que tiene
relación con la actividad del receptor. En las mujeres hay amenorrea, falta de respuesta del ovario a la LH y
respuesta normal a la FSH con desarrollo folicular normal.
Mutaciones que ocasionan pérdida de la función del receptor de la FSH
Son más raras. Clínicamente las mujeres homocigotas presentan disgenesia ovárica e hipogonadismo
hipergonadotropo, mientras que las mujeres heterocigotas son fenotípicamente normales.

BIBLIOGRAFÍA
1. Cañete R, Jiménez L. Ontogenia y diferenciación gonadal. Gónadas. Actualizaciones en Endocrinología.
McGraw-Hill Interamericana. Capítulo 1, 2000.
2. Barrio R, Ezquieta B. Mecanismos genéticos de la diferenciación sexual: sus alteraciones.
Gónadas. Actualizaciones en Endocrinología. McGraw-Hill Interamericana. Capítulo13,
2000.
3. De Roux N, Morel Y, Hardelin JP. Genetic anomalies of the gonadotropic axis. Rev Prat
1999; 49 (12): 1277-1282.
4. Tena-Sempere M, Aguilar E, Pinilla L. Fisiología del eje hipotálamo-hipofiso-ovárico. Gónadas.
Actualizaciones en Endocrinología. McGraw-Hill Interamericana. Capítulo 2, 2000.
5. Labarta JI y cols. Hipogonadismo hipergonadotrópico. En Pubertad normal y patológica, 2. º curso de
postgrado. Valladolid, 1996.
6. Veldhuis JD. Neuroendocrine mechanisms mediating awakening of the human gonadotropic axis in
puberty. Pediatr Nephrol 1996; 10 (3): 304-317.
7. Hopwood NJ. Pathogenesis and management of abnormal puberty. Spec Top Endocrinol
Metab 1985; 7: 175-236.
8. Minagawa M, Yasuda T, Niimi H J. Spinal and femoral bone mass accumulation during normal adolescence:
comparison with female patients with sexual precocity and with hypogonadism. Clin Endocrinol Metab 1996;
81 (3): 1248-1253.
9. Park KH, Lee SJ, Kim JY, Kim JY, Bai SW, Kim JW. A concomitant decrease in cortical and trabecular
bone mass in isolated hypogonadotropic hypogonadism and gonadal dysgenesis. Yonsei Med J 1999; 40 (5):
444-9.

34.- Masculino de 55 años que acude consulta por presentar dolor ocular y fotofobia intensa. En la
exploración ocular se observa ojo rojo principalmente alrededor del limbo corneal, pupila en miosis y
depósitos blanquecinos en endotelio. Se realiza la toma de presión ocular la cual es de 10 mm Hg (normal
>16). El diagnóstico probable corresponde a:

a) Uveítis.
b) Queratitis.
c) Conjuntivitis infecciosa.
d) Glaucoma agudo.
La uveítis se define como la inflamación de la úvea, lámina intermedia del ojo que se encuentra entre la
esclerótica y la retina, la cual aporta la mayor parte del suministro sanguíneo a la retina. La uveítis es una de
las causas del ojo rojo.1 Suele acompañarse de disminución de la agudeza visual, de curso lento y progresivo y
dolor. No produce secrecciones externas lo que la distingue de otras causas de ojo rojo, como la blefaritis,
conjuntivitis y el chalazión. Por extensión se llama uveítis a cualquier tipo de inflamación del interior del ojo.
Suele cursar con cierta intolerancia a la luz y requiere de un exhaustivo oftalmológico. Puede afectar a uno o
ambos ojos.

Tipos:

Se suele clasificar en cuatro tipos: anterior, intermedia, posterior y panuveítica'.

• Anterior. Es la más frecuente, entre el 70 y 90 por ciento de las uveítis. Es una inflamación del iris
del ojo, córnea y cuerpo ciliar,2 provocada a veces por una enfermedad autoinmune como la artritis
reumatoide o la espondilitis anquilosante, aunque en la mayoría de los casos su causa es desconocida.1
Cursa con ojo rojo, conjuntiva irritada, dolor y pérdida de visión parcial.

• Intermedia o parsplanitis. Es la inflamación de la pars plana, área estrecha ubicada entre el iris y la
coroides. Generalmente es un proceso leve que afecta a los hombres jóvenes y no se asocia a ninguna
otra enfermedad. Es posible que haya una asociación con la enfermedad de Crohn y con la esclerosis
múltiple. A veces se complican produciendo hemorragias o depósitos de material inflamado en la pars
plana.1

• Posterior. Es la inflamación de la coroides o (coroiditis). Si se afecta también la retina se llama


coriorretinitis. Puede producir pérdida de visión de intensidad variable, dependiendo del tamaño y la
localización de la cicatrización. Si se afecta la parte central de la retina, denominada mácula, la
visión central se deteriora.

• Panuveítis: se ve afectada toda la úvea, es decir, los segmentos anteriores y posteriores del interior
del ojo.

REFERENCIA BIBLIOGRAFICA:

Olitsky SE, Hug D, Smith LP. Disorders of the Uveal Tract. In: Kliegman RM, Behrman RE, Jenson HB,
Stanton BF, eds. Nelson Textbook of Pediatrics. 18th ed. Philadelphia, Pa: Saunders Elsevier; 2007:
chap.628.
35.-Se realiza una endoscopia digestiva a un paciente de 57 años, se reporta el diagnóstico de sospecha de
“esófago de Barrett”. ¿Cuál es, entre las siguientes, la conducta más adecuada?:

a) Esperar el resultado de la biopsia antes de tomar decisiones, para corroborar y tener seguridad del
diagnóstico.
b) Indicar tratamiento médico con inhibidor de la bomba de protones 40 mg día en ayuno durante 1 año y
regresar a valoración.
c) Prepararlo para la realización de funduplicatura.
d) Realizar esofaguectomía.

Esófago de Barret.
La Enfermedad por Reflujo Gastroesofágico es una de las causas más frecuentes de consulta médica en la
práctica diaria y esta se puede acompañar de Esofagitis por Reflujo (es la inflamación del esófago causada
por el reflujo del contenido gástrico al esófago), La Esofagitis por Reflujo se diagnostica mediante
Endoscopía, y este procedimiento permite la biopsia, la cual nos permite confirmar la presencia de Esófago
de Barret, una complicación poco frecuente pero de gran relevancia clínica.
El Esófago de Barret es una lesión premaligna, que predispone al Adenocarcinoma Esofágico,
patología neoplásica que ha aumentado su incidencia en los últimos años, incluso desplazando al Carcinoma
Epidermoide.

DIAGNOSTICO.
I- Presentación Clínica. La presencia de EB no provoca síntomas por sí mismo. Más bien, los síntomas
son una consecuencia de la Enfermedad por Reflujo Gastro-Esofágico (ERGE) o sus complicaciones. La
mayoría de los pacientes tienen antecedentes prolongados de síntomas de RGE como pirosis y regurgitación
(2, 6, 10).
Otros pacientes, aproximadamente la tercera parte, se presentan oligo ó asintomáticos (2, 8, 13), lo que
sugiere una disminución de la sensibilidad al reflujo ácido por parte del epitelio de Barret. De hecho, más del
90% de los pacientes con EB no buscan atención médica y el trastorno pasa inadvertido hasta que el proceso
se complica por el desarrollo de cáncer, debutando con disfagia (por formación de estenosis) o con
hemorragia digestiva (por ulceraciones profundas de la mucosa lesionada). (10)
II- Fibroendoscopía Digestiva Alta con toma de Biopsia. La endoscopia con biopsias dirigidas
constituye el patrón oro para el diagnóstico del EB (2, 6). Sin embargo, debido a la gran cantidad de
pacientes con RGE y la baja frecuencia de EB, no parece costo-efectivo el realizar endoscopia a todos ellos
(13). Se ha sugerido que los siguientes pacientes con RGE deben ser sometidos a una endoscopia (9):
· RGE complicado (disfagia, estenosis, úlcera, hemorragia);
· RGE con esofagograma que muestre patrón reticular o seudomembranas;
· RGE con sintomatología persistente a pesar del tratamiento;
· RGE asociado a esclerodermia.
Se sospecha EB en la endoscopia por la presencia de epitelio color naranja a rojo, aterciopelado, de tipo
gástrico; que contrasta con el color rosado blanquecino del epitelio esofágico normal (1, 2, 4, 8). La línea de
transición entre ambos epitelios (cambio mucoso) puede ser regular (circunferencial) ó, más
frecuentemente, irregular (en forma de lengüetas o islotes)(1, 2, 8,). La extensión de la metaplasia también
es variable, pudiendo abarcar desde 2 cm de longitud a partir de la unión gastro-esofágica (UGE) o puede
extenderse incluso hasta el esófago cervical (1).

Para confirmar el diagnóstico debe obtenerse la biopsia. Las muestras deben tomarse, fundamentalmente,
del límite de la UGE y hasta 1 ó 2 cm por debajo de la misma, ya que esta es la zona de mayor riesgo de
desarrollar un Adenocarcinoma, y es en esta parte más proximal del segmento metaplásico donde se sitúa,
principalmente, el epitelio columnar especializado (2). Por supuesto que deben tomarse muestras para
biopsias de todo el segmento metaplásico y, especialmente, de aquellas zonas en las que se observe alguna
alteración macroscópica.
Microscópicamente pueden identificarse tres tipos de epitelio de Barret (1, 2, 4, 8)
Metaplasia gástrica fúndica: es similar al epitelio del cuerpo o del fondo gástrico, y presenta células
parietales y principales;
Metaplasia gástrica cardiaca o transicional: como el epitelio del cardias gástrico, que exhibe acúmulos
profundos de glándulas mucosas y criptas; y
Metaplasia tipo intestinal o también llamado epitelio columnar especializado: que tiene características
de la mucosa gástrica y de la intestinal, que presenta células caliciformes dispersas entre las células
cilíndricas.

BIBLIOGRAFIA
1. Badaloni AE. Enfermedad por reflujo gastroesofágico. En: Ferraina P, Oria A, ed. Cirugía de Mitchans,
Vol 1. 5ª edición. Buenos Aires, Argentina. Editorial El Ateneo. 392-399.
2. Cucarella JF. Esófago de Barret. Gaceta de la sociedad española de patología digestiva [en línea] 1999.
[fecha de acceso 10 de noviembre de 2004] . URL. Disponible en:
http://www.sepd.org/fcontinuada/fcjul99.htm
3. Beers MH. Trastornos del esófago. Cáncer esofágico. En: El Manual Merck, 17ª edición. Edición
electrónica en CD-ROM. Madrid, España: Ediciones Harcourt: 1999; sección 3, cap 20.
4. McGarrity TJ. Barret`s esophagus: the continuing conundrum. Surveillance should be confined to the
surgically fit. BJM 2000; 321: 1238-1239
5. Heading R. ¿Es el seguimiento endoscópico del esófago de Barret una pérdida de tiempo y esfuerzo?
XXX Congreso Chileno de Gastroenterología. Medwave. Año 4. Nº 2. [en línea] Edición Marzo 2004.
[fecha de acceso 28 de diciembre de 2004]. URL. Disponible en:
http://www.medwave.cl/congresos/Gastro2003/1/2.act
6. Murra Saca JA. Esófago de Barret. Atlas de Video Endoscopia Gastrointestinal de El Salvador.[en
línea] 2004 [fecha de acceso 10 de noviembre de 2004] . URL. Disponible en:
http://www.gastrointestinalatlas.com/Espanol/Esofago/Esofago_de_Barrett/esofago_de_barrett.html
7. Ayre AM, Benitez Fernandez A, Cocco JE, y col. Tratamiento del cáncer de esófago: revisión. Revista
de Postgrado de la VI Cátedra de Medicina [en línea] 2003 [fecha de acceso: 15 de enero de 2005]; 126:
37-41. URL. Disponible en: http://med.unne.edu.ar/revista/revista126/tratamiento.htm
8. Crawford JM. El tracto gastrointestinal. Esófago de Barret. En: Cotran, Kumar, Robbins, ed. Patología
Estructural y Funcional. 5ª edición. Madrid, España: Ediciones McGraw-Hill-Interamericana. 845
9. Rodriguez A. Esófago de Barret. Boletín de la Escuela de Medicina. Universidad Católica de Chile. [en
línea] 1998 [fecha de acceso: 28 de diciembre de 2004] Vol 27. Nº 1. 1998. URL

36.-En la enfermedad por reflujo los siguientes pacientes tiene una indicación más clara de tratamiento
quirúrgico:

a) Paciente de 68 años de edad con esofagitis erosiva y antecedentes de infarto de miocardio hace 2 años
con insuficiencia cardiaca.

b) Paciente de 79 años con hernia hiatal de mediano tamaño y pirosis frecuente que responde bien a 20
mg/día de omeprazol en 6 semanas.

c) Femenino de de 45 años con molestias epigástricas tipo flatulencia y pirosis intermitente que responde
sólo ligeramente al tratamiento con omeprazol 20 mg

d) Masculino de 56 años con pirosis diaria diurna y nocturna de más de 10 años de evolución que
durante el tratamiento con 40 mg de esomeprazol se presenta asintomático y a la supresión o
disminución de dosis reinciden de forma inmediata los mismos síntomas.

¿QUÉ ENFERMOS DEBEN SER ENVIADOS A TRATAMIENTO QUIRÚRGICO?


1. Aquellos con sintomatología persistente, que afecta su calidad de vida y que dependen de la ingesta
constante de medicamentos.
2. Falta de apego al tratamiento médico.
3. Pacientes que para el control de sus síntomas requieren de un aumento progresivo de las dosis habituales
de omeprazol (20 mg), rabeprazol (20 mg), lansoprazol
(30 mg), pantoprazol (40 mg) o esomeprazol (40 mg).
4. Esofagitis erosiva que requiere tratamiento a largo plazo y con riesgo de complicaciones.
5. El enfermo con síntomas recurrentes y que decida de manera expresa la cirugía, siempre que no exista
una contraindicación.
La cirugía antirreflujo es una alternativa de tratamiento a largo plazo en enfermos bien seleccionados.
Es indispensable establecer con absoluta certeza el diagnóstico de ERGE con o sin hernia hiatal. El
fundamento del tratamiento quirúrgico es aumentar la eficacia de la barrera antirreflujo
Las complicaciones de la enfermedad como: Estenosis, úlcera o esófago de Barrett no son indicaciones
absolutas de cirugía.1

Los predictores de buena respuesta al tratamiento quirúrgico son:


1. Respuesta adecuada a tratamiento farmacológico.
2. Pacientes con pHmetría anormal. En los enfermos con pHmetría normal y ERGE se han informado malos
resultados en comparación con los que son llevados a funduplicatura con pHmetría anormal.
3. Personas menores de 50 años.2,3
La efectividad de la cirugía se ve reducida en sujetos con reflujo refractario a tratamiento médico,
enfermedad documentada de manera deficiente.

Enfermedad por reflujo no erosiva (ERNE)


En los casos de ERNE no se ha determinado bien el papel de la cirugía antirreflujo. Aunque podría
considerarse en caso de algunos síntomas extraesofágicos a nivel laríngeo o pulmonar, o en el caso de un
defecto anat ómico grande como hernia hiatal.4
El grupo consideró que la cirugía antirreflujo laparoscópica representa un avance terapéutico significativo,
sin embargo, los resultados dependen sustancialmente de la experiencia del cirujano. No hay en la actualidad
estudios que comparen el costo-efectividad de la Terapa éutica médica con la quirúrgica. Por lo tanto, la
elección dependerá de la preferencia del paciente en el caso de contar con un grupo quirúrgico entrenado.
(Nivel de Evidencia 2, Grado de Recomendación B).

Referencia Bibliográfica:

1. Armstrong D, Marshall J, Chiba N, Enns R, Fallone C, et al. Canadian Consensus Conference on the
management of gastroesophageal reflux disease in adults. Update 2004. Can J Gastroenterol 2005; 19: 15-
35.
2. De Vault K, Castell D. Update Guidelines for the diagnosis and treatment of Gastroesophageal reflux
disease. AJG 2005: 100: 190-200.
3. Catarci M, Gentileschi P, Papi C, Carrara A, Marrese R, et al. Evidence- Based Appraisal of Antireflux
Fundoplication. Ann Surg 2004; 239: 325- 37.
4. Westcot C, Hopkins B, Bach K, Postman G, Belafisky P, et al. Fundoplicationfor laringo-faringeal reflux
disease. Am J Coll Surg 2004; 199:23-30.
5. Brooks DC, Rattner D. Patient satisfaction following laparoscopic andopen antireflux surgery. Arch Surg
1995; 130: 289-94.
6. Catarci M, Gentileschi P, Papi C, Carrara A, Marrese R, et al. Evidence- Based Appraisal of Antireflux
Fundoplication. Ann Surg 2004; 239: 325-37.
7. Zoring C, Strate U, Fibbe C. Nissen vs Toupet laparoscopic fundoplication. Surg End 2002; 16: 758-66.
8. Freston JW, Triadafilopoulos G. Review article: approaches to the longterm management of adults with
GERD-proton pump inhibitor therapy, laparoscopic fundoplication or endoscopic therapy. Aliment Pharmacol
Ther 2004; 19(Suppl. 1): 35-42.
37.-Femenino de 34 años la cual labora en lavandería, hace varios años presenta enrojecimiento de los
pliegues proximales de las uñas de varios dedos de las manos que ocasionalmente le supuran. ¿Cuál es el
diagnóstico más probable?:

a) Dermatitis de contacto.
b) Paroniquia candidiásica crónica.
c) Liquen plano.
d) Dermatoficia.

Infección por Candida del lecho ungueal que se presenta con mayor frecuencia como una onicolisis asociada
con paroniquia, aunque también se observa la destrucción completa del lecho ungueal y la erosión de la zona
distal y lateral de las uñas de los dedos, sin distrofia ungueal total. La perionixis candidiásica se trata de la
inflamación con eritema, edema, dolor y salida ocasional de pus blanquecino escaso y espeso del reborde
ungueal de uno o varios dedos de la mano. Con frecuencia, se acompaña de onicopatía (discoloración,
onicolisis, distrofia total, etc.).

Predisponentes: Todas las labores manuales que generen humedad. (muy frecuente en amas de casas,
trabajadores de restaurantes, lavanderas, etc.)

BIBLIOGRAFÍA:

1. Crespo Erchiga V. Protocolo diagnóstico de contaminantes. En "Micología para dermatólogos" Ed.


Janssen, Madrid, 1994, pp:49-70.
2. Crespo V, De Luís B, Delgado V, Crespo A y Vera. Espectro etiológico de las onicomicosis en nuestro
medio. CO7. II Congreso Nacional de Micología. Santiago de Compostela. Junio, 1994.
3. Crespo Erchiga V, Delgado Florencio V y Martínez García S. Micología dermatológica. Ed. M.R.A.
Barcelona, 2006.
4. Daniel III CR. The Diagnosis of Nail Fungal Infection. Editorial Arch Dermatol 1991;127:1566-1567.
5. Delgado Florencio V. Protocolo de identificación de dermatofitos. En "Micología para dermatólogos".
Ed. Janssen, Madrid, 1994, pp:27-41.
6. Delgado Florencio V. Estrategia en el diagnóstico y tratamiento de las micosis superficiales. Ed. Aula
Médica, Madrid, 1994.
7. Delgado V, Abad Romero-Balmas J, Armijo Moreno M y Dulanto F. Scopulariopsis brevicaulis como agente
de onicomicosis. Actas Deermo-Sif. 1976; 9-10:693-700.
8. English MP. Nails and fungi. Br J Dermatol 1976; 94:697-701.
9. Fevilhade de Chauvin M. Onicomicosis. Dermatología práctica. 1994; 9:1-2.

38.- ¿Cual de los marcadores séricos para diagnosticar Infarto Agudo al Miocardio traduce lesión celular
(necrosis)?

a) Creatinfosfokinasa.
b) CPK-MB.
c) De4shidrogenasa láctica.
d) Troponina I.

Las troponinas cardiacas son proteínas que forman parte de los mecanismos de regulación de la contracción
del músculo cardiaco, están presentes en las fibras miocárdicas.

La troponina es una proteína globular de gran tamaño, contiene tres subunidades polipeptídicas: troponina C
(fijadora de calcio), troponina I (inhibidora de la interacción actina-miosina) y troponina T (fijadora de
tropomiosina).

Cuando se necrozan las células del tejido miocárdico pierden la integridad de la membrana celular y las
moléculas intracelulares difunden hacia la microcirculación y a los linfáticos. Estas macromoléculas se
detectan en la circulación periférica y constituyen los marcadores bioquímicos específicos de daño al
miocardio. .

-Owen Avril. Cardiac troponins: improved diagnosis and cost benefits. 2001. Clinical Laboratory
International. Volume 25. No. 8. P.14-15.

39.- La siguiente es una helmintiasis que cursa con autoinfección interna, indique cual:

a) Fasciolosis.
b) Trichinellosis.
c) Estrongiloidosis.
d) Trichurosis.
"Strongyloides stercoralis se diferencia por su capacidad de replicación dentro del ser humano hospedador,
característica poco habitual entre los helmintos. La producción de larvas infecciosas dentro del organismo
facilita la perpetuación de los ciclos de autoinfección. De este modo, la estrongiloidosis puede persistir
durante decenios sin necesidad de una nueva exposición del hospedador a larvas infecciosas del exterior. En
el hospedador inmunodeprimido en ocasiones se produce una amplia diseminación de un gran número de
larvas invasoras de Strongyloides, que puede ser letal. Además de su ciclo parasitario de desarrollo,
Strongyloides puede experimentar un ciclo libre de desarrollo en el suelo (fig. 210-1). Esta adaptabilidad
facilita su supervivencia en ausencia de mamíferos hospedadores. Las larvas rabditiformes eliminadas por
las heces se transforman en larvas filariformes infecciosas de manera directa o después de una fase de
desarrollo libre. El ser humano contrae la estrongiloidosis cuando las larvas fecales procedentes del suelo
contaminado penetran en la piel o las mucosas. Después, las larvas..."

Figura 1.
Fases biológicas del ciclovital de Strongyloides
venezuelensis:

1. Huevos embrionados
2. Larva L1
(rabditiforme)
3. Larva L3 (filariforme)
4. Parásito adulto
(hembra partenogenética

Ciclo biológico de Strongyloides stercoralis


Ciclo biológico de Strongyloides stercoralis:

1. Entrada de larva L3 por piel intacta.

2. Llegada de larva L3 a los pulmones.

3. Paso de larva L3 hacia el árbol bronquial y al aparato digestivo.

4. Parásito adulto en las vellosidades intestinales.

5. Producción de huevos.
a) huevo
b) huevo larvado.
c) larva rabditiforme.

6. Ciclo heterogónico en el medio ambiente.


A) diferenciación sexual.
B) larva L3 infectiva.

7. Auto infección por penetración de larva


L3 en mucosa colónica o perianal.

8. Otros hospedadores además del hombre.

Medicina Clínica
Strongyloides stercoralis: Factores de riesgo para estrongiloidosis diseminada.

Por Gemma Pardo Moreno a, Raquel Rodríguez Rodríguez b, M Teresa Campillos Páez c

a Centro de Salud Ángela Uriarte. Área 1 de Atención Primaria. Madrid.


b Área 9 de Atención Primaria. Madrid.
c Centro de Salud Potes. Área 11 de Atención Primaria. Madrid. España.
40.- ¿La diferencia principal entre los estudios observacionales y los experimentales es que en los estudios
experimentales?

a) Los grupos de estudio y de control son del mismo tamaño.


b) El investigador determina cuales sujetos recibirán la exposición.
c) Los grupos de estudio y de control son compatibles y tienen las mismas características.
d) Se usan controles.

Aunque existen múltiples clasificaciones de los diseños de investigación, la mayoría de los autores están
de acuerdo en la existencia de dos grandes tipos: experimentales y observacionales (no experimentales).
La diferencia básica estriba en el método empleado para asignar a los sujetos del estudio a la exposición
o intervención.

Referencia Bibliográfica:
– Ruiz M. A. Epidemiología Clínica, 1ª. ED. 2004; Pág.: 234.

41.- Un varón de 55 años que ha fumado 30 cigarrillos al día desde que tenía 25 años de edad es atendido
porque presenta hemoptisis. No refiere síntomas excepto tos que produce 5 a 10 ml. de esputo cada mañana.
Los resultados de la exploración física y la radiografía de tórax son normales. ¿Cuál de los siguientes es la
causa más probable de la hemoptisis en este enfermo?

a) Carcinoma broncógeno.
b) Bronquitis crónica.
c) Tuberculosis pulmonar.
d) Bronquiectasias.

Por definición (tos con producción de esputo), este paciente tiene bronquitis crónica. Los resultados
normales de la radiografía de tórax no descartan absolutamente un carcinoma, pero sí indican que este
diagnóstico es poco probable. Lo mismo es cierto para la tuberculosis y la bronquiectasia. Un fumador de
este grupo de edad requiere broncoscopia si la hemoptisis no cede pronto o si la situación clínica cambia. La
deficiencia de antitripsina a, es un factor genético que predispone a sufrir enfisema. A diferencia de la
bronquitis crónica, el enfisema se relaciona con poca tos y expectoración o ninguna.

BIBLIOGRAFIA

1. Knutson D, Braun C. Diagnosis and management of acute bronchitis. Am Fam Physician 2002;
65:2039-2044.
2. Braman S. Chronic cough due to acute bronchitis. Chest 2006; 129:95S-103S.
3. Armstrong G. Pinner R. Outpatient visits for infections diseases in the United States: 1980 through
1996. Arch Intern Med 1999; 159:2531-2536.
4. Irwin RS, Madison JM. The diagnosis and treatment of cough. N Engl J Med 2000; 343:1715-21.
5. Monto A. Gravenstein. Elliot M, et al. Clinical signs and symptoms predicting influenza infection.
Arch Intern Med 2000; 160:3243-3247.
6. Aagaard E., Gonzales R. Management of acute bronchitis in healthy adults. Infect Dis Clin N Am
2004; 18:919-937

42.- Una mujer de 25 años visita a su médico familiar por obstrucción nasal y rinorrea. No está embarazada
ni ha tomado medicamentos recientemente, excepto un descongestivo nasal en atomizador, que utiliza en
raras ocasiones para aliviar sus síntomas. Refiere que un episodio reciente de clima húmedo agrava los
síntomas. En la rinoscopia anterior, las cutirreacciones y el frotis citológico nasal no muestran infección o
anomalía anatómica o inmunitaria. ¿Cuál de los siguientes diagnósticos es más probable?

a) Abuso de descongestivo nasal en atomizador.


b) Rinitis alérgica.
c) Rinitis vasomotora.
d) Rinitis na alérgica eosinofílica.

Es probable que los síntomas de esta mujer se deban a rinitis vasomotora, síndrome caracterizado por
obstrucción nasal y rinorrea sin datos de enfermedad inmunitaria o infecciosa nasal. Con frecuencia, los
síntomas de rinitis vasomotora empeoran después que los pacientes sufren tensión emocional o
experimentan un cambio en la temperatura corporal o ambiental, en la posición corporal o clima húmedo. La
rinitis vasomotora no mejora con medicamentos. Los antecedentes de este enfermo no indican abuso de
descongestivos en atomizadores. El interrogatorio y la exploración también descartan embarazo y desviación
del tabique, ambas posibles causas de rinorrea y obstrucción nasal. La rinitis alérgica, que es un trastorno
mediado por IgE, se caracteriza por mucosa nasal pálida, abotagada, eosinofilia nasal y cuti-rreacciones
positivas. Aunque la obstrucción nasal y la rinorrea son síntomas de rinitis eosinófila no alérgica, este
trastorno se caracteriza por eosinofilia nasal pronunciada, que se hubiera identificado en el frotis nasal. Los
pólipos nasales son otros síntomas de rinitis no alérgica eosinófila. No es probable que los síntomas
relacionados con polipósis nasal sean episódicos.

Referencia Bibliográfica:

– Allen R. M. MMS Medicina Interna. 5ª. Edición. National Medical Series. Mc. Graw Hill. 2006.
(capítulo 7IIIE 2 a-h).
43.- Ante la sospecha de una torsión de testículo, ¿Cuál es entre las siguientes la prueba elección?:

a) Ecografía-doppler.
b) Ecografía.
c) Radiografía escrotal.
d) Palpación bimanual.

Ecografía eco-doppler color: Es la prueba diagnóstica de elección al permitir la visualización de la


vascularización testicular pudiendo distinguir si el flujo circulatorio es normal, ausente o aumentado. En la
torsión lo normal es que esté reducido o ausente. Tiene una especificidad del 100% y una sensibilidad del
80%, aunque tiene sus limitaciones en las subtorsiones.
Gammagrafía isotópica testicular con TC 99: Es también un método sensible y específico, pero del que no
siempre se puede disponer en urgencias. Se verá un área de captación disminuida o ausente, rodeado de un
área de captación aumentada por la hiperemia reactiva.

Bibliografía:
1) Sesions AE et al: Testicular torsión: direction, cadem, duration and desinformation. J. Urol
2003, 169: 663-665.
2) Cummings JM el al: Adult testicular torsión. J.Urol. 202, 167: 2109, 167: 2109-2110
3) Bedos F et al: Manual de Urología. Ed Masson SA. Barcelona 1996. 297-335.

44.- Principal mediador químico responsable del broncoespasmo en el paciente con asma:

a).- Histamina.
b).- Serotonina.
c).- PGE2.
d).- LTC4, D4, E4.

En la membrana del eosinófilo se liberan derivados del ácido araquidónico, producto de su metabolismo por la
vía lipoxigenasa que lleva a la producción de Leucotrienos (LTD4 y LTC4), los sulfapéptidos LTS y LTC4 y sus
derivados LTD4 y LTE4 los cuales provocan contracción muscular lisa potente y tiene propiedades
vasoactivas, se describen más de quince productos derivados de la vía lipoxigenasa a través del eosinófilo.
Existen además en la membrana celular del mastocito enzimas como la Adenilciclasa y Guanilciclasa que
catalizan la formación del AMPc y el cad los cuales influyen en diferentes funciones entre las que figuran la
liberación de mediadores para las células sensibilizadas y el proceso contráctil del músculo liso bronquial. La
liberación de mediadores inducida por alergenos es potenciada por agentes colinérgicos y la prostaglandina
F2á al aumentar el cad y por estimulación adrenérgica que disminuye el AMPc o posiblemente por bloqueo
de los receptores de adenosina. En el paciente asmático hay desbalance entre el ATP y el GMP que favorece
la liberación de mediadores químicos.10

Bibliografía:


MAZZEI J. A Asma en el segundo milenio MEDICINA – Volumen 60 – (Supl I), 2000 pp.5-10.

45.- Un varón de 18 años presenta cefalea persistente y fiebre, y después de cinco días, un cuadro de crisis
convulsiva tónica focal. Una tomografia computadorizada de la cabeza muestra una lesión con reforzamiento
anular en el lóbulo frontal derecho y un nivel hidroaéreo en el seno frontal del mismo lado. Lo más probable
es que un aspirado neuroquirúrgico de la lesión muestre:

a) Células mononucleares pequeñas sugestivas de linfoma de Burkitt.


b) Quistes de Toxoplasma gondii y taquizoítos (trofozoítos) .
c) Escherichia coli y Bacteroides fragilis.
d) Estreptococo hemolítico a y mezcla de anaerobios.

El absceso cerebral es una patología que se da entre personas jóvenes o de edad media (entre la segunda y
cuarta décadas de la vida), con una inexplicable predilección por el sexo masculino (Doble de frecuencia).
El 25% de los abscesos cerebrales ocurren en niños menores de 15 años.

De forma rara se presentan en neonatos pero tienen una alta morbi-mortalidad (Wu, Lai et al.2006).

Localización:

La localización de los abscesos está muy relacionada con la etiología de la infección, siendo la predilección de
los abscesos de origen otógeno por el lóbulo temporal o cerebelo, los de senos paranasales por el lóbulo
frontal, o la frecuente multiplicidad de los de origen hematógeno.

Etiología

El absceso cerebral se inicia cuando los gérmenes alcanzan el parénquima cerebral, lo cual se produce a
través de tres vías fundamentales: por inoculación directa o fístulas, en el caso de traumatismos o cirugía;
por una infección en contigüidad (bien por invasión directa, bien por tromboflebitis de las venas emisarias);
o por embolismos sépticos procedentes de infecciones a distancia. Tradicionalmente las infecciones crónicas
del oído y de los senos paranasales
han sido la causa más frecuente de abscesos cerebrales, pero su incidencia ha disminuido con la mejora del
tratamiento de estas infecciones, produciéndose así un relativo incremento de la infección de origen
hematógeno.

Microorganismos más frecuentes: Estreptococo, Estafilococo, Enterobacterias.


Los microorganismos más frecuentes son los estreptococos, 33%-50% anaerobios o microaerófilos.
Generalmente, se obtienen muchos microorganismos en 10%-30% de los cultivos, pero esta cifra puede
aproximarse a 80%-90%, y suelen hallarse cepas anaerobias (son frecuentes las Bacteroides sp.).
En los abscesos secundarios a sinusitis frontoetmoidal, se observan streptococcus milleri y Streptococcus
anginosus. Cuando son secundarios a otitis media, mastoiditis o abscesos pulmonares, por lo general, se
hallan varios tipos de microorganismos: estreptococos anaerobios, Bacteroides, enterobacterias (proteo o
proteus).Los abscesos postraumáticos suelen deberse a S. aureus o a enterobacterias.

En los pacientes inmunodeprimidos transplantados (tanto de médula ósea como de órganos sólidos), son
frecuentes las infecciones fúngicas, la mayoría por Aspergillus fumigatus a partir de una infección pulmonar
primaria.
Lactantes: son frecuentes los bacilos gramnegativos porque la IgM no pasa a la placenta.
Microorganismos que pueden generar abscesos en pacientes inmunodeficientes:
1.toxoplasmosis.
2.nocardia.

RM de un absceso multilobulado

Bibliografía:

1. Awad I, Bay JW, Petersen JM: Nocardial osteomyelitis of the spine with epidural spinal cord
compression--a case report. Neurosurgery 15:254-256, 1984.
2. Byrne E, Brophy BP, Perrett LV: Nocardia cerebral abscess: New concepts in diagnosis, management, and
prognosis. J Neurol Neurosurg Psychiatry 42:1038-1045, 1979.
3.Tonon, E., P. G. Scotton, et al. (2006). "Brain abscess: clinical aspects of 100 patients." Int JInfectDis10
(2):103-9.
4.Tseng, J. H. and M. Y. Tseng (2006). "Brain abscess in 142 patients: factors influencing outcome and
mortality." Surg Neurol 65(6): 557-62; discussion 562.
5.Visani, P., E. Schmutzhard, et al. (2006). "Subcortical deficit pattern after brain abscess: a
neuropsychological study." Eur J Neurol 13(6): 599-603.
6.Wandroo, F., P. Stableforth, et al. (2006). "Aspergillus brain abscess in a patient with acute myeloid
leukaemia successfully treated with voriconazole." Clin Lab Haematol 28(2): 130-3.

46.- Una mujer de 28 Años llega al servicio de urgencias con disnea y dolor torácico pleurítico. También
refiere que en los cuatro días previos tuvo síntomas muy marcados de tumefacción e hipersensibilidad en la
pantorrilla y el muslo derechos. Con base en la presentación clínica se sospecha una trombosis venosa
profunda que podría haber causado embolia pulmonar. ¿Cuál de los siguientes fragmentos de información de
los antecedentes de la enferma apoyan mejor este diagnóstico?

a) Antecedente de tabaquismo.
b) Antecedente de diabetes M. en la familia de la paciente.
c) Antecedente de lesión en la extremidad inferior.
d) Antecedente de hipertensión.

La lesión de las extremidades inferiores puede producir formación de coágulos sanguíneos y desarrollo de
tromboflebitis. Otros factores que contribuyen al desarrollo de trombosis venosa profunda incluyen uso de
compuestos con estrógenos (p. ej., anticonceptivos orales) o inmovilización de la extremidad inferior (p. ej.,
durante cirugía o reposo en cama prolongado) que da lugar a estasis venosa. La hipertensión, la diabetes
cademia y el abuso de drogas intravenosas no tienen relación con la trombosis venosa profunda. La disnea y
el dolor pleurítico sugieren que la trombosis venosa profunda ha causado embolia pulmonar, que es provocada
por desplazamiento de un trombo de las venas de las extremidades inferiores o la pelvis a la arteria
pulmonar.

Factores predisponentes para trombosis venosa profunda.

A mediados del siglo 19 Virchow enunció los tres factores fundamentales en la patogénesis de la trombosis
intravascular: daño de la pared vascular, estasis del flujo sanguíneo y cambios en la coagulabilidad de la
sangre. Hoy, ya próximos al siglo 21 dicha definición continúa vigente.

Factores predisponentes generales. En estudios epidemiológicos, la edad sobre 50 años, la obesidad, la


hipertensión arterial y el tabaquismo son factores que se asocian a riesgo de flebotrombosis.

Daño de la Pared Vascular. El trauma directo del endotelio vascular puede ocurrir durante algunos
procedimientos diagnósticos y terapéuticos efectuados por vía venosa femoral (por ejemplo: cateterismo
venoso para diálisis, estudios cardiológicos, etc.). El cateterismo venoso prolongado para quimioterapia,
hiperalimentación parenteral o monitorización, puede dar origen a flebotrombosis, a pesar del uso de
catéteres de material no trombogénico.
El trauma indirecto de la pared venosa puede ocurrir en contusiones y fracturas. Estudios recientes han
demostrado flebotrombosis entre el 50 y 70% de los pacientes con trauma mayor de tronco o extremidades
inferiores, siendo la lesión venosa endotelial un elemento predisponente inicial.

La trombosis también puede ser inducida por activación de las células del endotelio por citoquinas
provenientes de procesos traumáticos o inflamatorios a distancia.

Los procedimientos quirúrgicos, especialmente los ortopédicos (por ejemplo artroplastías de cadera o
rodilla), u operaciones de la cavidad pelviana (por ejemplo ginecológicas y urológicas) se asocian a un elevado
riesgo de flebotrombosis, el cual es atribuido al daño sufrido por estructuras venosas vecinas.

El daño endotelial, secuela de una flebotrombosis previa, también es un factor predisponente para una nueva
flebotrombosis.

Estasis sanguínea. La disminución de la velocidad del flujo venoso es un factor que favorece la
flebotrombosis. El enlentecimiento es normal en el reposo e inmovilidad muscular, por lo que la sola
permanencia prolongada en cama puede ser una causa predisponente. El enlentecimiento del retorno venoso
también puede tener su origen en un trastorno central, con disminución del gasto cardíaco, tal como ocurre
en la insuficiencia cardíaca.

La inmovilidad "forzada", con ausencia de la función de bomba muscular, propia de viajes prolongados sin
posibilidad de deambular, la inmovilización de una extremidad por esguince o fractura, la inmovilidad de
ambas extremidades por lesión neurológica encefálica o espinal, la inmovilidad por administración de
anestesia general o regional, son todas condiciones que comparten el mismo factor común predisponente: la
menor velocidad del flujo de retorno venoso.

El riesgo de flebotrombosis por enlentecimiento del flujo sanguíneo ocurre si hay compresión venosa
extrínseca, situación frecuente en el embarazo y menos frecuente en tumores o masas pelvianas o
retroperitoneales.

La dilatación venosa, sea del territorio superficial (várices) o profundo (secuelas postrombóticas,
insuficiencia venosa y aneurismas venosos) también es un elemento asociado a trombosis por estasis venosa.

Trastornos de la coagulación o trombofilias. La sangre se mantiene en estado líquido por el equilibrio


entre factores procoagulantes y factores anticoagulantes. Dicho equilibrio puede ser alterado en forma
transitoria por condiciones como el embarazo o durante el uso de anticonceptivos orales, favoreciendo la
coagulación espontánea. El desarrollo de algunos tumores, en especial en el cáncer de páncreas, ovario,
próstata, pulmón, mama, así como en el linfoma no Hodking, predispone a la enfermedad tromboembólica
venosa por mecanismos de hipercoagulabilidad poco conocidos, probablemente relacionados a proteínas
anormales de origen tumoral.

La mayor viscosidad sanguínea por aumento del hematocrito, como ocurre en individuos que viven en altura o
en las policitemias, también predispone a la flebotrombosis.
El déficit de proteínas anticoagulantes naturales (por ejemplo antitrombina III, proteínas C y S), la
mutación en algunos factores (ejemplo: resistencia a proteína C activada) o la aparición o acumulación
anormal de algunas sustancias circulantes como anticuerpos antifosfolípidos u homocisteína, son condiciones
reconocidas de mayor riesgo de flebotrombosis (ver artículo "Hipercoagulabilidad: estudio y tratamiento" en
esta monografía).

Referencias Bibliográficas:

1. Hirsh J., Hoak J. Management of deep vein thrombosis and pulmonary embolism. A statement for
healthcare professionals. American Heart Association, Circulation 1996; 93: 2212-45.
2. Schulman S, Rhedin A.S, Lindmarker P. et al. A comparison of six week with six month of oral
anticoagulant therapy after a first episode of venous thromboembolism. N Engl J Med 1995; 332:
1661-5.
3. The Columbus Investigators. Low molecular weight heparin in the treatment of patients with venous
thromboembolism. N Engl J Med 1997; 337: 657-2.
4. Koopman M.M.W., Prandoni P., Piovella F et al. Treatment of venous thrombosis with intravenous
unfractioned heparin administered in the hospital as compared with subcutaneous low molecular
weight heparin administered at home. N Engl J Med 1996; 334: 682-7.

47.- Masculino de 50años con antecedente de trastornos convulsivos experimenta una crisis de gran mal.
Sus pruebas de laboratorio, tomadas poco después, revelan lo siguiente: sodio sérico a 140 meq/L; potasio
sérico, 4.1 meq/L; cloruro sérico, 97 meq/L; bicarbonato plasmático (HCO3-), 16 meq/L; pH arterial, 7.15 y
Paco2, 46 mmHg. ¿Cuál de los siguientes describe mejor la alteración acidobásica?

a) Acidosis respiratoria.
b) Acidosis metabólica.
c) Acidosis metabólica más acidosis respiratoria.
d) Acidosis respiratoria más alcalosis respiratoria.

El paciente tiene academia (bajo pH arterial), relacionada con bajo Hco3- sérico; por tanto, debe haber
acidosis metabólica. Este trastorno se debe a acumulación de lactato debida a actividad convulsiva. Otro
dato es la gran brecha aniónica. Además, el paciente también tiene acidosis respiratoria, como se manifiesta
por PaCO2 alta. Con frecuencia, la hiperventilación acompaña a convulsiones de gran mal.
Acidosis Metabólica
La acidosis metabólica es un trastorno clínico caracterizado por un descenso en el pH arterial y en la
concentración de HC03 acompañado por una hiperventilación compensadora que se traduce en caida de la
pC02; esta acidosis metabólica se produce de dos maneras: por la adición de ácido o por la pérdida de HC03.

La respuesta del cuerpo a un aumento de la concentración de H+ arterial involucra cuatro procesos:


amortiguación extracelular, amortiguación intracelular, amortiguacion respiratoria y la excreción renal de la
carga de H+.

Sintomatología

La acidosis metabólica afecta habitualmente a tres esferas del organismo: la cardíaca, la neurológica y la
ósea.

La acidosis, sobre todo si el pH se encuentra entre 7.1 y 7.15, predispone a la aparición de arritmias
ventriculares potencialmente fatales y puede reducir tanto la contractilidad cardíaca como la respuesta
inotrópica a catecolaminas.

Se debe evitar la perpetuación de la acidosis láctica inducida por choque, ya que su corrección es crítica
para que se recupere adecuadamente la perfusión tisular. Los síntomas neurológicos oscilan de letargia a
coma y parecen depender más de la caida del pH del líquido cefalorraquideo, que del pH arterial. En general,
estas anormalidades neurológicas son más prominentes en la acidosis respiratoria que en la acidosis
metabólica. En otras ocasiones, los trastornos neurológicos ocurren a consecuencia de la hiperosmolaridad
por elevación de la glucemia más que por un descenso en el pH arterial. La mayoría de las acidosis
metabólicas son agudas; sin embargo, la insuficiencia renal y la acidosis tubular renal pueden asociarse con
acidosis crónica; en estas condiciones, parte de la amortiguación del H+ retenido se lleva a cabo con el
carbonato proveniente del hueso. Cuando esta alteración ocurre en niños, retarda el crecimiento y produce
raquitismo; en los adultos, da lugar a osteítis fibrosa quística y osteomalacia. En pacientes con ATR, la sola
corrección de la acidosis permite la cicatrización del hueso y un crecimiento normal.

Acidosis Respiratoria
La acidosis respiratoria es un trastorno clinico caracterizado por pH arterial bajo, elevación de la pCO2
(hipercapnia) y aumento variable en la concentración plasmática de HC03. La hipercapnia, también es una
compensación respiratoria a la alcalosis metabólica; sin embargo, en esta situación el incremento en pCO2 es
fisiológica y permite al organismo llevar el pH arterial a lo normal.
Fisiopatología
El metabolismo corporal normal produce alrededor de 15,000 mmol de CO2 por día; aún cuando el CO2 no es
en si un ácido, al combinarse con el H20 presente en la sangre, resulta en la formacion de H2CO3 que
aumenta al disociarse en H+ y HCO3 la concentración de iones de H+. Estos cambios, estimulan los
quimiorreceptores que controlan la ventilación pulmonar, especialmente aquellos localizados en el centro
respiratorio del bulbo raquídeo; estímulo que aumenta la ventilación alveolar y consecuentemente la
excreción de CO2. Este mecanismo es muy efectivo, ya que mantiene la pCO2 dentro de limites muy
estrechos (de 37 a 43 mmHg). Cualquier proceso que interfiera con la secuencia normal descrita, desde el
centro respiratorio bulbar, la pared torácica, los músculos respiratorios y el intercambio gaseoso del capilar
alveolar, pueden resultar en la retención de CO2 y en acidosis respiratoria (ver cuadro 9),
Diagnóstico
La presencia de un pH ácido e hipercapnia es en general diagnóstico de acidosis respiratoria. Debido a que la
respuesta corporal es diferente en la acidosis aguda y crónica, el diagnóstico correcto del trastomo es más
complicado que entre acidosis y alcalosis metabólica. Así, en la hipercapnia aguda, la elevación rápida en la
pCO2 se acompaña de un aumento discreto en el HCO3: aproximadamente 1 mEq/L por cada 10 mmHg que se
eleva la pCO2. Si esta alza alcanza 80 mmHg el HCO3 aumentará a 28 mEq/L y el pH caerá a 7.17; esta
compensación no es muy eficiente, ya que si el HCO3 se hubiera quedado en 24 mEq/L el pH hubiera
descendido a 7.1. Por el contrario, en la hipercapnia crónica la reducción progresiva y más lenta del pH
arterial, estimulará la secreción de H+ que se traducirá en reabsorción tubular de HCO3 hacia el líquido
extracelular. Esta compensación renal traerá como consecuencia, que por cada 10 mmHg de elevación de la
pCO2 el HCO3 aumentará 3.5 mEq/L. En consecuencia, si la pCO2 se incrementa crónicamente a 80 mmHg, el
HCO3 plasmático alcanzará una concentración de 38 mEq/L y el pH sólo disminuirá a 7.3.
Para un buen diagnóstico diferencial es necesario contar con una buena historia clínica para caracterizar
bien el trastorno ácido-básico y la existencia o no de factores agregados.

Equilibrio Acido-Base, Bibliografía:

– Alan W. Grogono, M.D.


Chairman and Merryl and Sam Israel Professor
Department of Anesthesiology .Tulane University School of Medicine, New Orleans.

48.- Se trata de un joven de 18 años de edad, acude a su clínica un lunes por la mañana, el motivo de
consulta es la presencia de más de 8 evacuaciones líquidas con sangre, moco y pujo, las últimas 24 horas,
precedidas de dolor tipo cólico que cede posterior a la defecación, además de tenesmo rectal. Llama la
atención que este cuadro inicio hace dos semanas, al principio solo existió disminución de la consistencia de
la materia fecal, hasta que, alrededor del cuarto día, solo había evacuaciones líquidas. Afebril y sin vómitos.
La exploración física demuestra mucosas orales humectadas. Dolor a la palpación de todo el abdomen, sin
rebote, con ruidos intestinales intensos y abundantes. Su presión arterial es de 100/75. ¿Cuál es el
diagnóstico en este caso?

a) Diarrea crónica.
b) Diarrea aguda.
c) Colitis por parásitos.
d) Enfermedad intestinal inflamatoria.

Definición:
Se considera diarrea aguda a la presencia de heces líquidas o acuosas, generalmente en número mayor de
tres en 24 horas y que duran menos de 14 días; la disminución de la consistencia es más importante que la
frecuencia.1 El número de evacuaciones intestinales hechas en un día varía según la dieta y la edad de la
persona. Los lactantes alimentados al seno materno tienen evacuaciones intestinales blandas frecuentes;
ésto no es diarrea.

Epidemiología.

La Organización Mundial de la Salud estima que cada año se presentan 1,300 millones de episodios de
diarrea en niños menores de cinco años en países en desarrollo (África, Asia, excluida China, y América
Latina), que ocasionan 4 millones de muertes, relacionadas en el 50-70% con deshidratación, lo que las ubica
dentro de las principales causas de defunción en estos países. La mayoría de los niños que sobreviven quedan
con algún grado de desnutrición y los desnutridos, no sólo padecen con mayor frecuencia de diarrea, sino que
los episodios son más graves.3 El tercer gran problema asociado a las diarreas, en niños mayores, es el
ausentismo escolar o laboral.

Etiopatogénia.

Los agentes etiológicos más frecuentes son, en orden decreciente, virus, bacterias y parásitos.

Los virus son la causa principal de las diarreas deshidratantes en niños menores de dos años, siendo los
rotavirus del grupo A, serotipos G1 y G3, los responsables de la mayoría de los episodios.8 La diarrea
osmótica que ocasionan se debe a que lesionan en forma focal las células de las vellosidades del intestino
delgado, disminuyendo la producción de las enzimas encargadas de la absorción de la lactosa, entre otros
disacáridos, lo que aumenta la osmolaridad en la luz intestinal y produce mayor secreción de agua que se
pierde a través de las heces. Sin embargo, las células de las criptas encargadas de reparar las vellosidades
lesionadas, migran para substituirlas en un periodo de 24 a 72 horas, con lo que desaparece la diarrea.9

Las enterobacterias, como Escherichia coli, Salmonella sp, Shigella, Campylobacter jejuni y Vibrio cholerae
01, producen diarrea a través de diversos mecanismos: 1. Liberación de enterotoxinas (V. cholerae 01, E. coli
enterotoxigénica) que estimulan la adenilciclasa y aumentan la secreción intestinal de agua, sodio y cloro; 2.
Enteroinvasión (E. coli enterohemorrágica) con disolución de la mucosa y del borde en cepillo y 3.
Proliferación intracelular, previa invasión de la mucosa ( Shigella) con aparición de sangre en las
evacuaciones, paso de microorganismos a la circulación sanguínea (bacteremia) y algunas veces sepsis.
Shigella dysenteriae, produce además enterotoxinas que estimulan la secreción de agua y sodio en el
intestino delgado y neurotoxinas que causan manifestaciones neurológicas, desde convulsiones hasta estado
de coma. En los dos últimos decenios ha sido posible documentar el papel de otros patógenos como
Campylobacter y Yersinia. Asimismo, se ha avanzado en los mecanismos fisio-patogénicos de las infecciones
producidas por algunos tipos de Escherichia coli, Clostridium difficile, Salmonella y Aeromonas hydrophila.9
Algunos parásitos producen diarrea sanguinolenta (Entamoeba histolytica) o diarrea prolongada (Giardia
lamblia). El mecanismo de producción de diarrea es a través de enteroinvasión (E. histolytica) o
enteroadhesión (G. lamblia), en donde el daño más grave es de carácter nutricional.

El reservorio de los agentes de las diarreas infecciosas es el hombre, exceptuando el caso de Salmonella que
es de origen animal. La fuente de infección es la materia fecal del hombre infectado, sintomático o
asintomático, siendo este último más peligroso ya que no presenta datos clínicos que permitan identificarlo;
en el caso de algunos virus, las secreciones nasofaríngeas pueden ser el origen. La infección genera
inmunidad específica, la cual es de duración prolongada en la etiología viral y más corta en la bacteriana.

El mecanismo de transmisión clásico es el ciclo ano-boca, entendido éste como la salida del agente infeccioso
con la materia fecal y su ingestión, casi siempre a través de las manos contaminadas o de fomites. Otro
mecanismo de transmisión frecuente es la ingestión de alimentos contaminados o de agua, otras bebidas o
hielo, no sometidos a algún proceso de purificación. Algunos factores de riesgo en el huésped son: el estado
nutricional (círculo vicioso diarrea-desnutrición-diarrea); enfermedades previas de tipo anergizante
(sarampión); ablactación temprana o ausencia de alimentación al pecho materno; saneamiento deficiente
(carencia de agua potable, fecalismo); falta de educación y hábitos higiénicos; ignorancia o patrones
culturales adversos (suspensión de alimentos, no aceptación de medidas sanitarias y de manejo oportuno del
paciente), y económicos (limitantes a la incorporación de obras de saneamiento, adquisición de nutrientes y
búsqueda de atención médica).

Referencias Bibliográficas:

1. World Health Organization. A manual for the treatment of acute diarrhoea for use by physicians and
other senior health workers. Geneve: Program for Control of Diarrhoeal Diseases, World Health
Organization, WHO/CDD/SER/80.2 Rev,2:1990.

2. World Health Organization. Division of Diarrhoeal and Acute Respiratory Disease Control. 25 years of
ORS: Joint VMO/ ICDDR,B Consultative Meeting on ORS Formulation. Dhaka, Bangladesh, 10-12 December
1994. CDR/CDD/95.2

3. Sepúlveda J, Willet W, Muñoz A. Malnutrition and diarrhea. Alongitudinal study among urban Mexican
children. Am J Epidemiol 1988; 127: 365-376.

4. Mota HF, Tapia CR, Welti C, Franco A, Gómez UJ, Garrido MT. Manejo de la enfermedad diarreica en el
hogar, en algunas regiones de México. Bol Med Hosp Infant Mex 1993; 50: 367-75.

5. Encuesta de manejo efectivo de casos de diarrea en el hogar. Dirección General de Epidemiología, SSA.
Consejo Nacional para el Control de las Enfermedades Diarreicas. México, 1993 (Documento interno).

6. Glass RI, Lew JF, Gangarosa RE, Lebaron CW, Ho MS. Estimates of morbidity and mortality rates for
diarrheal disease in American children. J Pediatr 1991; 118: 527-33.
49.- Una mujer de 66 años presenta deterioro al caminar. En la exploración se encuentra marcha
ligeramente espástica, mala posición y sensación de vibración en los dedos de los pies, reflejos de
estiramiento muscular en las rodillas +++ y reflejos aquíleos ausentes. ¿Cuál de los siguientes diagnósticos es
más probable?

a) Hipovitaminosis B12.
b) Esclerosis múltiple.
c) Hidrocefalia con presión normal (NPH).
d) Infección con virus linfotrópico humano de células T tipo I (HTLV-I).

La hipovitaminosis B12 causa degeneración combinada de sistemas. Los pacientes presentan alteración de la
marcha caracterizada por espasticidad y disminución de la sensación vibratoria y la posición. Puede haber
neuropatía leve que produce depresión de los reflejos aquíleos. Como es una causa tratable de marcha
anormal, es importante identificar la deficiencia de vitan B12. La esclerosis múltiple (MS) no se manifiesta en
este grupo de edad. La ausencia de reflejos aquíleos es un dato común de MS; más bien, la hiperactividad de
los reflejos concuerda con los datos de neurona motora superior encontrados a menudo. La hidrocefalia de
presión normal (NPH) es una causa de deterioro de la marcha. Sin embargo, hay donación cognitiva y los
pacientes pueden tener incontinencia urinaria. La alteración de la vibración y la sensación de posición, así
como la ausencia de reflejos aquíleos, no son caracteristicos de NPH. La infección virus linfotrópico de
células T humano tipo I (HTLV-I) es causa de mielopatía y debe sospecharse en pacientes que han recibido
transfusiones de sangre, usuarios de drogas intravenosas o personas que hayan habitado en áreas
endémicas. La adrenomieloneuropatía es un trastorno recesivo ligado a X que guarda relación con
adrenoleucodistrofia, la cual típicamente se presenta en varones jóvenes. El trastorno origina la acumulación
de ácidos grasos de cadena muy larga debido a una imposibilidad para catabolizar estos lípidos. Las mujeres
portadoras manifiestan una paraparesia espástica leve, pero no la gama de datos que se encuentran en esta
paciente.

BIBLIOGRAFIA:

Braunwald, Isselbachre, Petersdorf, Wilson, Martin, Fauci

“HARRISON: PRINCIPIOS DE MEDICINA INTERNA”

Undécima edición D.S.Mc Laren, M.Frigg

50.- Una mujer de 43 años inicia recientemente con fatiga, somnolencia, piel seca, estreñimiento y aumento
de peso de 5 kg. Su tiroides está firme y tiene el doble del tamaño normal. ¿Cuál de las siguientes pruebas
de laboratorio confirma el presunto diagnóstico de hipotiroidismo?

a) Tiroxina sérica (T4).


b) Triyodotironina sérica (T3).
c) Captación de resina T3.
d) Hormona estimulante de tiroides (TSH) en suero.

Hipotiroidismo
La instauración es habitualmente lenta y progresiva. Los síntomas se relacionan con una disminución en la
actividad funcional de todos los sistemas del organismo. Los más clásicos son cansancio, intolerancia al frío
(carácter muy friolero), apatía e indiferencia, depresión, disminución de memoria y de la capacidad de
concentración mental, piel seca, cabello seco y quebradizo, fragilidad de uñas, palidez de piel, aumento de
peso, estreñimiento pertinaz y somnolencia excesiva. En situaciones extremas puede evolucionar hacia la
insuficiencia cardiaca, la hinchazón generalizada (mixedema), insuficiencia respiratoria y abocar al coma
mixedematoso con pérdida de conocimiento que conlleva un alto grado de mortalidad.

Al igual que el resto de enfermedades del tiroides, el hipotiroidismo es más frecuente en el sexo femenino.
Es a partir de los 40-50 años cuando las mujeres tienden a desarrollar con más frecuencia hipotiroidismo de
causa autoinmune (tiroiditis de Hashimoto). El periodo postparto es igualmente propenso a la aparición de
este problema. La cirugía de tiroides y la aplicación de yodo radioactivo representan situaciones de riesgo
para el desarrollo de hipotiroidismo, lo que obliga a controlar evolutivamente la función tiroidea en estos
casos.Los recién nacidos de madres hipertiroideas, hayan recibido o no tratamiento antitiroideo durante la
gestación, deben ser evaluados en este sentido. Las personas en las que se detectan anticuerpos
antitiroideos (antimicrosomales, antitiroglobulina) tienden a desarrollar con el tiempo alteraciones de la
función tiroidea, por lo que deben ser evaluados crónicamente de forma periódica.

La determinación de TSH es el parámetro más sensible para el diagnóstico del hipotiroidismo. Su elevación
es indicativa de que la función del tiroides es insuficiente. Este fenómeno se produce antes de que
comiencen a descender en la sangre las concentraciones de hormonas tiroideas. Generalmente, en el
hipotiroidismo establecido, además de la elevación de TSH, se produce un descenso de T4. El nivel de T3 con
frecuencia se encuentra dentro de la normalidad. Así pues, cuando aparecen síntomas sugestivos, el médico
solicitará una determinación de TSH que es el mejor método para descartar que exista hipotiroidismo.
Puede acompañarse de una determinación de T4 y de anticuerpos antitiroideos si se desea conocer si la
causa se debe a fenómenos de autoinmunidad. En los casos de hipotiroidismo secundario debido a
disminución de la secreción de TSH por parte de la hipófisis, el diagnóstico se basa en confirmar
concentraciones disminuidas de T4 y TSH en la sangre. Cuando la elevación de TSH se acompaña de niveles
normales de T4 la condición es conocida con el nombre de hipotiroidismo subclínico. Si existe bocio puede
ser conveniente realizar una ecografía tiroidea. Cuando existe sospecha de alteraciones en el desarrollo de
la glándula o de deficiencia enzimática, puede ser útil obtener una gammagrafía tiroidea. Si se confirma un
diagnóstico de hipotiroidismo de causa autoinmune, es habitual evaluar la asociación de alteraciones en otras
glándulas como las suprarrenales, paratiroides o gónadas.

REFERENCIAS BIBLIOGRAFICAS:

(1.) Anderson R, Harnes J. 1975. Thyroid hormones secretion rates in growing and mature goats. J Anim Sci
40: 11301135.
(2.) Anke M, Henning A, Grun M, Partschefeld M, Groppel B. 1977. Der einluss des mangan, zink, kupfer, jod,
selen, molybdan und nickelmangels aauf die fortpflanzuggsleistung des wiederkauers. Mathem Natur Reihe
(Leipzig) 26: 283-292.

(3.) Balbuena O. 2003. Nutrición Mineral del Ganado. Sitio Argentino de Producción Animal: 1-5,
www.produccionanimal. com.ar.

(4.) Beckett GJ, Beddows SE, Morrice PC, Nicol F, Arthur JR. 1987. Inhibition of hepatic deiodination of
thyroxine is caused by selenium deficiency in rats. Biochem J 248: 443- 447.

(5.) Brem JJ, Pochon DO, Roux JP, Trulls H. 1998. Exploración diagnóstica de la función tiroidea en ovinos.
Rev Vet 8/9: 23-26.

(6.) Castillo V. 2001. Cambios de la función tiroidea en cachorros alimentados con dietas comerciales con
alto contenido de yodo. On line: http://www.idealibrary.com.

51.- ¿Cuál de las siguientes vacunas son conjugadas?

a) Pneumococo de 7 serotipos, meningococo de cuatro serotipos y Haemophilus influenzae tipo B.


b) Sabín, triple viral y varicela.
c) Influenza, hepatitis A y hepatitis B.
d) BCG, pneumococo polivalente y DPT.

Conjugadas de Haemophilus influenzae tipo b (Hib)

Las vacunas conjugadas de Hib unen covalentemente el polisacárido de Hib con una proteína acarreadora,
que induce una respuesta inmunológica dependiente de células T y memoria inmunológica que no se induce con
el polisacárido solo en la vacuna. Las proteínas acarreadoras autorizadas para infantes incluyen el toxoide
tetánico (vacuna PRP-T); un mutante, toxina diftérica no tóxica (HbOC); y la proteína de membrana externa
de Neisseria meningitidis (PRP-OMP). Una cuarta vacuna (PRP-D), conjugada al toxoide diftérico, está
autorizada sólo para uso en niños mayores (15 meses de edad o mayores) debido a su inmunogenicidad más
baja en infantes.

Referencia Bibliográfica:

1. Ammann AJ, Ashman RF, Buckley RH et al. Use of intravenous gammaglobulin in antibody
immunodeficiency: Results of a multicenter controlled trial. Clin Immunol Immunopathol 1982;
22:60-7.
2. Roifman CM, Lederman HM, Lavis S, Stein LD, Levison H, Gelfand EW. Benefit of intravenous IGG
replacement in hypogammaglobulinemic patients with chronic sinopulmonary disease. Am J Med
1985; 79:171-4.

3. American Academy of Pediatrics, 1994, Red Book: Report of the Committee on infectious Diseases,
23rd ed. Elk Grove Village, IL, 1994.

52.- Femenino de 26 años con antecedente de migraña se presenta con cefalea constante de tipo
compresivo relacionada con fotofobia y temperatura de 39.1°C. Por medio de punción lumbar se extrajo
líquido cefalorraquídeo con una concentración de proteínas de 62 mg/100 mi, glucosa de 76 mg/100 mi y 26
mononucleares/mm3. Dos días después la paciente se encuentra afebril y sólo tiene cefalea al levantarse
¿Cuál de los siguientes es el diagnóstico más probable de esta persona en este momento?

a) Migraña.
b) Cefalea postural posterior a punción lumbar.
c) Meningitis bacteriana.
d) Meningitis aséptica.

Lo más probable es que esta enferma haya tenido meningitis aséptica de resolución espontánea y sufra
cefalea postural posterior a punción lumbar (LP) al despertarse. Otros síntomas pueden ser náuseas, visión
borrosa, acúfenos y vómitos. La disminución de la presión de líquido cefalorraquídeo (LCR) puede ser la causa
de cefalea relacionada con punción lumbar Las opciones terapéuticas son reposo en cama, analgésicos y
parche sanguíneo epidural lumbar que en parte puede taponar un desgarro en la duramadre. Aunque la
paciente tiene antecedente de migraña, es inusual que las cefaleas vasculares se presenten sólo en
bipedestación. La meningitis bacteriana es una complicación rara de LP. Sin embargo, como la paciente está
afebril y sólo ha tenido cefalea en posición erecta, este diagnóstico es improbable. Del mismo modo lo es que
la cefalea se deba a meningitis aséptica persistente. Los enfermos pueden presentar hipersensibilidad y
tensión de los músculos cervicales después de meningitis aséptica. Esto puede confundirse con meningismo
persistente. Sin embargo, es más probable que esta persona sufra cefalea como resultado de complicaciones
de LP.

Bibliografía:
1. Headache classification committee of the International headache Society. Classification and
diagnostic criteria for headache disorders, cranial neuralgias and facial pain. Cephalagia 1.988; 8
(suppl 7): 1-96.
2. Kudrow L. : Diagnosis and treatment of cluster headache. Medical Clinics of North America 1.991;
75: 579-594.
3. Kudrow L. Subchronic Cluster headache. Headache 1987; 27: 197-200.
4. Mathew NT. Cluster headache. Memorias del 1er Congreso de Dolor Craneo- Facial 1.996: 63-96.
5. Helt- Hansen P, Paulson OB, Krabbe AA. Invasive adenoma of the pituitary gland and chronic
migrainous neuralgia. A coincidence or a causal relation-ship. Cephalalgia 1.982; 2:20.
6. Mani S, Dector J. Arteriovenous malformation of the brain presenting as Cluster headache: A case
report. Headache 1982; 22: 1.

53.- El encontrar elevación de a-fetoproteína en sangre de una madre que cursa con aproximadas 14
semanas de gestación, nos hace pensar en que:

a) El feto padece un retraso del crecimiento.


b) El feto es portador de una trisomía 21.
c) El feto tiene una hernia diafragmática.
d) El feto tiene un defecto del cierre del tubo neural.

Los Defectos de Cierre del Tubo Neural (DTN) son problemas congénitos severos en el cierre del tejido
nervioso, del que se forma el cerebro y el cordón espinal. La Anencefalia es el caso más extremo de los DTN.
La bóveda craneana está ausente y sólo queda la porción basal de los huesos frontal parietal y occipital. Su
defecto máximo lo constituye la Craneorraquisquisis.

El Encefalocele es la hernia del tejido cerebral a través de un defecto en el cráneo (3), constituyendo el
segundo tipo y la Espina Bífida es el DTN menos severo. Consiste en la existencia de uno o más arcos
vertebrales defectuosos a través del cual protruye meninges (Meningocele) o meninges, médula espinal y
nervios (Mielomeningocele). El grado más leve lo constituye la Espina Bífida Oculta.

Estas afecciones parecen ser de origen multifactorial. La incidencia varía según la localización geográfica,
edad materna, características raciales y étnicas, sexo y el nivel socioeconómico, además de la hipertermia y
los factores nutricionales. En Estados Unidos de América, la frecuencia es de 1 por cada 1000 recién
nacidos vivos aproximadamente.
Diagnóstico prenatal

1. La elevación de alfafetoproteínas, tanto séricas como en LCR, es un indicador precoz.


2. Ecografía, permite planificar conducta y ofrecer la mejor opción en resguardo de la vida del binomio
madre – hijo; en aquellos pacientes que presenten mielomeningocele no sindromático, se contacta con
los neurocirujanos previo a interrupción del embarazo que en este caso se planificará por vía alta.
3. Estudio citogenético, en algunos pacientes el DTN corresponde a manifestaciones de alteraciones
cromosómicas, como trisomía 18, que deben ser evidenciadas en el período prenatal y evitar acciones
innecesarias al momento del nacimiento; es fundamental además para las parejas estar informados
del pronóstico real.

Referencias Bibliográficas:

1. Jones K L , 1997 Smith’s recognizable patterns of human malformation , 5th edition Saunders
Philadelpia.
2. Emery’s Mueller RF Genética Médica 10 ° Ed Marban S L 2001. Madrid,España ”Genética y anomalías
congénitas” pág 223 - 243.
3. Plaguicidas en Chile La Guerra Química y sus víctimas Ma Elena Rosas 1995.
4. Whittle MJ, Connors JM Prenatal diagnosis in obstetric practice. Blackwell Scientific Publications,
Oxford,1989.
5. Estudio del niño malformado F Cortes; Diálogos en Pediatría IV. Editorial Mediterraneo, Santiago,
Chile.
6. Principios de Neurología Damas, Víctor, Romper Capitulo 38 Enfermedades del desarrollo del
sistema nervioso 6ta ED Mach Gras Hill México, 1997 Pág. 865-874.
7. Progresos en diagnostico prenatal Martínez ML “ Teoría de los múltiples puntos de cierre de tubo
neural” Vol. 8 N·4, 1996 Pág. 161- 172.
54.- Es una de las enfermedades tiroideas autoinmunes organo-específica más frecuentes. Es más común en
la mujer, posee una asociación directa con otras enfermedades autoinmunes y, por lo general, se presenta
con hipertiroidismo, bocio difuso, oftalmopatía, además, en algunos casos coincide con mixedema pretibial:

a) Carcinoma basocelular.
b) Enfermedad de Graves.
c) Adenoma tóxico.
d) Síndrome del eutiroideo enfermo.

Introducción

La enfermedad de Graves-Basedow (EGB) constituye la entidad más importante, por su frecuencia, entre las
enfermedades que producen hipertiroidismo. Se caracteriza por la presentación de la tríada sintomática
constituida por hipertiroidismo, bocio difuso y oftalmopatía (50%). Es más frecuente en la 3ª y 4ª década
de la vida y afecta con más frecuencia a mujeres (7/1 en zonas no bociógenas y 3/1 a partir de los 45 años
en zonas de bocio endémico) 1.

Etiopatogenia

Aunque no se conocen exactamente los factores que inician y mantienen la enfermedad, hay elementos que
permiten encuadrarla en el marco de la patología autoinmunitaria, como la presencia de autoanticuerpos
antitiroglobulina y antitiroperoxidasas (TPO), así como la presencia de inmunoglobulinas antireceptor de la
TSH (TRAb) que son estimuladoras de la función y del crecimiento del tiroides.

Para el desarrollo de la enfermedad parece que es necesaria la intervención de factores genéticos y


ambientales (exceso de Yodo) 2.

Clínica

La enfermedad se caracteriza por la presencia de síntomas de hipertiroidismo (nerviosismo 99%, sudoración


91%, intolerancia al calor y palpitaciones 89%, pérdida de peso 85%, aumento de apetito 69% y síntomas
oculares 55%) y signos clínicos (aumento uniforme del tamaño de la glándula 97%, oftalmopatía 60%,
taquicardia en reposo 90%, temblor 70% y retracción palpebral 38%) relacionados con el aumento de
receptores adrenérgicos que determinan las hormonas tiroideas 1,2.

La historia natural de la enfermedad se caracteriza por fases cíclicas de exacerbación y remisión, de


duración y presentación imprevistas; aunque, hoy en día, está artefactada por los tratamientos utilizados.
En aproximadamente el 25% de los pacientes, especialmente en aquellos con una forma leve de enfermedad,
el proceso se autolimita al año o más, regresando espontáneamente a un estado eutiroideo 3.

En cuanto a la oftalmopatía, hay una forma no infiltrativa o pálpebro-retráctil (afecta más frecuentemente
al sexo femenino, afecta simétricamente a ambos ojos y que evoluciona paralelamente al hipertiroidismo) y
una forma infiltrativa (infiltración del tejido retroorbitario por mucopolisacáridos y linfocitaria con
protusión ocular asimétrica)1.

Otras manifestaciones menos frecuentes son la aparición de mixedema pretibial o dermopatía infiltrativa y,
la acropaquia tiroidea 1.

Referencias Bibliográficas:

1.- Foz M. Enfermedades del tiroides. En Rozman C (ed): Farreras-Rozman Medicina interna. Editorial
Doyma. Barcelona, 1992:1997.

2.- Wartofsky L. Diseases of the thyroid. En Braunwald E, Isselbacher KJ, Wilson JD, Martin JB, Fauci AS,
Kasper DL (eds): Harrison’s principles of internal medicine. Editorial McGraw-Hill. EEUU, 1994: 1930.

3.- De Groot. Graves’ diseases and the manifestations of thyrotoxicosis. En De Groot LJ, Reed Larsen P,
Hennemann G.(eds): The thyroid and its diseases. 1996: 371.

4.- Haynes RC, Murad F. Drogas tiroideas y antitiroideas. En Goodman LS, Gilman A (eds): Las Bases
farmacológicas de la terapéutica. Editorial Panamericana. México, 1982: 1376.

5.- Feliciano DV. Everything you wanted to know about Graves’ disease. Am J Surg 1992, (164): 404.

6.- Harada T, Katagiri M, Ito K. Hyperthyroidism: Graves’ disease and toxic nodular goiter. En Clark OH,
Duh QY (eds): Textbook of endocrine surgery. Saunders, 1997: 47.

55.- Cuando se administra sulfato de magnesio para el tratamiento de a preecampsia-eclampsia y aparecen


signos de sobre dosificación ¿qué antídoto se debe emplear?:

a) Carbonato sódico.
b) Nitroprusiato.
c) Simpaticomiméticos.
d) Gluconato cálcico.
NIVEL DE PRIMER CONTACTO (ATENCION PRIMARIA)

Se debe instruir a todas las embarazadas que deben acudir inmediatamente a un centro de salud en
cualquiera de los siguientes casos:

• Edema que se desarrolla rápidamente (en pocos días).


• Cefalea severa y persistente.
• Dolor en la región abdominal superior.
• Visión borrosa.

Se debe realizar la medición de la presión arterial y un análisis de orina para la detección de proteinuria a
las mujeres que acudan a centros de salud presentando estos síntomas.

Convulsiones

Si se asiste a una mujer con eclampsia en un centro de atención primaria:

1. deben mantenerse las vías respiratorias permeables.

2. se debe colocar a la mujer de costado (posición decúbito lateral izquierda) para evitar la aspiración del
vómito u otras secreciones.

3. si es posible, se debe establecer una vía intravenosa.

4. se debe administrar sulfato de magnesio.

Monitoreo de la administración de sulfato de magnesio: Durante el tratamiento con sulfato de magnesio, se


recomienda realizar un control cada 4 horas, como mínimo, para detectar la presencia de: Reflejo rotuliano,
frecuencia respiratoria superior a 16 por minuto, volumen de orina >100 ml en las 4 horas previas.

- Sobredosis de sulfato de magnesio: Todo centro de salud que utilice sulfato de magnesio debe disponer de
ampollas de gluconato de calcio (1 g) como antídoto para la sobredosis de dicho fármaco.

• Se sugiere medir la presión arterial y administrar antihipertensivos según corresponda.


• Convulsiones recurrentes: en caso de convulsiones recurrentes, se administran otros 2 a 4 g de
sulfato de magnesio por vía IV en el lapso de 5 minutos, tanto para el régimen IM como el IV; la
dosis se determina en función del peso de la paciente.

El sulfato de magnesio es un fármaco usado en el control de las convulsiones eclámpticas, para suprimir o
controlar las contracciones uterinas sean estas espontáneas o inducidas, y como broncodilatador luego del
uso de beta agonistas y agentes anticolinergicos. También tiene indicación como terapia de reemplazo en la
deficiencia de magnesio, como laxante para reducir la absorción de tóxicos del tracto gastrointestinal. El
sulfato de magnesio esta ganando popularidad como tratamiento de inicio en el manejo de algunas arritmias,
particularmente en Torsades de Pointes, y en arritmias secundarias a sobredosis de antidepresivos
tricíclicos o toxicidad digitálica. Esta también considerado clase Ila (probable beneficio) para la fibrilación
ventricular refractaria y la taquicardia ventricular, luego de la administración de dosis de lidocaina y
bretilio.

FARMACODINAMIA

El sulfato de magnesio tiene la capacidad de alterar la excitabilidad de la fibra miometrial, afecta el


acoplamiento excitación – contracción y el proceso mismo de contracción, inhibe la entrada de calcio al
sarcoplasma y reduce la frecuencia de los potenciales de acción. Inhibe también la liberación de
acetilcolina. Por ser estas acciones comunes en las fibras musculares se pueden ver afectadas también la
musculatura voluntaria e incluso las fibras miocárdicas.(1)

CONTRAINDICACIONES

Este fármaco no debe ser utilizado en pacientes con antecedentes de cardiopatía o compromiso renal.

Esta contraindicado su uso concomitante con bloqueadores de los canales de Ca, por el riesgo de parálisis
respiratoria.

INTERACCIONES

El sulfato de magnesio es antagonizado por el calcio, potenciado por los bloqueadores neuromusculares y
sinergizado por opiáceos, barbitúricos y anestésicos.(1)[]

SOBREDOSIS, TOXICIDAD Y TRATAMIENTO

Una hipermagnesemia puede manifestarse por adormecimiento muscular, hipotensión, cambios en el


electrocardiograma, sedación y confusión. El peligro potencial es la parálisis respiratoria. La abolición del
reflejo patelar es un signo de uso clínico para detectar el inicio de una intoxicación por magnesio.

En casos de intoxicación se debe proceder a dar respiración artificial hasta que se le sea administrada una
inyección de calcio intravenosa al paciente. En adultos, la administración EV de 5-10 mEq de gluconato de
calcio al 10% es suficiente para revertir la depresión respiratoria o el bloqueo cardiaco debido a la
intoxicación por magnesio. En casos de extrema hipermagnesemia se requiere de diálisis peritoneal o
hemodiálisis. (3)

La hipermagnesemia en el neonato puede requerir resucitación y ventilación asistida vía intubación


endotraqueal o ventilación con presión positiva intermitente, así como calcio endovenoso.
Bibliografía:

1. Graves C. Fármacos que contraen o relajan el útero. En: Hardman J, Limbird L, Molinoff P, Ruddon R,
Goodman A, eds. Goodman & Gilman. Las Bases Farmacológicas de la Terapéutica. 9 ed. México DF:
McGraw-Hill Interamericana; 1996. pp. 1012-3.

56.- En un paciente diabético no obeso en el cual existe fracaso en su control metabólico posterior a la
aplicación de una dieta adecuada y un programa de ejercicio y que manifiesta francas polifagia, polidipsia y
poliuria, la acción terapéutica inmediata es:

a) Iniciar de inmediato una sulfonilurea en dosis bajas.


b) Ingresar al hospital para iniciar manejo con insulina rápida subcutánea.
c) Iniciar con metformina e insulina rapida.
d) Si no se prefiere la terapia con insulina, añadir una biguanida.

MODIFICACION a la Norma Oficial Mexicana NOM-015-SSA2-1994, Para la prevención, tratamiento y


control de la diabetes mellitus en la atención primaria para quedar como Norma Oficial Mexicana NOM-015-
SSA2-1994, Para la prevención, tratamiento y control de la diabetes.

APENDICE NORMATIVO I
ESQUEMA DE TRATAMIENTO PARA EL PACIENTE DIABETICO NO OBESO
57.- La ubicación más frecuente del embarazo ectópico es:

a) Ístmico.
b) Ampular.
c) Fímbrico.
d) Intersticial.

Los embarazos tubarios constituyen más del 99% de los embarazos ectópicos: Las ubicaciones en las
trompas son las siguientes: ístmico (25%), ampular (55%), fímbrico (17%), intersticial (2%). El embarazo
ovárico ocurre en <0.5 y los abdominales <0.1%.

Bibliografía:
– DeCherney A. (1999) Diagnóstico y tratamiento ginecoobstétricos. México. Ed. Manual Moderno. Pag
384.

58.- Una mujer de 24 años refiere que sus manos se tornan blancas y luego azules en el frío. ¿Cuál de los
siguientes datos sugiere más fuertemente esclerodermia como causa de síndrome de Raynaud en esta
paciente?

a) Engrosamiento cutáneo distal que se extiende en dirección proximal hasta las articulaciones
metacarpofalángicas.
b) Anticuerpos anticentrómero en suero.
c) Anticuerpos antinucleares en suero.
d) Cambios capilares distales en la valoración del lecho ungueal.

La Esclerodermia es una enfermedad crónica, auto-inmune del tejido conectivo la cual es generalmente
clasificada como una enfermedad reumática. También se le conoce como Esclerosis sistémica.

En algunos casos la enfermedad puede evidenciarse en forma localizada a nivel de la piel, mientras que en
otras adicionalmente la afectación ocurre en órganos internos, tales como aparato digestivo, pulmón, riñón,
corazón y otros. Existe la llamada Esclerodermia sine scleroderma, donde se presenta la fibrosis de órganos
internos sin afectación de la piel.

La enfermedad puede aparecer en diferentes grados de severidad y con progresión variable. Pueden
presentarse desde manifestaciones muy leves hasta casos muy severos y de los cuales algunos son
rápidamente progresivos hacia complicaciones renales y pulmonares capaces de provocar la muerte.

El Fenómeno de Raynaud (sensibilidad anormal al frío en las extremidades)

El fenómeno de Raynaud puede convertirse en uno de los signos más molestos de la enfermedad, ya que es
capaz de provocar ulceración muy dolorosa de las yemas de los dedos y eventualmente puede producirse
incapacidad para la adecuada utilización de las manos debido a la gran retracción de la piel.

El engrosamiento de la piel es una característica definitoria de esclerodermia.

Hinchazón de las manos y pies. Dolor y entumecimiento de las coyunturas, engrosamiento de la piel,
degeneración o deformación de las coyunturas de las manos, producto del trastorno circulatorio severo en
los vasos sanguíneos más pequeños.
Problemas en la estructura y forma del tubo digestivo a nivel de Esófago, Estómago e Intestinos, tales
anormalidades pueden causar intolerancia a muchos tipos de alimentos, lo que en definitiva genera un
marcado estado de desnutrición.

Dificultad para respirar debido a marcada fibrosis pulmonar que interfiere el intercambio gaseoso en los
alvéolos pulmonares.

Es frecuente también la afectación del riñón con consecuente insuficiencia renal, generalmente asociado a
hipertensión arterial.

El fenómeno de Raynaud es una anomalía vascular caracterizada por episodios de vasoespasmo de los vasos
sanguíneos periféricos. El vasoespasmo se ve inducido por diferentes estímulos como son el frío y los
diferentes estados emocionales, ocasionándose cambios en el color, temperatura y sensibilidad de la piel en
los dedos y otras zonas acrales.
La piel pasa de modo característico por una serie de alteraciones de color: palidez, cianosis y
enrojecimiento.

Enfermedades cutáneas dependientes de la temperatura.

Temperature-dependert skin disorders. Bibliografía:


– E.H. Page, N.H. Shear
Divisionof Dermatology,Departments of Medicine and Pediatrics.University of Toronto.
Journal of the American Academy Dermatology, 1998;18: 1003-19.
59.- Se trata de masculino de 70 años que consulta por pirosis, disfagia leve ocasional y episodios de
regurgitación nocturna desde hace 15 días. Comenta que desde hace 2 años viene presentando por
ocasiones pirosis y regurgitación. Se reporta endoscopia alta con esofagitis erosiva grave. ¿Qué
tratamiento farmacológico entre los siguientes, es el más adecuado?:

a) Antagonistas de los receptores H2.


b) Sucralfato.
c) Inhibidores de la bomba de protones.
d) Tratamiento combinado con anti-H2 y sucralfato.

Se ha demostrado que existe una relación directa entre la duración de la supresión del acido gástrico y la
menor acidez del esófago.
Los inhibidores de la bomba de protones (IBP) son los agentes preferidos para la curación de las lesiones
agudas y para la mantención de remisión. Los bloqueadores H2 presentan como única ventaja su rapidez de
acción, pero su potencia es menor. Su uso de rutina asociado con los IBP no se recomienda, ya que los IBP
actúan sobre las bombas de ácido activadas y cualquier inhibición de la secreción por otro agente retarda su
máximo efecto Los antiácidos son útiles para síntomas ocasionales y su uso no se contrapone con los IBP.
De no existir factores significativos modificables en los hábitos o en la anatomía, la ERGE debe
considerarse frecuentemente una patología crónica. No existe evidencia de peso que contraindique el uso
crónico de IBP. Los pacientes con esofagitis erosiva son los más susceptibles a desarrollar complicaciones
que los en GERD (endoscopy negative
Gastroesophageal reflux disease).Todos los IBP son útiles pero no son necesariamente iguales. Puede haber
diferencias étnicas en el número de células parietales o polimorfismo del citocromo p450 con diferente
metabolización de los IBP.
La infección por Helicobacter pylori confunde cualquier intento de comparar IBP. La gastritis de predominio
antral puede producir hipergastrinemia e hipersecreción. Si es de predominio corporal puede disminuir la
producción de ácido. Los estudios comparativos deben hacerse en individuos sin esta infección.
La duración del efecto de los IBP es importante. Muchas publicaciones demuestran la mayor duración de la
acción de la acción del esomeprazol, pero clínicamente doble dosis de los otros IBP pueden tener resultados
comparables. Katz y cols compararon los 5 IBP disponibles en pacientes con ERGE, todos resultaron en pH >
4 por al menos 8 a 10 h, pero la duración de acción del esomeprazol fue superior en el rango de los pH
intragástricos entre 2 a 6.
En los pacientes con enGERD el efecto de los IBP no es tan dramático, puede deberse a que no hay tantos
elementos objetivos de juicio como la curación de las erosiones.
Los IBP pueden usarse una vez al día, en la mañana y antes del desayuno. Los pacientes con daño
extraesofágico o cuadros severos obtienen mejor efecto con doble dosis fraccionada
(Antes de desayuno y cena). Esto mejora el control de la acidez nocturna.
En los pacientes cuya pHmetría demuestre que persiste escape nocturno algunos favorecen agregar un
antagonista H2 al acostarse. Rackoff y cols demostraron que 74% de los pacientes presentaban mejoría de
los síntomas nocturnos. Una nueva formulación de liberación y absorción rápida de omeprazol también podría
ser útil en estos casos.
La ausencia de síntomas no significa ausencia de daño. Los casos graves (y por supuesto el Barrett) deben
tener seguimiento endoscópico para asegurar su evolución. Un porcentaje significativo de pacientes sin
Barrett continúan presentando RGE patológico y bajo pH intragástrico a pesar de IBP bien llevado con total
remisión de síntomas.
El primer objetivo es la remisión de los síntomas de reflujo clásico. Esto debe lograrse en la primera semana
si la prescripción es adecuada. Las manifestaciones extraesofágicas tardan varios meses en controlarse,
pero deben exhibir una mejoría rápida en las primeras semanas que confirme la buena orientación de las
medidas indicadas.
La estrategia terapéutica de comenzar el tratamiento en forma poco agresiva y escalarlo si no hay
respuesta, me parece poco adecuada.
Al paciente se le debe insistir que los fármacos son una parte importante del tratamiento pero no la única y
que los cambios de hábitos y costumbres, la baja de peso, el ejercicio
Etc, son claves para el éxito.

Bibliografía:
1.- Miner P, Katz P, Chen Y, Sostek M. Gastric acid control with esomeprazole, lansoprazole, omeprazole,
pantoprazole, and rabeprazole: A five-way crossover study. Am J Gastroenterol 2003; 98: 2616-20.
2. Katz P, Miner P, Chen Y, Sostek M. Effects of 5 marketed proton pump inhibitors on acid suppression
relative to a range of pH thresholds. Am J Gastroenterol 2004; 99: S34.
3.- Rackoff A, Agrawal A, Hila A, et al. Histamine-2 receptor antagonists at night improve GERD symptoms
for patients on proton pump inhibitor therapy. Am J Gastroenterol 2004; 99: S18.
4.- Castell D, Goldlust B, Morelli G, et al. Omeprazole immediate-release oral suspension is more effective
than pantoprazole delayed-release capsules in reducing nighttime gastric acidity in GERD patients. Am J
Gastroenterol 2004; 99: S39.
5. Milkes D, Gerson L, Triadafilopoulos G. Complete elimination of reflux symptoms does not guarantee
normalization of intraesophageal and intragastric pH in patients with gastroesophageal reflux disease
(GERD). Am J Gastroenterol 2004; 99: 991-6.

60.- Masculino de 47 años con datos positivos de alcoholismo crónico, al cual se le diagnóstica pancreatitis
aguda ¿Cuál es la primera medida terapéutica a adoptar?

a) Iniciar antibióticos de amplio espectro.


b) Administración de inhibidores de la bomba de protones.
c) Dieta absoluta.
d) Aspiración nasogástrica.

El 80 % de los pacientes con pancreatitis aguda se tratan mediante medidas de sostén, suspensión de
la vía oral, hidratación intravenosa, analgésicos y alimentación parenteral cuando la vía oral se
restringe más allá de la semana.
La indicación de la suspensión de la vía oral se basa en el dolor y la intolerancia digestiva. No debe ser
prolongada y se debe restablecer secuencialmente luego de 48 horas sin dolor.
La alimentación parenteral no tiene ningún sentido si el restablecimiento de la vía oral se realiza dentro de
los primeros 7 días. De no ser así se sabe que esta patología grave provoca una agresión severa que
determina un estado hípercatabólico por lo tanto debe implementarse soporte nutricional para evitar la
desnutrición y las complicaciones que ella trae consigo (alteración de la modulación de la respuesta
inflamatoria, translocación bacteriana, inmuno supresión, etc.)
El 60 % de esta enfermedad presenta hipermetabolismo (1,5 veces el metabolismo basal), el 40 % son normo
o hipometabólicos. Según la Conferencia de Consenso de Nutrición de la Agresión (Francia 1998), los
triglicéridos no son contraindicados a menos que el paciente presente una hipertrigliceridemia importante.
La necesidad de aporte debe ser de 0,25 a 0,30 g/Kg. La suplementación de micronutrientes
fundamentalmente antioxidantes (vitaminas A, C, E y selenio) y zinc están indicadas.
La suplementación en base a glutamina, inmunomoduladores, nuevas emulsiones lipídicas en base a aceite de
oliva necesitan aun ser confirmadas para la pancreatitis severa.
La vía enteral se debe privilegiar ya que no solo es mas fisiológica sino que también presenta menor costo y
complicaciones habiendo actualmente estudios que muestran un menor índice de morbimortalidad en estos
pacientes ya que la integridad de la barrera intestinal limitaría la sobreinfección bacteriana pero
fundamentalmente la fúngica ( Kalfarentzos 97, Windsor 98, Pupelis 2000).

Referencias Bibliográficas:

– Banks PA, Freeman ML; Practice Parameters Committee of the American College of
Gastroenterology. Practice guidelines in acute pancreatitis. Am J Gastroenterol. 2006
Oct;101(10):2379-400.
– Frossard JL, Steer ML, Pastor CM. Acute pancreatitis. Lancet. 2008;371:143-152.

61.- Paciente masculino de 60 años de edad, con obesidad exógena manifestando cuadro doloroso de 3
días de evolución, súbito, en hipocondrio derecho, irradiado a región escapular derecha, acompañado
de nausea, vómito amargo y de color amarillento, agregándose a las 24 hs del inicio, ictericia de
escleras y tegumentos, acolia y coluria. Como antecedente, refiere cuadros dolorosos similares
desencadenados por la ingesta de alimentos grasos. ¿Cuál el su diagnóstico mas probable?

a) Hepatitis viral.
b) Colangitis.
c) Coledocolitiasis.
d) Pancreatitis.

La forma clásica de presentación de la coledocolitiasis es la coexistencia de dolor, ictericia y fiebre. Sin


embargo, esta tríada sólo se observa en un tercio de los pacientes; en la mayoría, la enfermedad se
manifiesta por sólo uno o dos de estos síntomas.

Habitualmente, el cólico biliar de la coledocolitiasis es complicado, de gran intensidad, y se asocia a estado


nauseoso y vómitos rebeldes. Puede prolongarse durante varias horas y, si se alivia con antiespasmódicos,
tiende a reaparecer precozmente. Se le puede confundir con un cólico ureteral derecho: es útil tener
presente que en este caso, el paciente aparece muy inquieto, mientras que, si el dolor es de origen biliar,
tiende a permanecer postrado en su cama.

La ictericia puede ser fugaz o subclínica, manifestándose sólo por una coluria transitoria. En otros casos, el
enclavamiento de un cálculo en la ampolla de Vater da origen a una ictericia prolongada, que en los enfermos
ancianos puede complicarse con una enfermedad tubular aguda.

La fiebre aparece en brotes aislados, precedidos por escalofríos; durante varios meses, estos episodios
pueden constituir la única manifestación de la enfermedad.

En los pacientes seniles, la coledocolitiasis suele ser causa de anorexia y de pérdida de peso, simulando una
enfermedad neoplásica.

Conviene insistir en que muchos enfermos con cálculos en el colédoco no tienen ningún síntoma que revele su
presencia. Por ello es tan importante, durante la colecistectomía electiva o de urgencia, la cuidadosa
exploración radiológica de la vía biliar. La incorporación rutinaria de este procedimiento ha reducido la
incidencia de coledocolitiasis residual de un 10 por 100, a un 1 por 100 menos. Si durante una laparotomía se
detecta la coledocolitiasis, se procede a la coledocostomía y limpieza del conducto, y se deja una sonda T de
calibre adecuado para el drenaje postoperatorio. Cabe señalar que la coledocostomía aumenta
significativamente la morbilidad y la mortalidad de la cirugía biliar.

Referencia Bibliográfica:

– Rev Gastroenterol Mex. 2004 Nov;69 Suppl 3:112-6. Rojas O., Arizpe B., Marin L., Cesin S., San R.
– Rev Gastroenterol Mex. 2006 Jan-Mar;71(1):16-21. Gutierrez-Bermudez J.A., Adalid-Martinez R.,
Guitron-Cantu A
62.- Se trata de paciente masculino de 3 años de edad que acude a urgencias con una historia de secreción
nasal purulenta y de mal olor unilateral desde hace 5 días. El diagnóstico más frecuente es:

a) Atresia de coanas unilateral.


b) Rinitis crónica por Rinovirus.
c) Cuerpo extraño intranasal.
d) Fibrosarcoma del correte nasal.

Una de las patologías más frecuentes que enfrenta el otorrinolaringólogo en su práctica médica, es la
presencia de cuerpos extraños animados o inanimados en vías aéreodigestivas superiores. La mayoría de las
veces su tratamiento (extracción) no reviste mayor importancia sobre todo cuando no han sido manipulados
por médicos de primer contacto, pero en otras, se convierte en un procedimiento muy difícil de realizar, ya
sea por las complicaciones que se produjeron o por la presencia del mismo.

En la población infantil es más frecuente por la curiosidad propia de esa edad y la tendencia a colocarse una
variedad de elementos en las fosas nasales.
A veces existen ciertas dificultades en el diagnóstico, pero la tríada sintomática de obstrucción nasal,
rinorrea unilateral y halitosis, nos sugiere la posibilidad de un cuerpo extraño hasta que no se demuestre lo
contrario. Independientemente de su naturaleza animada o inanimada, éstos pueden alojarse y/o impactarse
en distintos niveles.
El tratamiento electivo es la extracción por vías naturales de los mismos; en los niños que no colaboran es
necesario recurrir a la anestesia general.

Referencia Bibliográfica:

1. “Manual de Urgencias en Otorrinolaringología.” Manuel Tomás Barberán, Julio García-


Polo Alguacil, Guillermo Til Pérez.

63.- Femenino que desde hace varias semanas le han aparecido de forma eruptiva unas máculas y placas
eritematosas en el tronco, Refiere que hubo una lesión más grande que precedió a las demás. Las lesiones
presentan una descamación fina en la periferia y son discretamente pruriginosas. No existe afectación
palmoplantar, La serología luética es negativa. ¿Qué tipo de pitiriasis es la más probable?:

a) P. liquenoide crónica.
b) P. rubra pilaris.
c) P. rosada.
d) P. versicolor.

La pitiriasis rosada es una enfermedad exantemática autolimitada de causa desconocida que cursa con
lesiones maculopapulosas de aspecto asalmonado en áreas proximales de las extremidades y en tronco,
donde se distribuyen de forma paralela a las líneas de la hendidura (Stulberg DL, 2004).

Existen algunas características que sugieren una etiología vírica (pródromos, predominio estacional,
evolución variable, ausencia de recaídas, hallazgos histológicos...) que algunos autores relacionan con el
herpes virus humano 7, aunque otros lo desmienten (Chuh A, 2004).

Afecta con más frecuencia a niños mayores y jóvenes adultos (10-35 años) y es ligeramente más frecuente
en mujeres (Chuang TY, 1982).

El cuadro típico se inicia con una lesión redondeada u oval, de 4-8 cm. única, generalmente situada en el
tronco, de color asalmonado, discretamente descamativa y en ocasiones ligeramente pruriginosa, llamada
“madre”. Puede acompañarse de síntomas generales leves: astenia, anorexia, odinofagia, cefalea y artralgias.

En 1-2 semanas esta lesión se torna mas escamosa y clara en el centro al tiempo que aparecen otras de
menor tamaño que se diseminan por las extremidades (áreas proximales) y por el tronco distribuyéndose
simétricamente en forma arbórea a lo largo de las hendiduras costales. En la espalda es más evidente (“signo
del árbol de Navidad”) (Stulberg DL, 2004)

La evolución de las lesiones es similar a las de la lesión “madre” desapareciendo en 5-6 semanas, aunque en
algunos casos puede permanecer más tiempo y en otros verse áreas hiper o hipo pigmentadas durante algún
tiempo. La tasa de recurrencia a los 5 años es inferior al 2% (Chuang TY, 1982).

Pitiriasis Rosada

– Chuh A, Chan H, Zawar V. Pityriasis rosea--evidence for and against an infectious aetiology.
Epidemiol Infect. 2004 Jun;132(3):381-90.
– Chuh AA, Chan HH. Prospective case-control study of chlamydia, legionella and mycoplasma
infections in patients with pityriasis rosea.
– Chuh AA, Dofitas BL, Comisel GG, Reveiz L, Sharma V, Garner SE, Chu F. Interventions for
pityriasis rosea. Cochrane Database Syst Rev. 2007 Apr 18;(2):CD005068.
– Chuh AA. Quality of life in children with pityriasis rosea: a prospective case control study. Pediatr
Dermatol. 2003 Nov-Dec;20(6):474-8.
– Hsu S, Le EH, Khoshevis MR. Differential diagnosis of annular lesions. Am Fam Physician. 2001 Jul
15;64(2):289-96] Eur J Dermatol. 2002 Mar-Apr;12(2):170-3.
– Miranda SB, Lupi O, Lucas E. Vesicular pityriasis rosea: response to erythromycin treatment. J Eur
Acad Dermatol Venereol. 2004 Sep;18(5):622-5

64.- La diferencia básica entre una bulimia nerviosa de una anorexia nerviosa es:

a) El ejercicio excesivo.
b) La autoinducción del vómito.
c) La pérdida de control sobre la comida.
d) La preocupación excesiva por la figura y el peso.

ANOREXIA NERVOSA. CONCEPTO:


La anorexia nervosa es una enfermedad psiquiátrica compleja, con importantes manifestaciones físicas y
complicaciones endocrinológicas que afecta principalmente a mujeres adolescentes (1).
Se trata de un trastorno de la alimentación que lleva al paciente a un estado de inanición adquirido a través
de dietas severas o purgaciones cuando ha perdido la percepción de su imagen corporal y tiene un miedo
exagerado a engordar (2).

Es frecuente que asocie la práctica de un intenso ejercicio físico e intelectual


(3) que los pacientes llevan a cabo con una rígida disciplina autoimpuesta, que contrasta con su débil imagen
física.
Se presenta preferentemente en mujeres, aunque la incidencia en hombres está aumentando. Éstos tienden
a ocultar más el trastorno que las mujeres, por lo que la incidencia en el sexo masculino puede haber sido
subestimada.
Suele aparecer al final de la pubertad, aunque en niñas prepuberales son cada vez más frecuentes (4).
La mitad de estos pacientes reducen el peso restringiendo severamente su dieta y se conocen como
restrictores de anorexia; la otra mitad, los pacientes bulímicos anoréxicos, lo hacen mediante purgaciones.
Ambos tipos son graves y pueden conducir a un estado de emaciación total y a la muerte.
BULIMIA NERVOSA. CONCEPTO:
La bulimia nervosa es un trastorno de la conducta alimentaria en el que se produce, de forma compulsiva, un
ciclo de atracón-purgación.
Empieza generalmente en mujeres adolescentes, cuando intentan dietas restrictivas fracasan y reaccionan
comiendo excesivamente. En respuesta a los excesos los pacientes se purgan vomitando, tomando laxantes o
diuréticos (forma purgativa). En otros casos, realizan dietas severas o ejercicio físico intenso para
compensar el atracón (forma restrictiva) (5).
La bulimia nervosa es más común que la anorexia, y aunque los pacientes con bulimia nervosa pura también
están muy preocupados por su imagen corporal y tienen miedo a engordar, a diferencia de la anorexia
nervosa, suelen presentar normopeso o sobrepeso (especialmente los que llevan más tiempo de evolución). No
llegan nunca a un cuadro de malnutrición severa, aunque debido al ciclo atracón-purgación el peso puede
fluctuar mucho.
La falta de un estado de inanición, unido a la frecuente ocultación de los síntomas, hace que el diagnóstico
sea mucho más difícil y, por tanto, más tardío.

BIBLIOGRAFÍA
1. Emans SJ. Eating disorders in adolescent girls. Pediatr Int 2000; 42: 1-7.
2. Arnow BA. Eating disorders. Introduction. J Clin Psychol 1999; 55: 669-674.
3. Blanz BJ, Detzner U, Lay B, Rose F, Schmidt MH. The intellectual functioning of adolescents with
anorexia nervosa and bulimia nervosa. Eur Child Adolesc Psychiatry 1997; 6: 129-
135.
4. Maddox RW, Long MA. Eating disorders: current concepts. J Am Pharm Assoc (Wash) 1999; 39: 378-
387.
5. Kaltiala-Heino R, Rissanen A, Rimpela M, Rantanen P. Bulimia and bulimic behaviour in middle adolescence:
more common than thought? Acta Psychiatr Scand 100 1999; 33-39.
6. Feighner JP, Robins E, Guze SB. Diagnostic criteria ford use in psychiatric research. Arch Gen
Psychiatriy 1972; 26: 57-63.
7. American Psychiatric Association. Diagnostic and Statistical. Manual of mental disorders (DSM-III-R).
Washington D.C. 1987.
8. American Psychiatric Association: Diagnostic and Statistical Manual of mental disorders, 4.a ed.
Washington D.C., 1994.
9. Wiseman CV, Harris WA, Halmi KA. Eating disorders. Med Clin North Am 1998; 82: 145- 159.
10. Fernández Aranda F, Turón Gil V. Delimitación diagnóstica de la anorexia y la bulimia nervosas. Aspectos
prácticos. En Trastornos de la alimentacion. Barcelona, Masson. 1998;

65.- Recibe el siguiente perfil de hepatitis viral: anticuerpos contra Hepatitis C negativos (-); Hepatitis B:
Antígeno de superficie positivo (+), Anticuerpos contra antígeno de superficie negativos (-),
inmunoglobulina G contra Antígeno core positiva (+), Antígeno e (Anti-HBe) positivo (+). Con lo anterior
diagnostica:

a) Infección aguda C.
b) Infección antigua C.
c) Infección aguda por hepatitis B.
d) Infección crónica por hepatitis B, con replicación viral.

La infección aguda de hepatitis B se caracteriza por la presencia de inmunoglobulina M contra antígeno del
core, que no tiene este paciente. La presencia de inmunoglobulina G contra éste mismo antígeno la
caracteriza como crónica, y la presencia de antígeno e determina que existe replicación activa.

CRITERIOS DIAGNÓSTICOS
1. Determinar la presencia de infección crónica por el VHB, que vendrá dictada por la positividad del HBsAg
durante más de 6 meses.
2. Determinar la existencia de replicación viral activa, caracterizada por la presencia en suero del ADN-
VHB. Existen distintos métodos para determinar el ADN-VHB sérico; si se utilizan métodos poco sensibles,
como la hibridación molecular, se pueden obtener resultados falsamente negativos y por el contrario,
técnicas muy sensibles, como la reacción en cadena de la polimerasa, pueden ofrecer resultados positivos en
portadores del VHB en los que la replicación del virus no tiene repercusión clínica relevante.
3. Determinar la existencia de enfermedad hepática y conocer su estadío. La elevación de los valores de
transaminasas constituye el marcador más sencillo de la existencia de enfermedad hepática activa. En tales
casos, la realización de una biopsia hepática permitirá confirmar el diagnóstico de hepatitis crónica, conocer
su grado de actividad, así como el estadio de fibrosis existente.
4. Descartar otras causas de enfermedad hepática. Otros virus de la hepatitis (VHD o VHC), o incluso otras
causas de enfermedad hepática pueden ser los responsables, en asociación o no con el VHB, de la lesión
hepática en portadores crónicos de este virus. El conocimiento de estas otras posibles causas de
enfermedad hepática es imprescindible para planificarel tratamiento oportuno.
La infección crónica por el VHB es un proceso dinámico, sujeto a la interacción entre el propio virus y el
sistema inmune, que tiene como resultado la existencia de distintas fases, con datos virológicos y clínicos
diferenciales.

HEPATITIS CRÓNICA POR VIRUS DE LA HEPATITIS B

(Tabla 1).
Tabla 1. Fases de la infección crónica por el VHB
HBsAg HBeAg ADN-VHB Transaminasas Histología

Inmunotolerancia Positivo Positivo Positivo Normales Normal


Cambios
Mínimos
Hepatitis crónica Positivo Positivo Positivo Elevadas Hepatitis
HBeAg+
crónica

Hepatitis crónica Positivo Negativo Positivo Elevadas Hepatitis


HBeAg -
crónica
Portador sano Positivo Negativo Negativo Normales Normal
del VHB hepatitis
"residual"

BIBLIOGRAFÍA:

1. Kasper DL, Braunwald E, Fauci AS, Hauser SL, Longo DL, Jameson JL. Harrison´s Principles of
Internal Medicine. McGraw Hill. 16 Ed. 1835 p.
2. Benhamou Y, Bochet M, Thibault V et al. Long-term incidence of hepatitis B virus resistance to
lamivudine in human immunodeficiency virusinfected patients. Hepatology 1999; 30: 1302-1306.
3. Chayama K, Suzuki Y, Kobayashi M et al. Emergence and takeover of YMDD motif mutant hepatitis B
virus during long-term lamivudine therapy and re-takeover by wild type after cessation of therapy.
Hepatology 1998; 27: 1711-1716.
4. Chien R-N, Liaw Y-F, Atkins M. Pretherapy alanine transaminase level as a determinant for hepatitis
B e antigen seroconversion during lamivudine therapy in patients with chronic hepatitis B.
Hepatology 1999; 30: 770-4.

5. Dienstag JL, Schiff ER, Mitchell M et al. Extended lamivudine retreatment for chronic hepatitis B:
manteinance of viral suppression after discontinuation of therapy.
6. Lai C-L, Chien R-N, Leung NWY et al. A one year trial of lamivudine for chronic hepatitis B. N Engl J
Med 1998; 339: 61-8.

66.- Recién nacido de 38 semanas de gestación en su segundo día de vida, que presenta una dermatosis
diseminada a cara y tronco caracterizada por vesículas y pústulas, que respeta palmas y plantas. ¿Cuál es el
diagnóstico?

a) Eritema Tóxico.
b) Melanosis pustulosa.
c) Dermatitis atópica.
d) Rubéola congênita.

El eritema tóxico es una entidad sin significado patológico que se manifiesta con vesículas y pústulas sobre
una base eritematosa, respeta palmas y plantas. Aparece entre el primero y tercer día de vida. Están
formadas por un infiltrado de eosinófilos, el cultivo es estéril y desaparece en la primera semana. En
cambio, la melanosis pustulosa se presenta al nacimiento, se localiza en palmas y plantas, las lesiones están
formadas por PMN, el cultivo es estéril y desaparecen en varias semanas.
Bibliografía

1. Askin DF. Complications in the transition from fetal to neonatal life. J Obstet Gynecol Neonatal
Nurs 2002 May-Jun;31(3):318-27.
2. Bondas-Salonen T. New mothers' experiences of postpartum care--a phenomenological follow-up
study.:J Clin Nurs. 1998 Mar;7(2):165-74.
3. Desmond M. Franklin R, Vallbona C. et al: The clinical behavior of the newly born. Infant. The term
baby. J Pediatr 1965; 62:307.
4. Desmond M , Rudilph A, Phitaksphraiwan P.: The transitional Care Nursery. 1966 Pediatr Clin North
Am 13:651

67.- Recién nacido de término que presenta reflejo de moro asimétrico a expensas de inmovilidad del brazo
derecho; el resto de los reflejos no presentan alteraciones. La madre refiere que durante el parto presentó
distocia de hombros. ¿Cuál es el diagnóstico?

a) Parálisis de Déjerine – Klumpke.


b) Parálisis de Erb – Duchenne.
c) Hemiparesia derecha.
d) Lesión de plexo braquial.

La parálisis de Erb – Duchenne se presenta en recién nacidos que nacen por parto con distocia de hombros,
provocando una hiperextensión lateral del cuello, lesionando las raíces de C5 y C6; por lo que se manifiesta
con adducción y rotación interna del brazo ipsilateral, el reflejo del Moro es asimétrico. En caso de
afectarse la raíz de C4 se agregará parálisis frénica. En cambio, la parálisis de Déjerine - klumpke se
presenta en las distocias de nalgas, con hiperextensión del brazo, se lesionan las raíces de C7 y C8,
manifestándose con mano caída y ausencia del reflejo de prensión palmar del lado afectado; puede
involucrar la raíz de T1 originando síndrome de Horner.

Bibliografía:
1. Allieu Y, Cenac, P. Neurotization via spinal accessory nerve in complete paralisis due to multiple avulsion
injuries of the brachial plexus. Clin Orthop 1988, 237: 67-74.
2. Bennet GC, Harrrold, AJ. Prognosis and early management of birth injuries to the brachial plexus. Br
Med J 1976, I: 1520-1521.
3. Bentolilla V, Nizard R, Bizot P, Sedel L. Complete traumatic brachial plexus palsy. J Bone Joint Surg
1999, 81A: 20-28.
4. Birch R, Bonney G, y Wynn Parry, CB. Surgical disorders of the peripheral nerves. Edinburgh: Churchill
Livingstone, 1998.
5. Birch, R. Infraclavicular lesions. En: Boome RS, ed. The brachial plexus. New York: Churchill Livingstone,
1997, 79-88.
6. Bonnard C, Slooff ACJ. Brachial plexus lesions. Drawings of explorations and reconstructions by
Algimantas Otonas Narakas. Berlín: Springer, 1999.

68.- Una herramienta importante para realizar el siguiente diagnóstico es la triada de Gregg. ¿A cual de las
siguientes infecciones nos referimos?

a) Citomegalovirus.
b) Toxoplasmosis.
c) Rubeola.
d) Sífilis congénita.

Greeg, en 1941, fue el primero que describió la tríada característica del SRC: cardiopatía congénita,
cataratas y sordera. Posteriormente estudios en animales corroboraron los mismos hallazgos encontrados en
los fetos humanos; defectos cardiacos, oculares, esqueléticos, SNC (caso índice) restricción del crecimiento
fetal y óbito, siendo mayor el riesgo de teratogénesis cuando la infección ocurre en el periodo de la
organogénesis

La tríada de Gregg que consiste en hipoacusia neurosensorial, malfiormaciones congénitas (PCA o estenosis
pulmonar) y anomalías oculares (catarata, glaucoma, retinitis en sal y pimienta) se presenta en la rubéola;
otras manifestaciones son: corioretinitis, púrpura trombocitopénica y microcefalia. En la infección por
citomegalovirus también se presenta microcefalia y coriorretinitis, así como calcificaciones
periventriculares.

BIBLIOGRAFÍA

– De Santis M. Cavalliere A, Straface G, Caruso A. Rubella infection in pregnancy. Reproductive


Toxicology 2006;21:390-8.
69.- Diagnosticó un absceso hepático amibiano en un paciente masculino de 24 años. Indicó tratamiento con
Metronidazol, con lo que se observó respuesta clínica favorable. Posteriormente complementa el
tratamiento con el siguiente fármaco:

a) Emetina.
b) Cloroquina.
c) Albendazol.
d) Iodoquinol (hidroxiquinoleína).
l

Posterior al tratamiento para la forma invasiva (trofozoito), para el cual el tratamiento más efectivo al
momento es el Metronidazol, pero sobre el que tienen también efectos la emetina, Cloroquina y tinidazol, se
recomienda utilizar un agente con actividad cisticida (actividad luminal), como el iodoquinol o la
Paromomicina.


Bibliografía:

– Kasper DL, Braunwald E, Fauci AS, Hauser SL, Longo DL, Jameson JL. Harrison´s Principles of
Internal Medicine. McGraw Hill. 16 Ed. 1214-1217 pp.

70.- Una de las siguientes puede ser la causa de un edema no inflamatorio:

a) Un aumento de la permeabilidad vascular.


b) Una eliminación excesiva de sal y agua por el riñón.
c) La disminución de la presión hidrostática intravascular.
d) Un aumento de la presión hidrostática intravascular.

Edema

Acumulo de un exceso de líquido en el espacio tisular intercelular (intersticial) o en las cavidades del
organismo. El edema puede ser:

Orígenes:

Hemodinámico(transudado).
Linfedema (linfa): trastorno (obstrucción) de tejido linfático.
El edema localizado: se puede producir en el caso de una obstrucción del flujo seroso
(produciéndose la inflamación o hinchazón de una pierna por ejemplo). También puede ser causado por un
proceso infeccioso (como los abscesos causados por Staphylococcus aureus).

Edema generalizado: es de carácter sistémico como en el caso de insuficiencia cardiaca o de


síndrome nefrótico (que se caracteriza por una intensa proteinuria secundaria a la permeabilidad glomerular
anómala). Cuando el edema es intenso y generalizado de tal forma que provoca hinchazón difusa de todos los
tejidos y órganos de la economía, especialmente a nivel del tejido subcutáneo, se denomina Anasarca.

Exudado: líquido inflamatorio extravascular rico en proteínas, detritus celulares y leucocitos. Se


trata de un acumulo de líquido que se encuentran rico en restos celulares, leucocitos, restos proteínicos y
por lo general presenta una r>1.020. Se debe a un incremento en la permeabilidad endotelial con la salida de
proteínas plasmáticas (principalmente albúmina). Se presenta por infecciones piógenes, TB, etc. (de origen
inflamatorio).

Trasudado: en el líquido de edema de origen no inflamatorio y está relacionado con los casos de
insuficiencia cardiaca y nefropatías por lo general no hay (es pobre) proteínas, ni leucocitos, con una rÍ1.012.
Únicamente es el acumulo de líquidos.

El trastorno hemodinámico (trasudado) se presenta cuando se altera la Ley de Starling que es el


equilibrio normal de los líquidos (intravascular y extravascular). Se mantiene por la acción de dos grupos de
fuerzas opuestas, los que hacen que el líquido tienda a salir de la circulación son la presión osmótica del
líquido intersticial y la presión hidrostática intravascular; los que hacen que el líquido pase a la circulación
son la presión osmótica de las proteínas plasmáticas (principalmente albúmina) (presión coloidosmótica) y la
presión hidrostática tisular. Por lo tanto el equilibrio entre estas fuerzas es tal que en los capilares
musculares periféricos existe un movimiento neto de líquidos hacia fuera, pero éste líquido es drenado a los
linfáticos, por lo que no se produce edema.

Los factores que aumentan la presión hidrostática intravascular o disminuye la presión


coloidosmótica intravascular dan lugar al aumento de la salida del líquido desde los capilares, con el
consiguiente trastorno hemodinámico el edema.

Nota: en la lesión (por falta de la Angiotensina, que regula las sales) por pérdida de proteínas
(principalmente albumina) (o sea la proteinuria) se produce el Edema generalizado (Anasarca).
Fisiopatología del edema, Bibliografía:

• Autores: José María Sillero Fernández de Cañete


• Localización: Seminario médico, ISSN 0488-2571, Vol. 49, Nº. 1, 1997 , pags. 74-86.
• Organización Panamericana de la Salud ,Programa de Publicaciones (DBI/E)
525 Twenty-third Street, NW.

71.- En uno de los siguientes diagnósticos es característica la aparición de cilindros hemáticos en el


sedimento urinario:

a) Daño tubular.
b) Cualquier lesión de la nefrona.
c) Lesión a cualquier nivel de las vías urinarias.
d) Daño glomerular severo.
La presencia de cilindros hemáticos o sus derivados indica hemorragia dentro de la neurona que puede ser el
resultado de una lesión glomerular como la que se ve en la nefritis hemorrágica aguda, o una enfermedad
manifestada por necrosis vascular o del penacho vascular como ocurre en la Periarteritis nodosa púrpura de
Henoch-Schonlein, endocarditis bacteriana subaguda. También la necrosis tubular renal acompañada de
inflamación intersticial puede producir hematuria y formación de cilindros hemáticos.
Asimismo, la nefrosis hemoglobinúrica resultante de la administración de sangre incompatible y la hemólisis
intravascular intensa pueden producir hallazgos similares

Bibliografía:

– Bauer. J. D.; Toro. G... v Ackermann. P. G. (eds.): Brav's Clinical Laboratorv Methods.
ed. 6, 'St. ouisC,. V: M O S ~C~o.
Bibliografía nefrológica:

– FARRERAS-ROZMAN. Medicina Interna; Harcourt Brace España S.A.


– GUARDIA J, GRAU JM, NET A. Medicina Interna Fundamental; Springer Verlag Iberica S.A.

– ARTHUR GREENBERG. Tratado de enfermedades renales. Harcourt Brace.

72.- Un hombre de 47 años de edad afebril con antecedente de pancreatitis crónica, presenta una masa
abdominal palpable y amilasa sérica persistentemente elevada. ¿La mayor probabilidad de diagnóstico es?

a) Cistadenoma pancreático.
b) Pseudoquiste pancreático.
c) Carcinoma pancreático.
d) Colección pancreática aguda

El pseudoquiste pancreático es una colección de jugo pancreático localizada, por lo general, en el


interior o alrededor del parénquima pancreático. El pseudoquiste pancreático está confinado por una
capa no epitelializada de tejido necrótico, fibrótico y de granulación, que se desarrolla tras una lesión
pancreática. Para su formación requiere un mínimo de cuatro semanas desde que ese daño se produce.
El pseudoquiste pancreático es una complicación tanto de la pancreatitis aguda como de la crónica. Si
bien la mayoría de los pseudoquistes pancreáticos se localizan en la cabeza y el cuerpo del páncreas,
hasta un 20% de los mismos son extrapancreáticos (1). Se han descrito pseudoquistes pancreáticos en
múltiples localizaciones, como cavidad pleural, mediastino y pelvis (2). Se presenta el caso de un
pseudoquiste pancreático de localización hepática que apareció en el curso de una agudización de una
pancreatitis crónica, y que se resolvió sin necesidad de drenaje.

Diagnóstico:

Manifestaciones Clínicas:

1. Pacientes en la cuarta o quinta década de vida, antecedentes etiológicos.


2. Pacientes con pancreatitis aguda que no resuelve luego de 5 a 7 días de tratamiento o luego
de mejoría recae.
3. Sensación de cuerpo extraño y pesadez en la mitad superior del abdomen.
4. Si pancreatitis crónica, dolor abdominal o síntomas por compresión de víscera.
5. Nauseas, vómitos y pérdida de peso por obstrucción duodenal.
6. Ictero, si compresión del colédoco.
7. Masa en abdomen superior, lisa y dura, muchas veces insensible.
8. Más raramente ascitis y derrame pleural.

Complementarios Diagnósticos:
1. Ultrasonografía Abdominal: Muchas veces diagnóstico, preferido para vigilancia.
2. TAC: Ideal para diagnóstico.

Bibliografía

1. Hamm VB, Franzen N. Atypically located pancreatic pseudocyst in liver, spleen, stomach wall
and mediastinum: their CT diagnosis. Rofo 1993; 159 (6): 522-7.

2. Vitas GJ, Sarr MG. Selected management of pancreatic pseudocyst: Operative versus
expectant management. Surgery 1992; 111 (2): 123-30.

3. Mofredj A, Cadranel JF, Dautreaux M, Kazerouni F, Hadj-Nacer K, Deplaix P, et al. Pancreatic


pseudocyst located in the liver: a case report and literature review. J Clin Gastroenterol 2000;
30 (1): 81-3.

4. Balzan S, Kianmanesh R, Farges O, Sauvanet A, O'toole D, Levy P, et al. Right intrahepatic


pseudocyst following acute pancreatitis: an unusual location after acute pancreatitis. J
Hepatobiliary Pancreat Surg 2005; 12 (2): 135-7.

73.- Masculino de de 37 años refiere pérdida de fuerza progresiva en miembros inferiores, de unos 7 días
de evolución, dolores musculares y parestesias en pies y manos. En su exploración se aprecia únicamente
debilidad en los cuatro miembros, de predominio distal y en miembros inferiores, y arreflexia generalizada.
El diagnostico más probable será:

a) Una miastenia gravis.


b) Un proceso expansivo medular cervical.
c) Una polimiositis.
d) Una polirradiculoneuritis aguda.

SINDROME DE GUILLAIN-BARRÉ
El síndrome de Guillain-Barré fue descripto en el año 1834 por Ollivier y Wardrop, Landry, en 1859 hablo de
una paralisis ascendente que seguía con insuficiencia respiratoria y muerte. Osler, en 1892 hablo de una
polineuritis febril y Guillain-Barré y Strohl, en 1916 reconocieron una polineuritis benigna con disociación
albuminocitológica.
Tambien llamada polirradiculoneuritis aguda, caracterizada por alteraciones sensitivas, motoras, arreflexia
y disociación albúmino citológica en el LCR (líquido cefaloraquideo).
Es una enfermedad que puede aparecer a cualquier edad afectando por igual a ambos sexos.
La característica neuropatológica corresponde a focos de desmielinización focal y segmentaria de los
nervios periféricos en toda su longitud, comprometiendo incluso las raices. Los nervios del sistema nervioso
periférico presenta infiltrados inflamatorios perivasculares, y es ahí donde se observan las lesiones
mielínicas.
Continúa en boga la teoría inmunológica como la responsable de estas polirradiculoneuritis inflamatoria.
Muchas veces la enfermedad surge luego de un proceso de vacunación, aplicación de suero o de una infección
vírica (generalmente trancurre de 7 a 25 días entre el proceso que se cree causal y el comienzo de los
síntomas), Los organismos que se han visto con más frecuencia relacionados con este transtorno fueron el
virus de Epstein-Barr, el citomegalovirus, el micoplasma y el virus de la hepatitis B.
La enfermedad se presenta en su evolución en 3 (tres) fases:

1-FASE DE EXTENSIÓN.

2- FASE DE ESTADO.

3- FASE DE RECUPERACIÓN.

1- FASE DE EXTENSIÓN
El paciente comienza a sentir sensación de "hormigueo, las piernas se me duermen", "siento calambres en
ambas piernas"... las parestesias en las extremidades y el deficit motor son comunes, pero más frecuente es
que aparezca el compromiso motor. Se afectan los miembros inferiores precozmente y la extensión suele
ser ascendente, de forma progresiva, en un tiempo estimado de 1 semana, 1 semana y media. Hay que tener
mucho cuidado con estos pacientes por la posible aparición del compromiso de los músculos respiratorios o
algun trastorno en la deglución (En estos casos son pacientes pasibles de internación y de cuidados
intensivos).

2- FASE DE ESTADO
Es donde aparecen la mayoría de los signos neurológicos. El compromiso motor puede llegar al extremo,
puede aparecer una tetraplejía con compromiso respiratorio y deglutorio. Otros casos pueden limitarse al
compromiso de los miembros inferiores. Aparece arreflexia generalizada. Suceden parestesias, se
compromete la sensibilidad tactil y profunda (por compromiso de los cordones posteriores, vía de Goll y
Burdach). Toma nervios craneales, aparece una parálisis facial que puede ser uni o bilateral, puede verse
parestesia trigeminal. Cuando toma los pares craneales IX-X (glosofaringeo y neumogástrico) aparecen los
trastornos de la deglución.
Cuando toma el sistema nervioso vegetativo se ve que el paciente cursa con hipertensión, trastornos en la
repolarización cardíaca (visibles en el ECG), alteraciones en la glucemia, hiponatremia, SIHAD (secreción
inapropiada de hormona antidiurética) o bradicardias (que puede llevar hasta un paro cardíaco)
Un tema muy importante es el estudio del líquido cefalorraquídeo (LCR) que muestra una elevación de la
proteinorraquia que no se acompaña de hipercitosis (aumento de las células en el LCR), esto es lo que se
denomina disociación albuminocitológica. En el 80% de los casos la elevación corresponde a las
gammaglobulinas. Esta disociación no es inmediata, por lo que puede tardar un tiempo en aparecer. (esto
tiene que ver con que las punciones del LCR en un período temprano pueden dar normales).

3- FASE DE RECUPERACIÓN
Los signos neurológicos regresan en orden inverso al que aparecieron luego de 1 a 2 semanas de estabilidad
en los síntomas. Estos cambios lo hacen en forma más lenta en relación a como aparecieron. El tiempo de
regresión puede durar desde semanas hasta meses.
La recuperación puede ser total o dejar secuelas motoras o sensitivas hasta en un 20% de los casos.
DIAGNOSTICO DIFERENCIAL
Esta enfermedad es una de las polineuropatias más frecuentes, la que más rápido evoluciona y que tiene un
alto potencial mortal.
Cualquier polineuropatía, de evolución ascendente, que ponga en peligro de vida al paciente por compromiso
respiratorio en el plazo de unos pocos días, nos deberá hacer sospechar de un sindrome de Guillain-Barré.

Bibliografía:

– Pearce, j.m.s. Barré-Liéou “syndrome”. J Neurol Neurosurg Psychiatry 2004: 75, 319.
-Pritchard, J.; Hughes, R.A.C. Guillain-Barré syndrome. The Lancet 2004; 363: 2186-88.
-Thiébaut, F.J.A. Barré (1880-1967). J Neurol Sci 1968; 6: 381-2.

74.- ¿Qué tipo de diseño de investigación son los estudios de casos y controles?

a) Observacional analítico.
b) Observacional descriptivos.
c) Experimental descriptivo.
d) Ensayo clínico.

Los estudios descriptivos se relacionan con la distribución de las enfermedades, incluyendo la consideración
de qué población o subgrupos desarrollan o no enfermedad, en que localización geográfica es más o menos
común, y cómo la frecuencia de ocurrencia varía en el tiempo. Esto incluso puede llegar a dirigir a la
formulación de una hipótesis epidemiológica que sea consistente con el conocimiento existente de la
ocurrencia de enfermedad.
Los estudios analíticos que se focalizan en los determinantes de una enfermedad prueban las hipótesis
formuladas desde los estudios descriptivos, con el objetivo final de juzgar si una exposición particular se
asocia, causa o previene enfermedad, ampliándose también a aquellos estudios que evalúan los factores que
se relacionan con distintos resultados de una enfermedad (ej: mejoría o muerte).

CARACTERÍSTICAS DE UN ESTUDIO ANALÍTICO:


• Existe un grupo control.
• Comparabilidad en los grupos.
• Control de factores diferentes al de interés.
• Medición de efecto del azar (pruebas de significación).

Dentro de los estudios analíticos están:


• Estudios de Casos y Controles
• Estudios de Cohorte
• Ensayos Clínicos

Bibliografía:

– Ruiz A, Morillo L. “Epidemiología Clínica. Investigación clínica aplicada”. Bogotá, Editorial Médica
Panamericana, 2004.
– Fletcher R, Fletcher S, Wagner E. “Epidemiología Clínica: aspectos fundamentales”
MASSONWilliams & Wilkins España, S.A. 2º edición en español, 1998.
– Hulley SB, Feigal D, Martin M, “Diseño de Investigación clínica”. Ediciones Doyma, 1997.

75.- Debemos sospechar un retinoblastoma en un niño que presenta los siguientes síntomas:

a) Dolor, fotofobia y lagrimeo.


b) Lagrimeo, fotofobia y aumento del diámetro corneal.
c) Fotofobia y quemosis conjuntival.
d) Estrabismo y leucoria.

RETINOBLASTOMA

TUMOR OCULAR MÁS FRECUENTE EN INFANCIA.


1/20.000 RN.
80% en < 3 años.
Uni o bilaterales.

Herencia:
1. AD alta penetrancia (90-95%)
2. Esporádicos
Diagnóstico:
•Leucocoria.
•Estrabismo.
•Mala AV.
•Ojo rojo y doloroso.
•Celulitis orbitaria.
•Examen de rutina.

Bibliografía:

– Annals d.Oftalmologia 2001;9(2):74-92 N. Martín, MD. Coll, J. García, J. Sánchez de Toledo, E.


Triviño, M. Guitart, JJ. Gil.

– Unidad Oftalmología Pediátrica. Hospital. Maternoinfantil Vall d.Hebron.


– Departamento de biología-celular, fisiología e inmunología de la Universidad Autónoma de Barcelona.
– Servicio oftalmología Hospital General Vall d.Hebron.
76.- Para un paciente de 16 años con diagnóstico de torsión testicular de 8 horas de evolución, el
tratamiento más adecuado consiste en practicar:

a) Orquiectomia.
b) Distorsión manual.
c) Orquidopexia.
d) Distorsión quirúrgica.

La torsión testicular es una etiología frecuente de escroto agudo en la infancia, con una incidencia
aproximada de 1 por cada 4000 varones menores de 25 años.En el adulto la frecuencia de esta nosología
es menor, pero no inexistente, variando según las series entre el 8% y el 28%. La incidencia disminuye con
la edad. Refiriéndose en diversos estudios incidencias del 26 al 39% en mayores de 21 años que disminuye
hasta el 10% en mayores de 30 años.

Debido a la rareza de esta patología en adultos, y la mayor prevalencia de procesos infecciosos e


inflamatorios en varones de más de 30 años, como causa de escroto agudo, muchos de estos casos no son
diagnosticados o incluso son etiquetados erróneamente y tratados como procesos infecciosos.

El diagnóstico tardío, la creencia errónea de lo inusual de la torsión en el adulto, hace que la tasa de
orquiectomía en el adulto sea de un 75% frente a un 20 a 50% en niños.

Con el diagnóstico de presunción de torsión testicular, la prueba diagnóstica de elección es la


ultrasonografía con doppler, que alcanza una certeza diagnóstica de más del 90%. La conjunción del
doppler con la gammagrafia con tecnecio 99 marcado (Tc99m), se correlaciona en un 100% de los casos con
los hallazgos quirúrgicos encontrados20. En muchos centros la ausencia de disponibilidad de estos medios
diagnósticos, bien por falta de medios o de personal cualificado durante 24 horas; debe hacernos plantar
la exploración quirúrgica ante una intensa sospecha.

No sólo el tiempo de evolución es importante para la viabilidad del parénquima testicular, también influye
el grado de torsión del cordón. Así se han documentado cambios histológicos del parénquima testicular, a
las seis horas del inicio del dolor21. Sonda y Lapides, en estudios experimentales con perros, en los que se
le realizo una torsión de 1080º la viabilidad del testículo quedo reducida a menos de dos horas.
Resumen de torsión testicular:

Es un cuadro violento, grave y que requiere una inmediata solución.

Se presenta a cualquier edad, pero es mas frecuente al comienzo de la pubertad y en el periodo alrededor
del parto.

Se caracteriza por un cuadro de dolor intenso, de aparición brusca, que puede acompañarse con dolor
abdominal, nauseas y vómitos.

¿Qué se encuentra al examen?


Dolor testicular intenso, aumento de volumen, color rojo, disminución o ausencia del reflejo Cremasteriano y
posición alta y trasversal del testículo.

¿Qué examen puede ayudar al diagnostico?

La cintigrafia testicular y el eco Doppler de color, son los exámenes de mayor utilidad en el diagnostico, ya
que demuestran un marcado descenso o ausencia de flujo sanguíneo.

¿Cirugía de urgencia?
La torsión del conducto espermático es una urgencia quirúrgica, ya que en un lapso de seis a ocho horas la
recuperación del testículo es casi imposible.
Si la historia y el examen físico son altamente sugerentes de torsión no es necesario realizar exámenes
complementarios, solo deben usarse en caso de duda y cuando su disponibilidad sea inmediata.
¿Objetivo de la cirugía?

Foto Adam

El prepósito es destorcer el testículo afectado y realizar una fijación para evitar nuevas torsiones, si se
duda de la viabilidad se espera algunos minutos o sino se extrae finalmente para evitar mas complicaciones.

En el periodo perinatal sin embargo, tanto antes como después del nacimiento se puede producir una torsión
por ausencia de fijación al escroto. Se presenta como una masa tumoral dura e indolora acompañada de
aumento de hinchazón y color roja que puede confundir con un tumor o hernia atascada. Si la torsión se ha
producido antes del nacimiento no constituye una emergencia y la intervención puede efectuarse cuando
exista un menor riesgo anestésico.
Bibliografía:

1. WILLIAMSON RC.: Torsion of the testis and allied conditions. Brit J Surg 1976; 63: 465.

2. CHAPMAN RN, WALTON AJ.: Torsion of the testis and its appendages. Brit Med J 1972; 1:
164.

3. LEE LM, WRIGHT JE.: Testicular torsion in the adult. J Urol 1983; 130: 93.

4. WHITERINGTON R, JARREL TS.: Torsion of the sprematic cords in adult. J Urol 1990; 143:
62.

5. PERRY S, HOOPINGAM D.: Testicular torsion in the older patient. Ann Emerg Med 1983; 12:
319.

77.- Masculino de 50 con distensión abdominal en incremento e ictericia de 6 semanas de evolución. La E.F.
revela arañas vasculares, venas ingurgitadas alrededor del ombligo y ascitis. ¿Cuál es la causa más probable
de esta ictericia?

a) Hepatitis.
b) Colecistitis aguda.
c) Coledocolitiasis.
d) Cirrosis.

La cirrosis hepática es la cirrosis que afecta al tejido hepático como consecuencia final de diferentes
enfermedad crónicas.

Las consecuencias de la cirrosis hepática sobre la salud del individuo dependen fundamentalmente del grado
de funcionalidad que el hígado pueda conservar a pesar de la alteración histológica.

En muchos casos, el diagnóstico de la cirrosis es casual, puesto que como se ha dicho, en la fase compensada
de la enfermedad sus manifestaciones pueden ser poco aparentes, presentando síntomas vagos o
inespecíficos como dispepsia, astenia o hiperpirexia. Así, puede detectarse ante la existencia de
hepatomegalia en una exploración física de rutina, ante alteraciones en las pruebas de función hepática, o
ante la positividad en las pruebas de estudio de las hepatitis virales.
Entre las manifestaciones que es posible encontrar se encuentran algunos signos cutáneos. Ninguno es
patognomónico, pero pueden resultar útiles para la sospecha diagnóstica; entre estos "Estigmas de
hepatopatía" cabe destacar las arañas vasculares o spiders, distribuidas en el territorio de la vena cava
superior, En ocasiones, está presente también un enrojecimiento de las eminencias tenar e hipotenar que se
conoce como eritema palmar. En la cirrosis de origen alcohólico, puede aparecer hipertrofia parotídea y
contractura de Dupuytren.

En ocasiones aparecen xantelasmas en los párpados, sobre todo en las enfermedades colestáticas (colangitis
esclerosante primaria y cirrosis biliar primaria) mientras que en la enfermedad de Wilson aparece el anillo
de Kayser-Fleischer (anillo de coloración pardo-verdosa en el borde límbico de la córnea, por depósito de
cobre en la membrana de Descemet).

Al efectuar la exploración física abdominal suele observarse el hígado aumentado de tamaño con superficie
irregular y consistencia dura, si bien en los estadios finales de la enfermedad puede encontrarse totalmente
atrófico y retraído no siendo accesible a la palpación.

Dado que la hepatomegalia es en general indolora, la existencia de dolor abdominal debe hacer sospechar
algún fenómeno intercurrente como una pancreatitis o un cólico biliar, dada la elevada incidencia de la
litiasis biliar en el paciente cirrótico.

La esplenomegalia junto con la presencia de circulación colateral (múltiples venas dilatadas subcutáneas en la
pared abdominal), indican la existencia de hipertensión portal. Cuando la circulación colateral es prominente
alrededor de la vena umbilical en la zona del ombligo se denomina clásicamente como "cabeza de Medusa".

La hipertensión portal puede también condicionar la presencia de ascitis, que puede manifestarse como un
aumento del perímetro abdominal, indicando la presencia de líquido libre intra-abdominal. Las hernias de la
pared abdominal, sobre todo umbilicales son frecuentes cuando hay ascitis, así como el edema subcutáneo
que aparece en las zonas declives (las piernas generalmente)

Las alteraciones endocrinas son comunes en la cirrosis, sobre todo en las de etiología alcohólica; los varones
pueden presentar atrofia testicular, disminución de la libido y disfunción eréctil. La ginecomastia es
frecuente. Las mujeres suelen presentar alteraciones menstruales e incluso amenorrea.

La ictericia, es un signo que acompaña con cierta frecuencia a la cirrosis descompensada y generalmente es
un dato de enfermedad avanzada.
Cirrosis de hígado visto axialmente con tomografía computada de abdomen

Bibliografía:

– Detection of novel biomarkers of liver cirrhosis by proteomic analysis. Mölleken C, Sitek B, Henkel
C, Poschmann G, Sipos B, Wiese S, Warscheid B, Broelsch C, Reiser M,Friedman SL, Tornøe I,
Schlosser A, Klöppel G, Schmiegel W, Meyer HE, Holmskov U, Stühler K. Hepatology 2009;49:1257-
66.
– Computer-assisted image analysis of liver collagen: relationship to Ishak scoring and hepatic venous
pressure gradient. Calvaruso V, Burroughs AK, Standish R, Manousou P, Grillo F, Leandro G, Maimone
S, Pleguezuelo M, Xirouchakis I,

78.- El método más frecuentemente usado para el tratamiento conservador de la Displasia del Desarrollo
de la cadera es:

a) Uso de triple pañal.


b) Uso de cojín de Frejka.
c) Uso de arnés de Pavlik.
d) Uso de cojín de Creig.

ARNES DE PAVLIK

La displasia en el desarrollo de la cadera es la principal indicación para el uso del arnés de pavlik, teniendo
como función el mantener la cadera centrada y de esta forma se permita un mejor desarrollo de la misma.
Bibliografía:

1- Dezateux C. Rosendahl K.Developmental dysplasia of the hip. Lancet 2007; 369: 1541–52.
2- Lee M., Craig P. Growth and Development of the Child’s Hip. Orthop Clin N Am 37 (2006) 119 –132.
3- U.S. Preventive services task force. Screening for Developmental Dysplasia of the Hip:
Recommendation Statement. American Family Physician Volume 73, Number 11, Junio 1, 2006.
4- Langman S. Embriologia Medica con orientacion clinica. Decima edicion. Panamericana.
5- Rosselli P., Duplat J., Uribe i., Turriago C. Ortopedia Infantil. Editorial Panamericana.
6- Gelfer P, Kennedy K. Developmental Dysplasia of the Hip. J Pediatr Health Care. (2008). 22, 318-322.
7- Rachio KH. Simultaneuos open reduction and salter innominate osteotomya for developmentaldysplasia of
the hip. J Bone Joint Surg 2004;78B47:1-6.

79.- Masculino de 67 que se presenta porque desde hace 6 semanas presenta disminución en el calibre de las
heces, hematoquezia y constipación. Se realiza colon por enema encontrando una zona de estenosis en el
sigmoides, con imagen en manzana mordida. Estos datos le hacen sospechar fuertemente en:

a) Cáncer de colon.
b) Enfermedad diverticular.
c) Isquemia.
d) Enfermedad de Crohn.
Cuando sospechamos una patología de intestino grueso, el estudio principal es la colonoscopia, única
exploración capaz de confirmar macroscópica e histológicamente una lesión, además de que permite realizar
en muchos casos algunas opciones terapéuticas, y de que solo con esta prueba se puede detectar la relación
adenoma-cáncer y ese 5-10 % de casos en que hay tumores sincrónicos en el resto del colon.

Morfología endoscópica

Su clasificación es muy diversa, según las características macroscópicas e histológicas. La organización


Mundial de Endoscopia Digestiva (OMED)11 distingue 4 tipos endoscópicos de cáncer rectocolónico:

Tipo 1. Prominente polipoide y velloso.


Tipo 2. Ulcerado.
Tipo 3. Infiltrante.
Tipo 4. Avanzado inclasificable.

Tipo 1. El subtipo polipoide es un tumor exofíntico más frecuente en colon, que suele cursar con anemia
ferropénica por la pérdida hemática debida a las erosiones superficiales de la masa tumoral. Los cánceres
vellosos son una protrusión exofíntica a veces de tamaño considerable, con superficie en coliflor y tendencia
a la ulceración, con predilección por el recto.

Tipo 2. El tipo ulcerado es la forma más frecuente de cáncer rectocólico, con bordes prominentes,
irregulares y duros al tacto con la pinza de biopsia. Esta ulceración puede crecer y abarcar toda la
circunferencia intestinal con un claro compromiso de la luz colónica, cuya imagen radiológica característica
es la "estenosis en servilletero" o "manzana mordida". Este tipo es más frecuente en el colon izquierdo,
donde suele provocar síntomas de obstrucción intestinal.

Tipo 3. La forma infiltrante es menos frecuente y se localiza generalmente a nivel del recto o de la unión
rectosigmoidea. Se caracteriza por un engrosamiento difuso de la mucosa que impresiona una estenosis, en
ocasiones la mucosa está conservada o poco afectada. La biopsia es decisiva para esclarecer el diagnóstico y
poder diferenciarlo de tumoraciones benignas.

Tipo 4. El tipo avanzado inclasificable generalmente presenta una morfología mixta, en la que se
entremezclan características de todos los tipos anteriores.

Bibliografía:

1. Chávez FJ, Romero T, González S, Lence J, Santos T. Riesgo de morir por cáncer en Cuba. Rev Cubana
Oncol 1997;13(1):5-11.
2. Soriano J, Galán Y, Loaces P, Martín A, Arebóla JA, Carrillo G. Incidencia en Cuba del cáncer en la
tercera edad. Rev Cubana Oncol 1998;14(2):121-8.
3. Suárez López F, Santo Ruzo J, Vázquez Iglesias JL. Tumores malignos. En: Vázquez Iglesias JL.
Colonoscopia: diagnóstico y terapéutica. La Coruña: Hospital Juan Canalejo 1998:233-54.
4. Bond JH. Colon surveillance for neoplasia. Gastrointés Endoscopy 1999;49(3):35-9.
5. Romero T, Camacho R, Fernández L, Grau J, Chacón M. Actualidad y proyecciones del departamento del
programa nacional de control de cáncer. Rev Cubana Oncol 1996; 12(2):126-30.
80.- El germen causal más frecuente de artritis séptica en menores de 4 años de edad es:

a) Estreptococo B.
b) Estafilococo Aureus.
c) Gonococo Gonorrea.
d) Hemofilus Influenza.

56

Artritis Séptica
Diagnóstico:
Organismo
b. 1 mes a los 3 años de edad.
I. Estafilococo áureas.
II. Hemofilus influenza tipo :
º 20-30% meningitis asociada.
º no factor si PTE. Inmunizado.
III. Kingella Kingae.
º Niños sanos< 4 años.
º Asociados a infección VU.
IV. ANTIBIOTICOS.
º Cefotaxina o Ceftriaxona.
º Penicilina en KK.

Bibliografía:

1.-Rodd S, Harris E, Sledge C. Kelley’s Textbook of rheumatology. 6ª Edición. Saunders, Unites States
of America, 2001.
2. Klippel JH, Dieppe PA. Rheumatology. 2ª. Edición . Mosby, España, 1997.
3. Martínez-Elizondo D. Introducción a la reumatología. 2ª. Edición. Sociedad Mexicana de Reumatología,
México, 1997.
1) Klippel JH, editor. Primer on the rheumatic diseases. 2nd ed. Atlanta: The Arthritis Foundation;
2001.
2) Ruddy S, Harris ED Jr, Sledge CB, editors. Kelley´s Textbook of Rheumatology. 6th ed.
Philadelphia: Saunders; 2001.
3) Koopman WJ, editor. Arthritis and allied conditions. 14th ed. Philadelphia: Lippincott Williams &
Wilkins; 2001.
4) Klippel JH, Dieppe PA, editors. Rheumatology. 2nd ed. London: Mosby; 1997.

81.- Se trata de paciente masculino de 72 años que cursa con evento vascular cerebral isquémico ¿El
tratamiento de elección en este padecimiento es?

a) Antiagregantes, hemorreologicos, anticonvulsivantes.


b) Vasodilatadores, anticonvulsivantes, derivados sanguíneos.
c) Medidas generales, hipoglucemiantes, analgésicos.
d) Vasodilatadores, analgésicos, sedación.

Tratamiento
Fase aguda ataque isquémico menor:
* Estabilización general.
- 02 por puntas nasales.
- Control de la TA y alteraciones cardiacas (FA).
- Correción de variantes metabólicos (ES, glucosa, etc).
* Con o sin crisis convulsivas:
- DFH 750 mg. IV impregnación con 125 mg. IV c/8 hr.
- Carbamacepina 200 mg. VO cada 8 hrs.
* Mejorar estado circulatorio:
- Pentoxifilina 400 VO c/ 8 hrs.
- ASA 125 mg VO c/ 24 hrs.

Atacar causa subyacente:


- Control adecuado de la TA y la glicemia.
- Control de hiperlipidemia e hipercolesterolemia.
- Rehabilitación temprana.
- Eliminar tabaquismo.
* Vigilancia tomográfica de la evolución.
* Limitar el daño y prevenir su extensión.
* Educación al paciente y a la familia.
Bibliografía:

1.Aronson A y cols. Examen clínico neurológico, 3ª Edición. La Prensa Médica Mexicana,


México, 1995.

2. Uribe CS, Arana A, Pombo PL. Neurología, 5ª Edición. Corporación para investigaciones biológicas.
Colombia, 1996.

3.-Adams R, Víctor M. Principles of Neurology. Mc Graw Hill. 7th ed. USA 2001.

4.-Bradley W.G. Neurology in clinical practice. Butterworth Heinemann. 4th ed.

82.- En una paciente con amenorrea secundaria que presenta menstruación posterior a la
administración de progestágenos, ¿Lo más probable es que tenga?

a) Estrógenos bajos.
b) Estrógenos normales.
c) Progesterona elevada.
d) Gonadotropinas altas.

La presencia de estrógenos se puede establecer de dos maneras: con la prueba de desafío con acetato de
medroxiprogesterona (AMP) y el índice de maduración vaginal. Si hay estrógenos, luego de la administración
de 10 mg de AMP diarios por 10 días debería presentarse un sangrado por vagina. Y de esta forma
corroborar los niveles de estrógenos serian normales. La ausencia de esta metrorragia sugiere una
obstrucción o un hipogonadismo. La otra alternativa es realizar el índice de madurez vaginal que es un
procedimiento sencillo que puede obtenerse en el consultorio. Usando una espátula como las de Papanicolaou
el ginecólogo extrae una muestra de la vagina y la extiende en un portaobjeto. Este portaobjeto se procesa
y se estudia en el microscopio. Un extendido maduro tendrá células epiteliales superficiales grandes y
numerosas con un núcleo rodeado por una gran cantidad de citoplasma. Por el contrario, una muestra no
estrogénica tendrá un mayor número de células parabasales y basales con núcleo grande rodeado por escaso
citoplasma.

Bibliografía:

– Speroff L, Glass RH, Kase NG, eds. Clinical Gynecologic Endocrinology and Infertility, 5th ed.
– Baltimore: Williams & Wilkins; 1994:334-335.
83.- La neoplasia benigna mas frecuente encontrada en la glándula mamaria es:

a) Galactocele.
b) Necrosis grasa.
c) Fibroadenoma.
d) Papiloma intraductal.

Entre los tumores benignos de la mama se encuentra el fibroadenoma, así como la enfermedad fibroquística,
papilomas intraductales y la mastitis. Por lo general los tumores se presentan en ambas mamas.
El fibroadenoma es un tumor sólido, se mueve al tacto, no duele o puede molestar ligeramente antes de la
menstruación y está bien delimitado. Es un tumor muy común en mujeres jóvenes.
Su tratamiento puede ser a base de medicamentos o procedimientos quirúrgicos dependiendo de cada caso.

Bibliografía:

. Basson R. Women’s sexual dysfunction: revised and expanded definitions. CMAJ


2005;172:1327-1333.

84.- Masculino de 67 años con cáncer de próstata, que presenta hemorragia gingival, hematomas, no hay
pulsos distales, acrocianosis y datos de isquemia miocárdica, por lo que usted sospecha:

a) Púrpura No Trombocitopénica.
b) Coagulación Intravascular Diseminada.
c) Púrpura Trombocitopénica.
d) Hemofilia.

La coagulación intravascular diseminada consiste en una activación excesiva de la coagulación, lo que


ocasiona trombosis. Como consecuencia el consumo progresivo de los factores de coagulación y plaquetas en
las fases finales, produce el fenómeno opuesto, con hemorragias generalizadas. Las causas son: infecciones
sobre todo sepsis por gram negativos, problemas obstétricos (abruptio, retención de feto muerto,
embolismo de líquido amniótico, aborto séptico, toxemia) neoplasias (leucemias promielocíticas, cáncer de
páncreas, próstata, pulmón y esófago), fenómenos autoinmunes y traumas masivos. En la CID aguda debiada
a complicaciones obstétricas o traumatismo predominan la diátesis hemorrágica, mientras que en la CID
crónica se manifiesta inicialmente por complicaciones trombóticas.
Bibliografía:

1. Cotran R, Kumar V. Robbins Patología estructural y funcional. 6° edición. Mc Graw Hill. Colombia
2003673.

85.- Acude al servicio de urgencias paciente masculino de 33 años que fue embestido por una motocicleta,
presenta herida cortante, sangrante, la cual no ha sido controlada, inicia con datos de hipovolémia, el común
denominador en el tratamiento del estado de choque es la administración de:

a) Líquidos (cristaloides).
b) Productos hemáticos.
c) Vasopresores.
d) Inotrópicos.

CANTIDAD DE LIQUIDOS Y FORMA DE ADMINISTRARLOS

1. Acceso vascular

A todo paciente en choque se le deben canalizar por lo menos dos venas con agujas de grueso calibre. En
orden de preferencia se deben tomar primero las de los antebrazos; si fracasa el intento de punción se pasa
a la disección de la safena interna o de una vena del miembro superior. El catéter subclavio o yugular se
utilizan para monitorizar y muy raramente como acceso primario en caso de emergencia por hipovolemia, por
lo que su uso se debe limitar a los pacientes en quienes se vaya a realizar una monitoría hemodinámica. En los
pacientes estables, la disección de la vena safena se debe evitar debido a la alta incidencia de flebitis y
tromboflebitis séptica. En estos casos se prefiere la punción subclavia o yugular interna o externa (3, 8). En
niños menores de 5 años cuando se dificulta obtener un acceso vascular, se puede utilizar la vía intraósea. La
velocidad del flujo es limitada, pero permite aplicar soluciones mientras se obtiene una vía más expedita. El
sitio preferido de punción es la tibia, 1 cm distal y medial a la tuberosidad (8). En muchos centros de trauma
se utiliza la vena femoral, la cual se canaliza con un catéter 7 a 8.5 French. El uso de este tipo de catéteres
y la disponibilidad de infusores, permite introducir al espacio intravascular fácilmente 1.000 mL de
cristaloides por minuto.

Como es obvio. Se debe evitar una vena cuyo flujo sea tributario de un área traumatizada o con sospecha de
trauma. Por ejemplo, si el paciente tiene una herida por bala subclavia o axilar derecha, está contraindicado
suministrar líquidos en el miembro superior de ese lado. De igual forma, en el trauma abdominal es
preferible aplicar las soluciones en las venas de los miembros superiores, del cuello o del tórax.

1. Volumen inicial

Al comienzo se suministran de 1.000 a 2.000 mL en e adulto y 20 mL/kg de peso en los pacientes a gran
velocidad y sin riesgo significativo a un paciente en choque hipovolémico sin enfermedad cardiopulmonar
previa es un tercio de la volemia estimada, es decir, el equivalente a la pérdida capaz de producir choque (2,
3, 8, 23). Para acelerar la velocidad del goteo se utilizan la siguiente técnica:

• Usar aguja de calibre 18, 16 ó 14 que son las de mayor diámetro. Recordar que en las agujas
hipodérmicas, a menos número mayor calibre.

• Utilizar agujas cortas. Mientras más corta mayor es la velocidad de flujo.

• Elevar la bolsa o frasco de cristaloides. La velocidad aumenta proporcionalmente a la altura de la


bolsa o frasco.
• Utilizar varias venas simultáneamente. En tal caso es preferible usar extremidades diferentes para
cada venoclisis. Se debe evitar la aplicación de venoclisis en extremidades lesionadas por heridas o
fracturas.

• Aplicar presión sobre la bolsa. Esto se puede realizar manualmente. También existen infusores
comerciales similares al manguito de un tensiómetro para comprimir la bolsa de infusión endovenosa.

• Utilizar un infusor con bomba neumática electromecánica. Algunos tienen calentador incorporado, de
tal forma que es posible obtener infusión rápida y de soluciones tibias. De estas técnicas, las de
mayor eficacia son las agujas de mayor calibre, y el uso de varias venas simultáneas.

Bibliografía:

1. Ferrada R: Manejo Inicial del Traumatizado. En: Echavarría HR, Ferrada R, Kestenberg A:
Urgencia Quirúrgica. XYZ, Cali, 1996.
2. ATLS. Advanced Trauma Life Support. Core course.USA 1994.

86.- El colapso pulmonar completo con desplazamiento de mediastino, disminución del retorno venoso e
inestabilidad hemodinámica en un paciente politraumatizado nos indica:

a) Neumotórax a tensión.
b) Neumotórax abierto.
c) Barotrauma.
d) Neumotórax simple.

NEUMOTÓRAX A TENSIÓN

Se considera "Neumotórax a tensión" cuando la presión intrapleural excede a la presión atmosférica en la


espiración. Se produce cuando una rotura en la pleura provoca un mecanismo valvular de una sola dirección
que se abre en la inspiración permitiendo el paso de aire y se cierra en la espiración. En la inspiración, por la
acción de los músculos respiratorios, la presión pleural se hace negativa y el aire sale desde los alvéolos al
espacio pleural. Durante la espiración, con los músculos respiratorios relajados, la presión pleural se hace
positiva. Esto debe ir acompañado del mecanismo valvular antes mencionado, puesto que si no el aire pasaría
desde el espacio pleural a los alvéolos. Como el aire se sigue acumulando en el espacio pleural, el pulmón
homolateral se comprime y el hemidiafragma se desplaza hacia abajo, el mediastino se desplaza hacia el lado
contralateral y comprime el pulmón contralateral y el corazón. Estos cambios comprometen la ventilación
dando lugar a una importante hipoxemia, debida a grandes shunts a través del pulmón colapsado, seguido de
hipercapnia y acidosis respiratoria, probablemente por hipoxia cerebral, además hay dificultad en el retorno
venoso por la presión pleural positiva que conlleva un descenso en el gasto cardíaco.

Es más frecuente en pacientes sometidos a ventilación mecánica con presión positiva al final de la espiración
o tras resucitación cardiopulmonar. En los pacientes conscientes se manifiesta por intenso dolor que llega a
hacerse sincopal con disnea extraordinariamente intensa, taquipnea, taquicardia, sudoración, cianosis y tos.
Los pacientes pueden tener ingurgitación yugular que puede estar enmascarada si hay deplección de volumen
(sobre todo en traumatizados). En pacientes bajo ventilación mecánica es más difícil el diagnóstico porque el
paciente suele estar sedado y paralizado, puede sospecharse si las presiones pico en la vía aérea aumentan
bruscamente. También debe sospecharse en pacientes que han sido resucitados y que tienen dificultades
para ventilarse o están en disociación electromecánica.

Bibliografía:

– Advanced trauma Life Support 7° Edición, 2004, Colegio Americano de


Cirujanos, Chicago.

87.- ¿Las líneas de Hebra nos ayudan a diagnosticar en adultos la siguiente dermatosis?

a) Herpes Zoster.
b) Escabiosis.
c) Larva migrans.
d) Moluscos contagiosos.

La escabiosis o sarna es una enfermedad de la piel causada por el ácaro parásito Sarcoptes scabiei, llamado
comúnmente arador de la sarnilla. Es una ectoparasitosis de distribución mundial en todas las razas. Es una
afección cosmopolita, extremadamente contagiosa, que se observa en particular en las personas que viajan a
menudo. Alcanza a todas las capas de la población y constituye una dermatosis muy frecuente.

Características Clínicas
Los sitios de predilección son los pliegues interdigitales en manos, caras laterales de dedos, cara anterior de
muñeca, pliegues axilares anteriores, región submamaria, areola, pezones, brazos, antebrazos, caras
internas de muslos, pene, escroto y región ínter glútea.

Las lesiones son pápulas con costras hemáticas debidas al rascado intenso de predominio nocturno. Rara vez
se observan los túneles que produce el parásito. En adultos se respetan las líneas de hebra, mientras que en
lactantes o niños pequeños no sucede esto, siendo en la edad pediátrica un cuadro es generalizado.

Debido al prurito intenso y el rascado que este produce, una complicación frecuente es la impetginización,
mientras que la automedicación es la causa de la dermatitis por contacto que en ocasiones también complica
a esta enfermedad.

Bibliografía:

1. Reyes H. y Neghme A. Sarna, otras acariasis y garrapatas. En: Atias - Negúme, Parasitología clínica.
Tercera edición, 65: 540-552, 1991.
2. Darmstadt G.L. y Lane A. Picaduras de artrópodos e infestaciones parasitarias. En: Nelson. Tratado
de Pediatría. Tercera edición. Tomo 2: 2373-2378, 1997.
3. Tracy J.W. and Webster L.T. Fármacos utilizados en la quimioterapia de las helmintiasis. Capítulo
42, en Goodman y Gilman, Las Bases Farmacológicas de la Terapéutica. Novena edición, 1073-1091,
1996.
4. Koch H. Ivermectina: novel systemic antiparasitic agent. Pharm. Intl., 55-56, 1984.
5. The Medical Letter. Ed. Esp. XIX: 18, 1997.
6. Meinking T.L. et al. The Treatment of Scabies with Ivermectin. N. Engl. J. Med. 333:26-30, 1995.

88.- ¿Cuál es el agente etiológico de una dermatosis localizada en piel cabelluda, caracterizada por múltiples
placas de 1 a 3 cm. de seudoalopecia en un niño de 7 años de edad?

a) Staphilococo dorado.
b) Estreptococo beta hemolítico.
c) Virus herpes tipo II.
d) Trychophyton tonsurans.

La tiña de la cabeza se presenta actualmente en 4 a 10% de las dermatofitosis, predomina en preescolares y


escolares entre un 69 a 90%. La variedad seca se observa en 90% y la inflamatoria en 10%. En la década de
1960 a 1970 era fundamentalmente tricofítica, ahora M. canis ocasiona el 80% y T. tonsurans el 15%; este
último agente se observa en algunas áreas rurales y, sobre todo, en la frontera norte del país [1-2]. En la
Ciudad de México en
un estudio de 125 niños, su frecuencia con respecto a otras tiñas fue de 2%; en el 98% afectó niños con
edad promedio de siete años, en 30% menores de cinco y en 6% mayores de 11 años. Se aislaron M. canis en
78%, T. tonsurans en 17%, T. mentagrophytes en 3%, T. rubrum en 2% y M. gypseum en 0,8%; la variedad
seca predominó con 82% y el querion y granuloma tricofítico con localización cefálica se observaron en 15%
y 2%, respectivamente.

Bibliografía:
– Rev Iberoam Micol 2002; 19: 63-67.

89.- Se reporta una citología cervicovaginal de una mujer de 26 años, madre de un hijo de 4 años, una sola
pareja sexual e inicio de vida sexual a los 20 años y quien se encuentra asintomática es de infección por
Virus de Papiloma Humano. La conducta más apropiada en este caso será:

a) Prueba de Schiller y observación.


b) Colposcopía y toma dirigida de biopsia.
c) Cono terapéutico.
d) Criofulguración.

Ante una citología positiva o indeterminada (ASCUS, AGUS) consideramos el test de cribado positivo y
realizaremos la prueba diagnóstica que es la colposcopia/biopsia. La colposcopia consiste en la visión directa
con lupa binocular de 10-20 aumentos, del cérvix. Se completa con la visión con
filtroverdeytrasañadirácidoacéticoal3%,que tiñe de blanco las lesiones y, posteriormente, el test de
schiller, que consiste en la tinción con una solución yodada de lugol. Las áreas que no captan la tinción son
sospechosas de malignidad y, por lo tanto, se deben biopsiar.
Si no podemos ver totalmente la unión escamocolumnar, consideramos que la colposcopia no es satisfactoria
y entonces necesitaremos realizar un legrado endocervical, es decir, obtener material para analizar de la
zona que no podemos considerar como normal, ya que no la vemos completamente.

Bibliografía:

1. Braunwald E. Kasper D, et al. Harrison: Principios de Medicina Interna. Editorial Mc Graw Hill, 16a
edición. México, 2006. pp 1645.
2. Manual CTO 7ª edición, Ginecología y Obstetricia, pág18.

90.- Masculino de 52 años con padecimiento localizado a glúteo, caracterizada por una placa de 6 x 7 cm
eritematosa con borde pobremente definidos cubierta de escama fina. La biopsia de piel reporta infiltrado
monomorfo de células con núcleos cerebriformes, formando Microabscesos de Pautrier. ¿El diagnóstico más
probable es?:

a) Dermatitis Atópica.
b) Micosis Fungoide.
c) Pitiriasis Rosada.
d) Tiña del Cuerpo.

La micosis fungoide es un linfoma de células T de la piel (cutánea) de bajo grado, representando el 2-3% de
todos los linfomas; el crecimiento de la enfermedad es habitualmente lentamente progresivo y crónico. En
los sujetos con micosis fungoide, el comienzo suele ser con placas persistentes eritemato-descamativas
(parapsoriasis en placas), aunque se han descrito otras formas de inicio (pápulas, pústulas, ampollas,
eritrodermia, lesiones hipopigmentadas o granulomatosis). La piel se infiltra con el tiempo (años), aparecen
placas y nódulos formados por linfocitos atípicos. En los casos avanzados, se pueden presentar tumores
ulcerados e infiltración de los ganglios linfáticos.

La enfermedad se disemina tardíamente a otras zonas del cuerpo, incluyendo el aparato gastrointestinal, el
hígado, el bazo o el cerebro. Histológicamente, consiste en un polimorfo infiltrado celular con
polimorfonucleares, eosinófilos y linfocitos en los que se reconocen células atípicas de mayor tamaño y con
núcleo cerebriforme invadiendo la epidermis (exocitosis), agrupándose en lo que se conoce como
microabscesos de Pautrier.

Otras células atípicas son mayoritariamente linfocitos T cooperadores CD4+. Tres formas clínicas tienen
identidad propia: el síndrome de Sézary, la reticulosis pagetoide y la forma tumoral de inicio. La etiología es
desconocida, barajándose las posibilidades de exposición a carcinógenos, respuesta celular anómala a un
antígeno cutáneo persistente, infecciones por retrovirus (HTLV-10 o similar).
Instituto Nacional del Cáncer.

1. «Información general sobre la micosis fungoide y el síndrome de Sézary» (en español). Consultado el
27 de junio de 2009.
2. Rigel, Darrell S; Robert Friedman, Leonard M. Dzubow, Douglas S. Reintgen, M.D., Jean-Claude
Bystryn y Robin Marks (2006). Cáncer de piel (en español). Elsevier, España, pp. 349-358. ISBN
8481748757.
3. Girardi, M, Heald PW, Wilson LD. (mayo de 2004). «The pathogenesis of mycosis fungoides.». N Engl
J Med. May 350 (19): 1978-88. PMID 15128898. Consultado el 27 de junio de 2009.
4. Ferri, Fred F. (2006). Ferri consultor clínico, 2006-2007: Claves diagnósticas y tratamiento.
Elsevier, España, pp. 550. ISBN 8481749141.
5. GARZONA NAVAS, Dra. Laura, MOREIRA HIDALGO, Dr. Federico, HIDALGO MATLOCK, Dr.
Benjamín et al. Micosis Fungoide: Revisión de tema y presentación de un caso. Rev. costarric. salud
pública. [online]. jul. 2007, vol.16, no.30 [citado 27 Junio 2009], p.46-53. Disponible en la World
Wide Web: [1]. ISSN 1409-1429.
6. Du Vivier, Anthony; Phillip H. McKee (2002). Atlas of clinical dermatology, 3ra edición (en inglés),
Elsevier Health Sciences. ISBN 0443072205.
7. Revista de medicina y cirugia practicas (en español). Publicado en 1911, pág 335, 405-406.
Procedencia del original: la Universidad de California. Digitalizado 16 Abr 2007.

91.- La triada de Beck realizada para el diagnóstico de un tamponade cardiaco en un paciente femenino de
44 años consiste en:

a) Hipotensión, PVC elevada, RsCs disminuidos o velados.


b) Hipotensión, PVC elevada, disminución del tamaño del complejo.
c) Hipotensión, PVC elevada, ingurgitación yugular.
d) Hipotensión, PVC elevada, taquicardia.

El taponamiento cardíaco se define como la compresión del corazón que resulta de la acumulación de líquido
en el saco pericárdico y que produce un severo trastorno hemodinámico. El efecto principal de la compresión
cardíaca es una alteración en el llenado de las cavidades durante la diástole, lo cual lleva a una disminución
del gasto cardíaco y de la presión arterial en un espectro variable en el que el paciente severamente
comprometido puede estar en shock cardiogénico.

•Signos Agudos:-Triada de Beck: Presión arterial baja, elevación de la presión venosa central, Ruidos
cardiacos apagados (corazón quieto)-La injurgitaciónyugular puede estar ausente debido a hipovolemia.-
Paciente estuporoso, agitado-Extremidades frías y húmedas-Taquipnea y taquicardia.
Bibliografía:

1. Cheitlin MD, Abbott JA. Urgencias cardíacas. En: Diagnóstico y Tratamiento de Urgencias. Editado
por MT. Ho y CE Saunders. Editotial El Manual Moderno. México DF, 1991.
2. Hancock EW. Cardiac Tamponade.
3. Med Clin North Am 63:1, 1979.
4. Herrera Márquez JH. Pericarditis. En: Cardiología Fundamentos de Medicina. H Vélez, J Borrero, J
Restrepo editores. Corporación para Investigaciones Biológicas. CIB. Medellín, 1981.

92.- Se trata de femenino de 6 años de edad el cual acude a consulta ya que ingirió 4 tabletas masticables
de paracetamol 300 mg , para tomar medidas en caso de intoxicación ¿Cuál es la dosis tóxica oral de éste
fármaco?

a) 10 mg/kg/dosis.
b) 20 mg/kg/dosis.
c) 40 mg/kg/dosis.
d) 140 mg/kg/dosis.

La dosis oral tóxica en humanos está alrededor de los 10g o 150mg/kg.


Existe riesgo hepatotóxico a partir de una ingesta de 15g aunque se ha descrito alguna muerte a dosis
menores de 10g.

En los últimos años se está comprobando la aparición de hepatotoxicidad ante dosis terapéuticas altas (6
g/día) en población a riesgo.

Biblografía:

1. Bromer MQ, Black M. Acetaminophen hepatotoxicity. Clin Liver Dis 2003;7:351-67.

93.- ¿Cuando detectamos un nódulo tiroideo único, el estudio inicial recomendado es?

a) Biopsia por aspiración con aguja fina.


b) Gammagrama tiroideo.
c) Tomografía computada de cuello.
d) Pruebas de función tiroidea.

El nódulo tiroideo solitario se define como el crecimiento localizado de la glándula tiroides, usualmente es
benigno, la prevalencia es del 4 al 7% en la población general. Aunque el cáncer tiroideo es el tumor
endocrino más común sólo representa el 1% de todos los cánceres y 5% de todos los nódulos tiroideos. El
estudio clínico diagnóstico y terapéutico debe iniciarse con historia clínica completa, exploración física y
exámenes de laboratorio que incluyan un perfil tiroideo para evaluar función de la glándula. La citología
tiroidea por aspiración (CTA) es el principal procedimiento diagnóstico en los pacientes con nódulo tiroideo
solitario, por ser capaz de diferenciar lesiones benignas de las malignas, sus principales ventajas son:
segura, reduce costos de atención médica, selecciona mejor los pacientes que serán sometidos a tratamiento
quirúrgico y se realiza en pacientes ambulatorios.

Bibliografía:

1. Torres AP, Hernández SE, Caracas PN, Serrano GI et al Diagnóstico y tratamiento del nódulo
tiroideo. Rev Edocrinol Nutr 2000; 8 (3): 87-93.

94.- Masculino de 70 años, diagnosticado de carcinoma de próstata, en este momento acude al servicio de
urgencias por presentar confusión mental, náuseas, vómitos y estreñimiento. Se realizan pruebas de
laboratorio y destaca una calcemia de 16mg/Dl. ¿Cuál es, la primera medida terapéutica que debería tomar
ante éste paciente?

a) Hormonoterapia (leuprolide y estrógenos).


b) Difosfonatos por vía oral.
c) Glucocorticoides por vía intravenosa.
d) Administración de Solución salina y furosemida por vía intravenosa.

Los síntomas que provoca la hipercalcemia están en relación con sus niveles en sangre. Valores comprendidos
entre 10,5 y 12 gr/dl no suelen provocar síntomas; a partir de estos valores la sintomatología es progresiva,
afectando con más intensidad y gravedad a todos los sistemas del organismo. No está establecido un orden
de aparición de la sintomatología, ni su correspondencia con los valores de calcemia.

Síntomas de la hipercalcemia:

Sistema nervioso central


Desórdenes mentales.
Dificultades cognitivas.
Ansiedad.
Depresión.
Confusión, estupor y coma.
Calcificación corneal.
Suicidios (descritos aisladamente).
Sistema neuro-muscular
Fatiga o cansancio muscular.
Mialgias.
Descenso de la función de músculos respiratorios.
Laxitud articular.
Sistema renal
Nefrolitiasis.
Diabetes insípida nefrogénica (poliuria y polidipsi).
Deshidratación.
Nefrocalcinosis.
Sistema gastrointestinal
Náuseas y vómitos.
Anorexia.
Estreñimiento.
Dolor abdominal.
Pancreatitis.
Úlcera péptica.
Sistema esquelético
Dolor óseo..
Artritis.
Osteoporosis.
Osteítis fibrosa quística.
Resorción subperióstica.
Quistes óseos.
Embarazo
Hipoparatiroidismo neonatal.
Tetania neonatal.
Bajo peso al nacer.
Retraso crecimiento intrauterino.
Hiperemesis gravídica.
Alta morbilidad neonatal y materna.
Partos pretérmino.
Sistema cardiovascular
Hipertensión arterial.
Calcificación vascular.
Calcificación miocárdica.
Hipertrofia miocárdica.
Acortamiento intervalo QT.
Arritmias cardíacas.
Otros
Queratitis.
Conjuntivitis.
Anemia normocítica normocrómica.
Gota o pseudogota.

El tratamiento de la hipercalcemia está orientado fundamentalmente a eliminar la causa que la produce. En


los casos sintomáticos se requiere de un tratamiento inicial específico. Debe ser tratada la que presenta
síntomas o supera los 14 mg/dl. Los mecanismos para actuar sobre la hipercalcemia son:

Aumento de la eliminación renal de calcio.


Disminución de la absorción intestinal de calcio.
Disminución de la resorción ósea de calcio.

El tratamiento siempre debe iniciarse con la rehidratación del paciente, que produce un aumento del volumen
extracelular, alcanzando una ganancia de volumen de 1,5- 2,5 litros en las primeras 24 horas.

A continuación, se utilizará un diurético de asa, tipo furosemida, que aumenta la excreción renal de sodio y
calcio. La dosis de diurético se ajustará en función de las cifras de calcio a lo largo del tratamiento.

Cuando la función renal está comprometida puede recurrirse a la hemodiálisis con calcio bajo en el líquido de
diálisis.

En los casos en los que esté implicado un aumento de la resorción ósea como causa de hipercalcemia,
deberemos controlarla con:

Bisfosfonatos: disponemos de clodronato y pamidronato. El más empleado es el primero, porque la


dosificación es más fácil. Su administración en el caso de la hipercalcemia es intravenosa, inicia el
efecto a las 72 horas y alcanza el máximo a la semana.
Calcitonina: actúa más rápido que los bisfosfonatos, pero sólo mantiene el efecto durante 72 horas.
Otros fármacos antirresortivos: Mitramicina y el nitrato de galio, con menor experiencia, pueden
usarse en caso de fracaso de los anteriores.

En aquellos casos en los que está aumentada la absorción intestinal de calcio, como en la producción
endógena de vitamina D (enfermedades granulomatosas o linfomas) deben tratarse con glucocorticoides.

95.- Se trata de femenino de 31 años, con un cuadro clínico de sangrado menstrual irregular, dismenorrea,
dispareunia, y una esterilidad de 3 años de evolución, con resultados de laboratorio hormonal normal y un
estudio ecográfico transvaginal que informa de un útero normal y sendas formaciones quística ováricas
bilaterales de 4 cms con signos ecográficos de sospecha. ¿Cuál sería la orientación diagnóstica?:

a) Hemorragia uterina disfuncional.


b) Síndrome del ovario poliquístico.
c) Endometriosis.
d) Quistes dermoides bilaterales.

Causas y síntomas de endometriosis

Las causas de la endometriosis aún no se conocen. Las células del revestimiento interno del
útero de alguna manera se desplazan hasta zonas externas al mismo y siguen creciendo. Este
desplazamiento podría quizás deberse a que pequeños fragmentos del revestimiento uterino,
desprendidos durante la menstruación, retrocedan hacia las trompas de Falopio en dirección a
los ovarios hasta entrar en la cavidad abdominal, en lugar de salir con el flujo menstrual a
través de la vagina.
La endometriosis causa dolor en la parte inferior del abdomen y la zona pélvica, irregularidades
menstruales (como manchar antes de la menstruación) e infertilidad. Algunas mujeres con
endometriosis grave no presentan síntomas, mientras que otras con la enfermedad en grado
mínimo sufren un dolor invalidante. Con frecuencia, el dolor menstrual debido a la endometriosis
no aparece hasta años después de desarrollar la enfermedad. En algunos casos, se constata
dolor durante el coito (dispareunia), antes o durante la menstruación.
El tejido endometrial adherido al intestino grueso o a la vejiga urinaria puede provocar
hinchazón abdominal, dolor durante las deposiciones, hemorragia rectal durante la menstruación
o dolor en la parte inferior del abdomen durante la micción. Así mismo, cuando el tejido se
localiza en un ovario o una estructura cercana puede dar lugar a la formación de una masa llena
de sangre (endometrioma). En ocasiones, el endometrioma se rompe bruscamente o se escapa
algo de su contenido, lo que causa un agudo y repentino dolor abdominal.

96.- Mujer de 25 años de edad que presenta en la cara diseminado comedones, pápulas y pústulas,
crecimiento excesivo de vello en mejillas mentón y cuello, acompañada de seborrea, refiere caída de pelo
exagerada. Inició desde la adolescencia. El diagnóstico clínico es:

a) Acné e Hirsutismo.
b) Síndrome de masculinización.
c) Lupus eritematoso discoide.
d) Alopecia androgenética.

El hirsutismo es el crecimiento excesivo de vello terminal en mujeres siguiendo un patrón masculino de


distribución, en zonas andrógeno-dependientes: patillas, barbilla, cuello, areolas mamarias, tórax, en área
inmediatamente superior o inferior al ombligo, así como en muslos, espalda. Frecuentemente se asocia a
acné, calvicie con patrón masculino (alopecia androgénica) e irregularidades menstruales.

Es un trastorno que afecta aproximadamente al 10% de las mujeres en edad fértil, y puede ser leve, lo que
representa una variación del patrón de crecimiento normal, y en raras ocasiones es signo de un trastorno
subyacente grave.
Por lo general, es idiopático, pero puede estar relacionado al exceso de andrógenos, como el síndrome de
ovario poliquístico o la hiperplasia suprarrenal congénita.

Los pacientes con acné presentan de un modo más o menos frecuente seborrea de cuero cabelludo, con o sin
pitiriasis.

Otras alteraciones andrógeno-dependientes pueden asociarse, pero no de un modo habitual, como sucede
con el hirsutismo y la alopecía, que pueden presentarse en menos del 10% de las pacientes.

En el Síndrome de Cushing Hipofisario, la ACTH estimula la producción suprarrenal de hormonas andro-


génicas, resultando esto en acné e hirsutismo.

En el síndrome de poliquistosis ovárica (Stein-Leventhal), hay un incremento de la secreción gonadal de


andrógenos a nivel del hilio y de la teca interna. La hipersecreción de andrógenos dotados de bioactividad
puede inducir acné e hirsutismo. La presencia de acné en una mujer -adolescente o adulta - con niveles de
testosterona sérica superiores a 300 ng/dl sugiere la presencia de un tumor ovárico.

En la Anorexia Nerviosa, el acné se desencadena generalmente en la fase de recuperación, donde puede


coexistir con un ovario poliquístico. En fase de amenorrea LHRH, LH, FSH, estrógenos y progesterona están
disminuídos. El cortisol plasmático está elevado con vida media prolongada y pérdida de la variación diurna,
otro factor que puede producir acné.

No hay que olvidar aquellos síndromes donde el acné coexiste con importantes trastornos osteoarti-culares,
como ser el Síndrome de Apert y el Síndrome SAPHO.

El acné se halla generalmente presente en los pacientes epilépticos - tal vez provocado en gran medida por
la medicación - y en pacientes psiquiátricos (maníaco-depresivos y obsesivo-compulsivos). Es importante
tener en cuenta la asociación de una neurosis de angustia inespecífica y el acné.

Bibliografía:
1. Azziz R, Carmina E, Sawaya ME. Idiopathic hirsutism. Endocr Rev 2000;21:347-62.
2. Hirsutismo. Gonzalez Guerra. Servicio de dermatología.Fundación Jiménez Díaz (Madrid).

97.- Una niña de 2 años de edad presenta lesiones rojas, con costras melicéricas en la cara, cuero cabelludo,
área de pañal y extremidades, desde aproximadamente los 2 meses de edad. El evitar sustancias irritantes
no ha funcionado en el tratamiento. La niña se rasca constantemente las áreas afectadas. Hay una historia
familiar positiva para asma y fiebre. ¿Cuál es el diagnóstico más probable?

a) Dermatitis atópica.
b) Celulitis.
c) Dermatitis de contacto.
d) Liquen cimple crónico.
La dermatitis atopica (DA) (Wise-Sulzberger, 1993), llamada neurodermatitis diseminada, por las escuelas
Europea, es un estado reaccional de la piel, intensamente pruriginosa, más frecuente en los niños,
multifactorial, en la que combinan factores constitucionales y factores ambientales, por lo tanto de difícil
tratamiento, muy frecuente en la consulta diaria del pediatra y del dermatólogo.

DATOS EPIDEMIOLOGICOS
La enfermedad originalmente conocida como prúrigo de Besnier y eccema constitucional, es ubicua, afecta
a todas las razas y existe en todos los países. Se señala su existencia hasta en el 2% de la población general
y en el 14% de los niños. La enfermedad se inicia antes del primer años de la vida en más del 60% de los
casos, la curva desciende hacia los 12 años de la cifra de inicio es apenas del 5%-y es excepcional que la
enfermedad se inicie en la edad adulta. La enfermedad sufre exacerbaciones en las temporadas de calor o
de frío cuando hay sequedad de la atmósfera.

CUADRO CLÍNICO
Tradicionalmente se han descrito tres etapas que en la actualidad ya no se presentan pues se suman unas a
otras debido a los tratamientos que reciben los pacientes desde su inicio.
Etapa de lactante. La enfermedad suele iniciarse en los primeros meses de vida, a veces casi desde el
nacimiento. Las lesiones afectan la cara: mejillas, frente (respetando su centro); afectan también la piel
cabelluda, los pliegues, las nalgas y el dorso de los pies. Las lesiones son de tipo eccematosos (piel llorosa):
eritema, vesícular y costras melicéricas con costras hemáticas como signo del rascado. Estas lesiones
altamente pruriginosas evolucionan por brotes y en general si el paciente no es yatrogénicamente tratado,
tienen tendencia a involucionar al año de edad.
Fase del escolar. Se inicia hacia los 3 a 7 años, cuando el niño empieza a ir a la escuela. Las lesiones son
preferentemente flexurales: cuello, pliegues de codo y huecos poplíteos y la morfología corresponde a la de
una dermatitis crónica: zonas de eritema y liquenificación (índice de rascado crónico) y costras hemáticas
periódicamente sufren un proceso de eccematización sobre todo por los tratamientos indebidos. La
enfermedad evoluciona por brotes, hay temporadas en que no existen lesiones aun cuando persiste a veces
una piel xerótica (seca) y pigmentada con intenso prurito.

Etapa del adulto. Se veía con poca frecuencia y ahora es habitual en las consultas diarias debido a los malos
tratamientos. En este caso además de las zonas flexurales, se presenta lesiones periorbitarias; y
peribucales y lesiones vesiculosas en las manos. Son lesiones tanto liquenificadas como eccematosas, muy
pruriginosas, que alternan con periodos asintomáticos.

Con el uso inmoderado de los corticoesteroides no hay diferenciación entre las etapas de la enfermedad y
se ven casos que arrastran su enfermedad casi desde que nacen hasta la edad adulta.

DIAGNÓSTICO
Es clínico y relativamente sencillo, aunque no toda dermatitis flexural es necesariamente de origen atópico.
Deben tomarse en cuenta los antecedentes del propio enfermo y los familiares. En los lactantes debe
diferenciarse de la dermatitis seborreica que afecta sobre todo la piel cabelluda y el centro de la cara. En
ocasiones hay mezcla de las dos dermatitis: la atópica y la seborreica y es difícil diferenciarlas. La
dermatitis del pañal predomina en los lactante; en las zonas glúteas y genitales, se presenta con eritema y
lesiones vesículopapulosas muy pruriginosas. La pitiriasis alba corresponde a los clásicos “jiotes” de los niños:
manchas hipocrómicas cubiertas de fina escama en la cara.

Estos procesos pueden coincidir con manifestaciones de DA, pero nunca se ha demostrado la relación de
causa efecto.

TRATAMIENTO
Medidas generales. Es conveniente una explicación amplia al paciente y su familia señalando la naturaleza y
evolución de la enfermedad y lo que se espera del tratamiento que vamos a indicar.

Evitar el sol excesivo, el agua clorada de las albercas, el uso de jabón (indicar jabones neutros o sustitutos
de jabón), prohibir el uso de pomadas y remedios caseros así como los cosméticos que irritan a la piel.

Las dietas restrictivas han sido y siguen siendo un tema muy controversial, pues mientras los alergistras
siguen insistiendo sin muchas bases en el beneficio de estas dietas, los más hemos comprobado su inutilidad.
Existe un grupo muy limitado en que se puede demostrar que una dieta restrictiva de huevo, leche, fresas,
etc. puede mejorar los brotes de DA y en tales casos (sólo en esos casos) se aconsejaría la supresión de
tales medicamentos; en lo general se permite al paciente que coma de todo. En la actualidad se están
limitando por los pediatrias ciertos alimentos como el huevo, las fresas, el plátano y la leche de vaca en el
primer año de vida por ser muy alergénicos y se discute la utilidad de la leche materna en estos niños.
Es necesario hacer ver que estos pacientes requieren de una atención más personal, que sienten la necesidad
de cariño por lo que es de recomendarse que el niño sea atendido personalemente por la madre.

Tratamiento tópico. Depende del estado de la piel; si está eccematosa debe secarse antes de aplicar
cualquier pomada que será rechazada por la piel llorosa. El uso de fomentos con agua de manzanilla o suero
fisiológico es útil. Los fometos con agua de végeto (subacetato de plomo) al 20% son potentes
antiexudativos, pero deben limitarse a áreas restringidas y no usarse en niños. Estando la piel ya seca, se
usan pasta inertes que llevan óxido de zinc y calamina en una base de vaselina y lanolina que son protectoras
a la vez que antiprutiginosas. Si la piel está muy seca y liquenificada, los fomentos y baños serán emolientes,
con almidón y aceites seguidos de cremas o pomadas más grasosas que llevan vaselina, coldcream y óxido de
zinc. En el comercio existen numerosos preparados humectantes que ayudan a mantener el manto ácido
grasa-agua, que se pierde en la DA. Si hay dermatitis por contacto o impétigo hay que tratar primero estas
complicaciones con sulfato de cobre al 1: 1000 y pomadas con vioformo o mupirocín. En el caso de
eritrodermia, los baños emolientes y el uso del petrolato (vaselina) será lo indicado.

Cuando hay mucha liquenificación, pueden usarse cremas con alquitrán de hulla al 3% en base de coldcream
por tiempo y zonas limitadas.

Los corticoesteroides tópicos son los medicamentos más usados en esta enfermedad y muchas veces causa
de las complicaciones que se presentan. Estos medicamentos no curan nada, solo engañan al paciente y al
médico haciéndoles creer que la enfermedad va curando cuando sólo se oculta y modifica. Al pasar el efecto
de estos medicamentos, invariablemente se presenta el rebote y más tarde la corticodependencia, haciendo
a la enfermedad incontrolable. Nunca deben usarse los corticoesteroides fluorinados en niños y en la cara y
zonas genitales o en los pliegues por su posibilidad de absorción. La hidrocortisona es de baja potencia, hace
menos daño, pero también es menos efectiva; algunos la recomiendan.

Tratamiento sistémico. El uso de antihistamínicos sobre todo de la primera generación que son sedantes
como la clorfeniramina y la hidroxicina ayudan a mejorar el prurito; el ketotifeno y la oxotamida por su
acción dual: inhiben la producción de histamina por los mastocitos y bloquean los receptores Hl, también son
de ayuda, al igual que los sedantes suaves tanto para el paciente como para la madre que está en perenne
angustia que transmite al pequeño paciente. La talidomida ha mostrados ser de ayuda en casos de DA
conticoestropeada, en su fase eritrodérmica, no tanto en los niños, a dosis de 100 mg al día. Los antibióticos
tipo dicloxacilina, serán necesarios cuando haya infección o simplemente eccematización por el papel que
tiene el estafilococo dorado.

Bibliografía:

1. Criterios Diagnóstico para Dermatitis Atópica (DA), MedicalCriteria.com.


2. Dr. Luciano Domínguez-Soto, Dr. Amado Saúl Cano, Dermatitis atópica o neurodermatitis
diseminada, Dermatología, Parte C, Libro ) "

98. -Se trata de paciente femenino de 52 años de edad a la cual se le diagnostica neumonía por aspiración,
aislando en esputo bacilo Gram negativo anaerobio, en su historia clínica niega alergia medicamentosa ¿Cuál
es el tratamiento antimicrobiano de elección?:

a) Metronidazol.
b) Eritromicina.
c) Ceftriaxona.
d) Ciprofloxacino.

NEUMONIA POR ASPIRACION

La aspiración de contenido gástrico ocurre con relativa frecuencia en pacientes severamente enfermos. Los
factores predisponentes incluyen alteraciones mentales, anestesia general, traqueostomía e intubación
nasogástrica. Las consecuencias de la aspiración dependen de la naturaleza del contenido gástrico:

a. Entre mayor sea el volumen aspirado, mayor será el daño causado


b. La acidez gástrica causa inflamación con exudación, disminución de surfactante y mayor riesgo de
atelectasis
c. Las partículas alimenticias inician una reacción granulomatosa
d. Las bacterias de las secreciones orofaringeas complican la respuesta inflamatoria

La aspiración es causa de fiebre y de la aparición de infiltrados pulmonares. La presencia de broncoespasmo


e hipoxemia sugieren el diagnóstico que evoluciona hacia un cuadro de neumonitis y una infección bacteriana.
La consolidación aparece rápidamente acompañada de tos productiva y esputo purulento. Los cultivos de
esputo muestran en el 63% de los casos flora aeróbica, en el 7% anaerobios y en el 30% flora mixta.

ANAEROBIOS
Los anaerobios son la flora predominante en las secreciones orafaringeas. Por lo tanto, la vía de infección
por estos gérmenes es la aspiración. Existen cuatro patrones de infección: 1) neumonitis aguda; 2) neumonía
necrotizante; 3) absceso pulmonar y 4) empiema. Todos están relacionados entre sí puesto que una neumonía
aguda si no se trata, evoluciona a un proceso necrotizante que se puede localizar, dando origen a un absceso
que al romperse hacia el espacio pleural produce un empiema.

Hallazgos clínicos. Tos productiva con esputos de mal olor, dolor pleurítico; si existe un absceso la
respiración es anfórica o cavernosa.

Hallazgos de laboratorio. Una muestra obtenida por punción transtraqueal permite su cultivo en un
ambiente para anaerobiosis. Para obtener un resultado óptimo la muestra se debe llevar al laboratorio en
jeringas sin aire. Las infecciones pulmonares anaeróbicas que aparecen en pacientes hospitalizados
generalmente son de tipo mixto y con frecuencia se encuentran Staphylococcus o bacilos gram-negativos.
Los microorganismos más importantes son el Bacteroides melaninogenicus y el Fusobacterium nucleatum y
los estroptococos anaeróbicos propios de la flora existente en la orofaringe. En el caso de enfermedades
abdominales como la obstrucción abdominal o cirugía, una causa frecuente de infección es el B. fragilis.

Tratamiento. La mayoría de los microorganismos, con excepción del B. fragilis son sensibles a la penicilina G.
y la carbenicilina. Casi todos, incluyendo el B. fragilis son sensibles al cloranfenicol y a la clindamicina. El
metronidazol es eficaz contra la mayoría de los anaeróbicos incluyendo del B. fragilis. En la neumonía aguda
se recomienda la penicilina cristalina 2 millones de unidades IV cada 4 horas; en la neumonía necrotizante y
el absceso pulmonar se prescribe penicilina cristalina más metronidazol. Es aconsejable practicar una
fibrobroncoscopia ante la presencia de un absceso para descartar un cuerpo extraño o una lesión maligna. En
el empiema anaeróbico, la maniobra terapeútica más importante es el drenaje adecuado por medio de un
tubo torácico. Se utilizan dosis altas de penicilina cristalina (3 millones de unidades cada 4 horas), más
clindamicina 600 mg IV cada 6 horas o metronidazol, 15 mg/kg/6 horas, IV.

Bibliografía:

– Des Jardins T. Neumonía. En: Enfermedades Respiratorias. Editorial El Manual Moderno. México
D.F., 1993.
– Glackman RA. Oral empirical treatment of pneumonia. The Challenge of Choosing the Best Agent.
Postgrad Med 95:165, 1994.
– Jauregui A. Infecciones de vías respiratorias superiores y neumonías. En: Manual de Terapéutica
Médica. Instituto Nacional de la Nutrición Interamericana. Mc Graw Hill. México D.F., 1994.
– La Force-FM. Antibacterial therapy for lower respiratory tract infections in adults: a review.Clin
Infect Dis. 14 Suppl 2:S233; discussion S244, 1992.
– Londoño F. Neumonías. En: Neumología. Editado por Jorge Restrepo M, Darío Maldonado G.
Fundamentos de Medicina, Corporación para Investigaciones Biológicas. Medellín, 1986.
– Tobin M. Diagnosis of pneumonia, techniques and problems. Clin Chest Med 8:513, 1997.

99.- Un paciente de 36 años de edad acude a consulta por presentar nausea y vómito, aumento en el número
de evacuaciones de consistencia aguada, (diarrea sin productos patológicos), afebril, T.a 110.70 mmhg,
refiere que su hijo presentó los misma sintomatología casi al mismo tiempo que el, como antecedente
refiere haber comido juntos en un restaurante hace aproximadamente 4 hrs ¿De los siguientes
microorganismos cual es el de mayor probabilidad de ocasionar éste cuadro?

a) Salmonella enteriditis.
b) Shigella sonnei.
c) Staphylococcus aureus.
d) E. Coli.

La contaminación de alimentos por S. aureus, está asociada con una forma de gastroenteritis que se
manifiesta clínicamente por un cuadro caracterizado por vómitos (76% de casos) y diarrea (77% de casos).
El corto período de incubación de 1-6 horas orienta a la sospecha de enfermedad producida por ingestión de
una o más enterotoxinas preformadas en el alimento que ha sido contaminado con cepas de S. aureus
productor de la misma.
Son raramente observados signos de toxicidad sistémica, tales como fiebre e hipotensión
En general, es un cuadro autolimitado que típicamente se resuelve en 24-48 horas desde el inicio.
No está claro si se desarrolla en humanos inmunidad a largo plazo, pero anticuerpos frente a una SE no
necesariamente confieren inmunidad frente a la intoxicación por S. aureus, ya que existe múltiples SE
capaces de producir enfermedad. En algunos casos, anticuerpos producidos frente a una SE confieren
protección cruzada contra otra SE, ya que algunas comparten epítopes.
Todas las SE son capaces experimentalmente en primates de producir emesis, y no se registra
enterotoxemia, excepto en dosis muy altas, probablemente debido a su dificultad para atravesar mucosas.
El 99% de casos de intoxicación alimentaria por enterotoxinas
Estafilocóccicas está asociado a S. aureus y ocasionalmente se reportan casos por Staphylococcus
epidermidis.
Las cepas estafilocóccicas enterotoxigénicas aisladas de alimentos implicados en brotes de infección son
mas a menudo lisadas por fagos del grupo lll, y menos frecuentemente simultáneamente por los grupos l y lll.

Tratamiento.
Como para la mayoría de enfermedades trasmitidas por alimentos autolimitadas, las medidas de sostén son
la base del tratamiento. No está indicado tratamiento con antimicrobianos.

Referencias bibliográficas.
- Manual of Clinical Microbiology. Murray, P. 1995 6th edition.
- Principles and Practice of Infectious Diseases. Mandell, Douglas, Bennett.
1995. 4th edition.
- Dinges M, Orwin P, Schlievert P. 2000. Exotoxins of Staphylococcus aureus.
Clinical Microbiology Reviews, vol 13; 16-34.
100.- Masculino de 30 años que fue atropellado por vehículo automotor ingresado hace 3 días por fractura
subtrocantérea de fémur izquierdo y fractura diafisaria conminuta de tibia izquierda y fractura de clavícula
izquierda que se inmovilizaron provisionalmente en espera de cirugía de osteosíntesis. Inicia un cuadro
brusco de estupor y obnubilación intensos acompañado de disnea y de aparición de petequias difusas. ¿Cuál
será su sospecha clínica?

a) Shock neurogénico por dolor.


b) Síndrome de embolia grasa.
c) Neumonía nosocomial por estasis.
d) Shock hipovolémico.

El síndrome del embolismo graso se produce por las fracturas de los huesos largos. La presentación clásica
consiste en un intervalo asintomático seguido de manifestaciones pulmonares y neurológicas, combinadas con
hemorragias petequiales. El síndrome sigue un curso clínico bifásico. Los síntomas iniciales son
probablemente causados por un mecanismo de oclusión de múltiples vasos sanguineos por glóbulos grasos que
son de demasiado tamaño para pasar a través de los capilares. El otro acontecimiento de la embolia, es que la
oclusión vascular del embolismo graso es frecuentemente temporal o incompleta, porque los glóbulos grasos
no obstruyen completamente el flujo de sangre de los capilares por su fluidez y deformidad. La presentación
tardia se piensa que es el resultado de la hidrólisis de la grasa y la irritación de los ácidos grasos libres que
emifgran a otros órganos por la circulación sistémica.

ESCALA DE SHIER PARA EL RIESGO DE SEG

Shier MR, Wilson RF, James RE, Riddle J, Mammen EF, Pedersen HE. Fat embolism
prophylaxis: a study of four treatment modalities. J Trauma 1977; 17 (8): 621-629.
Lugar de la fractura Puntuación
Cabeza del fémur 2
Cuerpo del fémur 4
Pelvis 2
Tibia 2
Húmero 2
Radio 1
Peroné 1
Cúbito 1
La puntuación total es la suma de los puntos de cada fractura.

CRITERIOS DIAGNÓSTICOS DE SEG

Schonfeld SA, Ploysongsang V, Dilisio R, Crisman JD, Miller E, Hammerschmidt DE, Jacob HS.
Fat embolism with corticosteroids. Ann Intern Med 1983; 99: 438-443.
Síntoma/signo Puntuación
Petequias 5
Infiltrados alveolares difusos 4
Hipoxemia* 3
Confusión 1
Fiebre** 1
Taquicardia*** 1
Taquipnea**** 1
(*) presión arterial de O2 <70 mm Hg; (**) temperatura >38ªC; (***) frecuencia cardíaca >120
lpm; (****) frecuencia respiratoria >=30 rpm.

Referencias Bibliográficas:

– Capan LM, Miller SM, Patel KP: Anesth Clin N Amer, 11:1 (Mar), 1993.
– Gossling HR, Ellison LH, Degraff AC: Fat embolism: The role of respiratory failure and its
treatment. J Bone Joint Surg 56A: 1327, 1974.
– Gossling HR, Pellegrini VD: Fat embolism syndrome: A review of the pathophysiology and physiologic
basis of treatment. Clin Orthop 165:68, 1982.
– Peltier LF: The diagnosis and treatment of fat embolism. J Trauma 11:661

S-ar putea să vă placă și